Transcript
Page 1: Bedtime gastroenterology

BedtimeGastroenterology

Page 2: Bedtime gastroenterology
Page 3: Bedtime gastroenterology

BedtimeGastroenterology

Manoj K GhodaMD MRCP (England)

Consultant GastroenterologistMahashakti

Stadium Circle, NavrangpuraAhmedabad (Gujarat)

e-mail: [email protected]

JAYPEE BROTHERSMEDICAL PUBLISHERS (P) LTD.

NEW DELHI

Page 4: Bedtime gastroenterology

Published by

Jitendar P VijJaypee Brothers Medical Publishers (P) LtdB-3 EMCA House, 23/23B Ansari Road, DaryaganjNew Delhi 110 002, India,Phones: +91-11-23272143, +91-11-23272703, +91-11-23282021,+91-11-23245672, Rel: 32558559, Fax: +91-11-23276490, +91-11-23245683e-mail: [email protected] our website: www.jaypeebrothers.com

Branches

2/B, Akruti Society, Jodhpur Gam Road SatelliteAhmedabad 380 015, Phones: +91-079-26926233, Rel: +91-079-32988717Fax: +91-079-26927094, e-mail: [email protected]

202 Batavia Chambers, 8 Kumara Krupa Road, Kumara Park EastBengaluru 560 001, Phones: +91-80-22285971, +91-80-22382956, Rel: +91-80-32714073,Fax: +91-80-22281761, e-mail: [email protected]

282 IIIrd Floor, Khaleel Shirazi Estate, Fountain Plaza, Pantheon RoadChennai 600 008, Phones: +91-44-28193265, +91-44-28194897, Rel: +91-44-32972089Fax: +91-44-28193231, e-mail: [email protected]

4-2-1067/1-3, 1st Floor, Balaji Building, Ramkote Cross RoadHyderabad 500 095, Phones: +91-40-66610020, +91-40-24758498, Rel:+91-40-32940929Fax:+91-40-24758499, e-mail: [email protected]

No. 41/3098, B & B1, Kuruvi Building, St. Vincent RoadKochi 682 018, Kerala, Phones: 0484-4036109, +91-0484-2395739, +91-0484-2395740,e-mail: [email protected]

1-A Indian Mirror Street, Wellington SquareKolkata 700 013, Phones: +91-33-22451926, +91-33-22276404, +91-33-22276415Rel: +91-33-32901926, Fax: +91-33-22456075, e-mail: [email protected]

106 Amit Industrial Estate, 61 Dr SS Rao Road, Near MGM Hospital, ParelMumbai 400 012, Phones: +91-22-24124863, +91-22-24104532, Rel: +91-22-32926896,Fax: +91-22-24160828, e-mail: [email protected]

“KAMALPUSHPA” 38, Reshimbag, Opp. Mohota Science College, Umred RoadNagpur 440 009 (MS), Phones: Rel: 3245220, Fax: 0712-2704275,e-mail: [email protected]

Bedtime Gastroenterology

© 2007, Manoj K Ghoda

All rights reserved. No part of this publication should be reproduced, stored in a retrieval system, ortransmitted in any form or by any means: electronic, mechanical, photocopying, recording, orotherwise, without the prior written permission of the author and the publisher.

This book has been published in good faith that the material provided by author is original. Everyeffort is made to ensure accuracy of material, but the publisher, printer and author will not be heldresponsible for any inadvertent error(s). In case of any dispute, all legal matters are to be settledunder Delhi jurisdiction only.

First Edition: 2007ISBN 81-8448-125-XTypeset at JPBMP typesetting unitPrinted at Ajanta Offset

Page 5: Bedtime gastroenterology

I always found didactic textbook description of disease veryboring, as it does not relate to how the patient presents to aclinician. In real life patient presents with various complaints,which are at times very confusing and often misleading ifnot taken in context. This may lead to unnecessary delay indiagnosis with its consequences, and unnecessary expense,which is not desirable even if the patient is affording.Abdomen particularly poses extra problems as it accommo-dates a large number of organs, and each one can go wrongin different ways. A good first hand description of illnessand good logical deduction helps to come to a conclusion,which, otherwise, would take a long time and expenses.Here I describe few cases, as they were seen, and ourthinking behind solving them. As they were collected overseveral years you will notice how investigating and treatmentmodalities have changed over a time.

Manoj K Ghoda

Preface

Page 6: Bedtime gastroenterology
Page 7: Bedtime gastroenterology

Contents

CASE 1: A Case of Recurrent Jaundice ........................... 1

CASE 2: A 3-year-old with Huge Liver and AbnormalLiver Functions ..................................................... 4

CASE 3: A Case of Constipation and Bleeding PR ........... 6

CASE 4: A Case of Pancreatitis and FatalHaematemesis ..................................................... 7

CASE 5: A Case of Painful Swallowing and Left PleuralEffusion ................................................................. 9

CASE 6: A Case of Hemiplegia Followed byProgressive Jaundice ........................................ 11

CASE 7: A Case of Diarrhoea and Failure to Thrive ....... 13

CASE 8: A Case of Persistent Pain AfterCholecystectomy ............................................... 15

CASE 9: A Case of Severe Itching and JaundiceFollowing Pregnancy .......................................... 17

CASE 10: A Young Girl with Abdominal Pain andLymphadenopathy ............................................. 19

CASE 11: A Case of Abdominal Pain and Weight Loss .... 21

CASE 12: Middle Aged Woman with Recurrent Vomiting . 23

CASE 13: An Alcoholic with Mass in the Liver .................. 24

CASE 14: A 15-year-old Boy with Prolonged Jaundice .... 26

CASE 15: A Case of Recurrent Chest Pain andDysphagia ........................................................... 28

Page 8: Bedtime gastroenterology

CASE 16: Operation Room Assistant with Right SidedLower Chest Pain ............................................... 30

CASE 17: A Middle Aged Man with Past History ofVagotomy and Diarrhoea ................................... 32

CASE 18: An 18-year-old with Fever and Diarrhoea ........ 34

CASE 19: An Old Man with Haematemesis and Melena .. 36

CASE 20: A 30-year-old Lady with Haematemesis .......... 39

CASE 21: A Case of Sudden Onset of Bloody Diarrhoea 41

CASE 22: A Student Nurse with Fever and Diarrhoea ..... 43

CASE 23: Never Ending Diarrhoea in a Middle Aged Man 45

CASE 24: Sudden Onset of Excruciating Right UpperQuadrant Pain ..................................................... 48

CASE 25: An Old Man with Sudden Enlargement ofLiver and Ascites ............................................... 51

CASE 26: A Case of Sudden Onset of Left SidedColicky Pain ........................................................ 53

CASE 27: A Young Girl with Recurrent Breathlessnessand Abdominal Pain ........................................... 55

CASE 28: A Young Boy with Recurrent Pancreatitis ........ 57

CASE 29: Bank Executive Who Spent Sleepless NightOver a Cystic Lesion in the Liver ...................... 59

CASE 30: Sore Mouth and Throat and Difficulty inSwallowing .......................................................... 60

CASE 31: Chronic Left Sided Abdominal Pain .................. 61

CASE 32: An Unmarried Man with Odynophagia andDysphagia and Unexplained Diarrhoea ........... 62

CASE 33: A Middle Aged Man with Weight Loss anda Giant Gastric Ulcer ......................................... 64

viii BEDTIME GASTROENTEROLOGY

Page 9: Bedtime gastroenterology

CASE 34: A Judge Who could not Perform his Dutydue to Extreme Tiredness ................................. 66

CASE 35: Primipara with Jaundice in Last Trimester ....... 68

CASE 36: Man with Recurrent Post-prandial Vomiting ..... 71

CASE 37: A Case of Recurrent Duodenal Ulceration ...... 73

CASE 38: A Menopausal Lady with Jaundice ................... 74

CASE 39: A Young Girl with Recurrent Jaundice andVague RUQ Mass ............................................... 76

CASE 40: A Young Boy with Recurrent IntestinalObstruction ......................................................... 78

CASE 41: An Old Lady with Diarrhoea and SuddenIncrease in the Size of a Swelling in her Neck . 80

CASE 42: A Young Girl with Lower Abdominal Pain andDyspareunia ....................................................... 82

CASE 43: An Old Man Who got his Liver Tests doneOut of Curiosity .................................................. 84

CASE 44: A Young Boy with Progressive Jaundice andNegative KF Ring ............................................... 86

CASE 45: A Middle Age Man Whose Holiday Turned intoa Nightmare with Severe Abdominal Pain ....... 88

CASE 46: Recurrent Central Abdominal Pains with NoLocalising Sign ................................................... 90

CASE 47: A 12-year-old with Recurrent Fever andSplenomegaly ..................................................... 92

CASE 48: A Case of Recurrent Jaundice ......................... 94

CASE 49: Isolated Rise of SGPT ........................................ 95

CASE 50: A Middle Age Man with Early Satiety ................ 97

CASE 51: A Housewife with Acute Hepatitis-B andChanges of Portal Hypertension ...................... 98

ixCONTENTS

Page 10: Bedtime gastroenterology

CASE 52: A Successful Broker with Abdominal Pain .... 100

CASE 53: Haemolytic Anaemia and Gallstones ............. 103

CASE 54: A Case of Chest Pain Requiring SpecialManoeuvre to Swallow ..................................... 104

CASE 55: Recurrent Dimness of Vision and IntestinalPerforation ........................................................ 106

CASE 56: A Case of Hepatic Coma Following Delivery .108

CASE 57: A Case of Malaria with Jaundice andMultiorgan Failure ............................................ 110

CASE 58: A Devoted Lady Who Developed Diarrhoeaand Weight Loss after her Father’s Death ...... 112

CASE 59: An Alcoholic with Ascites ................................ 114

CASE 60: A Young Boy with Ascites ................................ 116

CASE 61: A 54-year-old Police Constable with Non-resolving Acute Pancreatitis .......................... 118

CASE 62: A Month Old Child with Jaundice .................... 120

CASE 63: A Case of Recurrent Pancreatitis .................. 122

CASE 64: A Case of Painful Dysphagia and WeightLoss ................................................................... 124

CASE 65: A Case of Sudden Onset of Abdominal Pain ..126

CASE 66: A Case of Right Sided Pleural Effusion withFever and Cough .............................................. 128

CASE 67: Young Girl with Recurrent Vomiting ................ 129

CASE 68: Acute Pancreatitis with Rapidly FillingPseudocyst ....................................................... 131

CASE 69: A Case of Recurrent Diarrhoea ...................... 134

CASE 70: A Case of Sudden Onset of Abdominal Pain ..136

x BEDTIME GASTROENTEROLOGY

Page 11: Bedtime gastroenterology

CASE 71: A Case of Alcoholic Hepatitis andConvulsions ...................................................... 138

CASE 72: A Case of Sudden Onset of Right SidedChest Pain during Endoscopic Treatment ...... 140

CASE 73: A 20-month-old Child Who Returned to theClinic after a Lapse of 12 Months ................... 141

CASE 74: A Case of Insidious Onset of Ascites ............. 144

CASE 75: A 6-month-old with Progressive Jaundice ..... 146

CASE 76: A Case of Jaundice and Convulsions ............ 148

CASE 77: A Case of Recurrent Abdominal Pain ............ 149

CASE 78: A Case of Severe Diffuse Abdominal Pain .... 151

CASE 79: Two Cases of Ascites of Uncertain Origin ..... 153

CASE 80: Two Cases of Recurrence of Jaundice .......... 155

CASE 81: A Case of Epigastric Pain and Vomiting ......... 157

CASE 82: A Case of Sudden Appearance of a Polypin Anal Area ...................................................... 158

CASE 83: A Case of Abnormal Liver Function TestsFollowing a Laparoscopic Cholecystectomy . 159

CASE 84: A Case of Recurrent Haemoperitoneum ....... 161

CASE 85: A Case of Persistent Jaundice and WeightLoss ................................................................... 163

CASE 86: A Young Woman with Fever and Jaundice ...... 165

CASE 87: A Case of Bleeding PR and Obstruction inSigmoid Colon .................................................. 167

CASE 88: Tales of Two Old Men with CholestaticJaundice ........................................................... 168

CASE 89: A Case of Recurrent Epigastric Pain ............. 172

xiCONTENTS

Page 12: Bedtime gastroenterology

CASE 90: A Case of Recurrent Vomiting ......................... 173

CASE 91: A 15-year-old Boy with Jaundice andBleeding PR ...................................................... 174

CASE 92: A Young Hotel Chief with Recurrent FleetingAbdominal Pain ................................................ 176

CASE 93: A Case of Persistent Dyspepsia .................... 178

Index .................................................................. 179

xii BEDTIME GASTROENTEROLOGY

Page 13: Bedtime gastroenterology

1CASE 1

CASE 1: A CASE OF RECURRENT JAUNDICEAn 18-year-old boy was referred for recurrent jaundice. Hewas well before 3 month when he first fell ill with fever andchills, headache and slight nausea. He was treated withparacetamol. In a couple of day’s time he noticed jaundice.His blood tests, reported below showed jaundice and hewas treated with supportive treatment. He continued to havefever and hence he was given injectable chloroquine andamoxycillin, which he took for 2 days and then stopped, ashe felt better. His bilirubin and SGPT normalised. About amonth later he again developed jaundice with minimal feverfor which he look Ciprofloxacin and remaining doses ofchloroquine left over from the first time. He felt better overfew days and returned to work. He felt unwell for the thirdtime about a month later and was found to have jaundice.There was no associated fever, nausea or vomiting.

On examination he appeared well although slightlyjaundiced. There was no anaemia, no nodes and no stigmataof chronic liver disease. Liver was not palpable and spleenwas just palpable. Kayser-Fleischer ring was absent by slit-lamp examination. Sonography revealed mild splenomegalybut no evidence to suggest chronic liver disease. Liver profileis shown below.

16/8 1/9 24/9 27/10

S. Bilirubin 4.5 mg 1.5 5.0 4.0(3.0 direct) (1.0 direct) (3.5 dir) (3.0 direct)

SGPT 104 iu 42 68 56

Alk Phos. 122 iu . 114 108(Up to 125)

Contd...

Page 14: Bedtime gastroenterology

2 BEDTIME GASTROENTEROLOGY

Contd...

Hb: 12.4 . 12.0 HBsAg:negative

TC: 7.0 . 6.4 HAV-lgM:negative

PS for MP Neg. . Neg. Albumin:4.0 mg

Q. What differential diagnosis you would consider?What further tests you would consider? What is yourfinal diagnosis?

A. This patient has recurrent “cholestasis”, but noobstruction on sonography and normal alk. phosphatse andhence obstructive jaundice is unlikely for all practicalpurpose. Hepatitis-B and C needs keeping in mind as bothcan give recurrent jaundice and Wilson’s, alcohol, etc.although LFT’s did not suggest chronic liver disease. Drugscan sometime give rise to jaundice but there was no suchhistory. Falciparum malaria could give rise to this pictureand this needs ruling out. We see two types of liverinvolvement of liver with falciparum malaria, one what wecall “hepatopathy” where there is predominantly a direct hyperbilirubinaemia and SGPT which is generally less than threetimes normal. The other, the” hepatitic” picture is almostindistinguishable from viral hepatitis. Falciparum liverinvolvement almost always start with fever and headachebut nausea, vomiting, etc. are not very prominent. Inendemic area it is now mandatory to include falciparummalaria in differential diagnosis of acute hepatitis. Somewhatsimilar picture is also seen in septicaemia, major trauma,

Page 15: Bedtime gastroenterology

3CASE 1

electrocution, etc. Tumour necrosis factor Alpha or otherchemokines may be responsible. Falciparum hepatopathyif treated in time, recovery is the rule. Fortunately for us hisfalciparum was picked up straight away and he respondedto appropriate therapy. Final diagnosis is Falciparumhepatopathy.

Page 16: Bedtime gastroenterology

4 BEDTIME GASTROENTEROLOGY

CASE 2: A 3-YR-OLD WITH HUGE LIVERAND ABNORMAL LIVER FUNCTIONS

A 3-year-old girl was referred for huge hepatomegaly andpersistently abnormal liver function tests.

She was first born, full term normal delivery. There wasno consanguineous marriage and there was no significantfamily history. Mother’s health during pregnancy wasexcellent. At birth a paediatrician had seen her and nothingunusual was found. At 6 months, her abdomen was notedto be distended. Liver was enlarged and non-tender withrounded edges. Spleen was not enlarged. Ultrasoundconfirmed the findings; portal and splenic veins were ofnormal size, lymph nodes were not enlarged. Cardiovascular,respiratory and neurological examination was normal. Sheappeared intellectually normal and all her milestones werenormal.S. Bilirubin: 1.8 mg, Alk. Phos: 351 iu, SGPT: 98 iuGGT: 84 iu PT 18/15 sec. Fasting blood sugar: 88 mg.Antibodies against TORCH, HAV, HBV, and HCV were allnegative.

Her development for next 2 years was normal with normalmilestones and intellectual development, normal hearing andnormal vision. No bony deformity was detected but repeatedliver function studies showed elevated bilirubin and liverenzymes. She was advised liver biopsy but parents wereconcerned and opted for further opinion.

On examination the child looked normal except hugehepatomegaly.

Liver function showed abnormality similar to her earlierreports. CBC, renal functions and bloods sugar were normal.Examination of eye including slit-lamp examination wasnormal. Sonography as above.

Page 17: Bedtime gastroenterology

5CASE 2

Q. What do you suspect? How will you proceedfurther? Would you do liver biopsy?

A. This was an interesting case. She was born to Keralianfamily. Her hepatomegaly was detected soon after birth andall known viruses were negative. Liver was huge and nodevelopmental defect was noted and hence all in all someform of storage disease of mild severity was most likely.Repeated normal sonography rules out any structural lesionof the liver. Most of the glycogen storage diseases haverecurrent hypoglycaemia and/or acidosis, muscle pain, poorphysical growth. This patient had none of these. Most of themucopolysaccharidoses (MPS) have associated spleno-megaly, mental retardation, corneal clouding, bony defor-mities, etc. in various combinations. This patient wasinvestigated in collaboration with a molecular genetist.Preliminary screening ruled out glycogen storage diseaseand attention was focussed on MPS. Her urine showeddermatan sulphate and subsequently leucocytes Arylsulphate was found to be low. In clinical terms this is mildform of type VI MPS, “Martaux-Lamy,” Where normaldevelopment is possible. Abnormal liver enzymes arebecause of infiltration of dermatan sulphate. As diagnosiscould be made this way liver biopsy was not indicated. Threeyear on the child is doing well.

Page 18: Bedtime gastroenterology

6 BEDTIME GASTROENTEROLOGY

CASE 3: A CASE OF CONSTIPATIONAND BLEEDING PR

A 58-year-old male was referred for bleeding pr. He was fit andwell but bothered by constipation for quite some time. About6 years prior to this he had similar problem of bleeding andhad flexible sigmoidoscopy where an ulcer was seen onanterior rectal wall. For six years he was taking “kayam churn”-a herbal laxative cocktail. He denied any fever, weight loss,anorexia. Blood was bright red and on the stool surface. Atsigmoidoscopy he had an indolent looking ulcer withhyperaemic edge on the anterior rectal wall some 7-8 cm, awayfrom anal verge. Mucosa up to splenic flexure was otherwisenormal.

Q. What “leading” questions you will ask? What isyour diagnosis?

A. This was a prototype of many cases that we see in thiscountry. He had a similar looking ulcer in the past thuspractically ruling out carcinoma. Carcinomatous or eventuberculous ulcers, both of which are very common in thisarea, have typical pathological look about themselveswhereas this ulcer has typical indolent, inactive, look withhyperaemic edges. This is a typical solitary rectal ulcerand once seen never forgotten. Even biopsy is not requiredin a classical case. When you see this, you invariably askthe patient about digital faecal evacuation. colloquially knownas “ Ganesh Kriya “ and most of the times it is the relativeswho say that there is nothing like that but the patient correctsthem and admits the fact. Biopsy has typical appearance. Itis believed to be caused by trauma during digital evacuation.They are notorious for not healing for years. Argon plasmalaser is said to have good results.

Page 19: Bedtime gastroenterology

7CASE 4

CASE 4: A CASE OF PANCREATITIS ANDFATAL HAEMATEMESIS

A 56-year-old male was admitted with severe supraumbilicalpain of sudden onset after heavy alcohol intake. Pain wasradiating to the back. He had severe nausea and vomitingwhich did not contain any blood. His investigation revealedvery high serum lipase and amylase. Sonography showedbulky pancreas and a dynamic CT scan with oral and IVcontrast confirmed pancreatitis but did not show muchnecrosed tissue on 3rd day of illness. Acute alcoholicpancreatitis was diagnosed. He improved conservatively andwas ready to be discharged on 10th day when he hadhaematemesis. He was stabilised and at endoscopy a largeblood clot on inferior and anterior wall of duodenal bulb wasseen. This was thought to be due to underlying ulcer and hewas put on high dose of omeprazole on which he appearedto settle down. A repeat sonogram showed a pseudocyst inthe pancreatic head region. On 4th day he had fatalhaematemesis and died before any resuscitative measurescould be instituted. An autopsy was carried out.

Q. What do you suspect to be autopsy finding? Howyou would have managed this case?

A. Sometimes in practice we make mistake that we regretfor rest of the life. This patient had pancreatitis and he wasfollowed by a “herald” bleed which was mistakenly thoughtto be due to a duodenal ulcer. At autopsy there was apseudocyst of pancreas with a large blood clot. There waspseudoaneurysm of gastroduodenal artery which was erodedcausing fatal bleed. In retrospect we should have taken more

Page 20: Bedtime gastroenterology

8 BEDTIME GASTROENTEROLOGY

care to find out what was behind that clot found atendoscopy. We should have also resorted to selective Celiacartery angiography. Either of these two could have alertedus to the impending disaster and prompted eitherembolisation or surgical intervention.

Page 21: Bedtime gastroenterology

9CASE 5

CASE 5: A CASE OF PAINFUL SWALLOWING ANDLEFT PLEURAL EFFUSION

A 50-year-old woman was referred with history of chest painand vomiting. She previously had heart burn which wastreated with ranitidine. Once she was diagnosed ashypertensive and put on atenolol but this was stopped. Thatmorning she had nauseous feeling and few large vomits ontaking food. After second vomit she had severe retrosternalchest pain and painful swallowing. She rushed to local nursinghome where she was found to have hypotension andnonspecific ST, T charges. She was suspected to haveunstable angina and treated accordingly. She remained quiteuncomfortable with tachypnoea and tachycardia. Onexamination she had no specific findings. Portable chestX-ray showed doubtful left sided pleural effusion. Permissionfor endoscopy or barium was not given by the attendingcardiologist in view for suspected unstable angina. Herdistress continued to worsen and she was shifted to CCUof the local hospital where she was treated for unstableangina and put on intensive treatment and an angiographywas planned. First set of cardiac enzymes were normal.Repeat ECG was unchanged. Next morning she remainedunwell and was found to have puffy face and crepitationswere felt in her neck. Urgent portable chest X-ray showedmoderate left pleural effusion and surgical emphysaema.

Q. What do you suspect to be the cause ofinvestigation?

A. This lady had what is known as “Boerhaave’s syndrome”which is transmural rupture of oesophagus following violent

Page 22: Bedtime gastroenterology

10 BEDTIME GASTROENTEROLOGY

retching. This is an extension of Mellory-Weiss syndrome.History if well taken is always very suggestive. Unfortunatelydoctors don’t take pains to get good history and this type oftragedy results. Worst still, after suspecting the diagnosisshe had barium swallow which resulted in fatal mediastinitis.Endoscopy is preferred. Left sided effusion is because theterminal part of oesophagus sweeps to the left to join thestomach and being mobile vulnerable to this type of injury.Sudden onset of chest pain and left sided pleural effusionfollowing retching or vomiting should mean only “Boerhaave’ssyndrome”. Most cases require surgery and only a few withprobably small tear could be managed expectantly.

Page 23: Bedtime gastroenterology

11CASE 6

CASE 6: A CASE OF HEMIPLEGIA FOLLOWED BYPROGRESSIVE JAUNDICE

A 63-year-old male was referred for progressive jaundice.He was well before 3 months when he had an attack ofhemiplegia due to cerebral thrombosis. This was treatedconservatively along the usual lines. He was referred tophysiotherapist for mobilisation. During the follow-up he wasalso found to have hypertension. This was treated withEnalapril. He was noticed to have jaundice by thephysiotherapist during his follow-up and he was referredback. His results were.Bilirubin 4 mg (3 direct), SGPT 56 iu; Alk. Phos 232 iu(up to 125 normal) sonography did not show anyobstructive changes.

Over a period of one month his bilirubin progressivelyincreased to 14 (10 direct), SGPT 44 iu, alk. Phos. 347 iurepeat sonography did not show any obstructive picture.On examination he looked well and there were scratch marksdue to itching. Liver was just palpable on deep inspiration,spleen was not palpable. He otherwise looked well. He wastaking Atenolol, Enalapril, Liv-52, B-Complex and someherbal remedies which were started after he was found tohave jaundice. His urine was dark yellow and his stoolswere slightly pale. LFTs were almost identical, ANF/ANAwere negative.

Q. What is your investigation? Likely diagnosis?

A. This patient had Enalapril induced jaundice. Althoughhe had ERCP to rule out sclerosing cholangitis perhaps, atleast with hindsight this was rushed into without giving

Page 24: Bedtime gastroenterology

12 BEDTIME GASTROENTEROLOGY

enough time to drug withdrawal. He responded dramaticallyto Enalapril withdrawal. Drugs are an important cause ofliver function abnormality and any patient who is on anydrug must be suspected to have liver function abnormalitydue to drugs unless otherwise proved.

Page 25: Bedtime gastroenterology

13CASE 7

CASE 7: A CASE OF DIARRHOEA ANDFAILURE TO THRIVE

A 16-year-old boy was brought with complaints of diarrhoeaand failure to thrive. He had this problem almost since hewas weaned off breast milk. His motions were profuse, foulsmelling and contained oil drops. There were very loudborborygmi. He was not growing as well. There was no familyhistory of note. In the past he had several courses ofantibiotics and anti-tuberculous drugs without any benefit.

On examination he appeared alert and oriented. Weight18 kg. He was severely stunted and appeared like a 7-year-old child. There were evidences of vitamin and proteindeficiency. His investigations revealed dimorphic anaemia,hypoproteinaemia and hypocalcaemia.

Q. Likely diagnosis? Investigations?

A. This child, almost from word goes, had Celiac disease.He had all the features of severe malnutrition. A jejunalbiopsy was obtained which showed total villous atrophy. Hewas put on Gluten-free diet and responded extremely well.We had facility to do anti-tissue transglutamase antibody(tTGA), anti-gliadin antibodies and anti-endomysialantibodies that would have helped. A repeat biopsy after sixmonths of gluten withdrawal showed near normal villi. Celiacdisease is uncommon in India but this may not be true.Increasingly patients are identified who have iron deficiencyanaemia as a presenting feature of celiac disease. AlphaGliadin is supposed to be the offending agent. It is morefrequently seen in people positive for DQ2 and DQ8 class IIHLA haplo types. Wheat protein breaks down into Alpha

Page 26: Bedtime gastroenterology

14 BEDTIME GASTROENTEROLOGY

gliadin and other glutamine rich proteins. There is amidationby tissue Transglutaminase (tTG) of these products resultingin products with certain properties which allows binding toHLA class II molecule on enterocytes. These in turns resultinto activation of local CD4 T lymphocytes and immunereaction with resultant tissue damage. It is envisaged thatanalogous peptide may be synthesised which will bind toclass II HLA molecules on enterocytes but will not stimulatethem to activate resident CD4 T lymphocytes. Recently anti-trans-glutamase antibodies (TGA) is used as a diagnostictest of choice and is claimed to have sensitivity and specitynearing 95% and 98% respectively and if this is borne outby the test of time then diagnising celiac disease will becomean integral part of work up of any chronic diarrhoea.

Page 27: Bedtime gastroenterology

15CASE 8

CASE 8: A CASE OF PERSISTENTPAIN AFTER CHOLECYSTECTOMY

A 45-year-old man was seen because of severe diffuseabdominal pain. He had this pain off and on for nearly threeyears. Sometimes the pain would get worse with food. Therewas progressive weight loss over this time. He had upperGI endoscopy a year ago when gastritis was found and thiswas treated along the usual lines with Famotidine, etc. Ashis pain persisted he had a sonography done which showedgallstones for which he had laparoscopic cholecystectomywhich was uneventful. A general survey at that time did notreveal any gross anatomical abnormality. Soon after he keptcomplaining of pain. He had various pain killers and sedativesfor this. His pain got progressively worse and to an extentthat it was unbearable. He had no fever and no tachycardia.His abdomen was soft, no localised tenderness wasdetected, and there was no distension. Pain was diffuse.There was no associated nausea or vomiting. Peristalseswere present and rather vigorous. His plain X-ray abdomenwas unremarkable. He was referred to the psychiatrist whodiagnosed depression and was treated accordingly!!!. Onthird day postoperative day he developed hypotension,tachycardia and tachypnoea and abdominal distension. Hisplain X-ray abdomen showed multiple air and fluid levels.His white cell count was elevated.

Q. What is your thinking about this case? Yourtentative diagnosis?

A. This patient had mesenteric artery thrombosis producingextensive gangrene. Retrospectively his symptoms were

Page 28: Bedtime gastroenterology

16 BEDTIME GASTROENTEROLOGY

suggestive of abdominal angina. Doppler study is a goodscreening test if the index of suspicion is high. Angiographycould be useful but not easily available and not suitable forscreening. Pain he had is sometimes very suggestive.Ischaemia generally produces diffuse and severe abdominalpain without any localising signs. Age old saying thatabdomen is a Pandora’s Box is still true.

Page 29: Bedtime gastroenterology

17CASE 9

CASE 9: A CASE OF SEVERE ITCHINGAND JAUNDICE FOLLOWING PREGNANCY

A 29-year-old lady was admitted with jaundice and severeitching. She was well till about a year and a half when shebecame pregnant with her third child. During the pregnancyshe developed itching which progressively got worse andshe consulted a skin specialist. Few dietary restrictions weresuggested together with anti-histaminics with good effectinitially but soon her itching returned and further treatmentdid not help. She aborted at around 23 weeks. Followingabortion there was some relief in her itching but it soonreappeared and now she had obvious jaundice.

She had no previous illness. Her mother had hypothyroi-dism. Apart from itching and fatigue she was otherwise well.Her appetite was preserved. There was no fever, nausea orvomiting. On examination she had mild icterus and scratchmarks. Liver was just palpable and spleen was palpable.Systemic examination was normal. Her liver function testswere:

Bilirubin: 2.5 mg, SGPT: 46 iu, Alk. Phos. : 459 iuProthombine time 16/13 sec.USG: Liver mildly enlarged, No IHBR dilatation seen, CBDappeared normal; there were two small gallstones. Spleenwas slightly enlarged and there was slight dilatation of portalvein. Liver biopsy showed bile duct proliferation and lymphoidinfiltration.

Q. What further tests you would carry out? What isyour diagnosis?

A. Although gallstones were there, the duration of illnessand normal looking biliary tree, we thought they were

Page 30: Bedtime gastroenterology

18 BEDTIME GASTROENTEROLOGY

incidental only. She was likely to have primary biliarycirrhosis. We carried out anti-mitochondrial antibodies whichwere strongly positive. Anti-nuclear anti-bodies were negative.Upper GI endoscopy revealed early oesophageal varices.Thyroid functions were checked and they were normal. Therewas also no evidence of diabetes or obvious pulmonaryinvolvement as these things sometimes go together. PBCis an autoimmune condition of uncertain aetiology.Predominantly affects women generally in 4th to 6th decadebut younger age group could also be affected as in thiscase. Ursodeoxycholic acid could be useful for itching. Apartfrom this there is no other specific treatment.

Page 31: Bedtime gastroenterology

19CASE 10

CASE 10: A YOUNG GIRL WITH ABDOMINALPAIN AND LYMPHADENOPATHY

A 5-year-old girl was admitted with recurrent abdominal painand lymphadenopathy. She had problem with recurrent peri-umbilical abdominal pain for about a year. This was transientand would come at any time and subside in a very shorttime. No associated fever, nausea or vomiting. No urinarycomplaints. Her appetite was well and her weight was steadyand within normal range for her age. She had one older brotherand all her family members were normal.

On examination she appeared cheerful in no particulardistress. She had generalised lymphadenopathy, which weresmall and soft. Nothing else to find on general examination.CBC, LFT, RFT, electrolytes were normal. Chest X-ray wasnormal. USG showed several small to large lymph nodes inabdomen and in the neck. No ascites or thickening of bowelloop was seen. Sonography was otherwise normal.

Q. What would you do now? What is your tentativediagnosis?

A. With more powerful sonography machines and skilledsonographers we are picking up more and more cases ofthese types of lymphadenopathy cases which defy any setpattern. Obviously taken in isolation this lymphadenopathycould be anything from tuberculosis to lymphoma but thechild had symptoms which were not consistent and shewas otherwise remaining well. In this particular case we didan excision biopsy of a neck gland which appeared soft andsupple and showed non-specific reactive changes with whichthe operating surgeon agreed. PCR for tuberculosis was

Page 32: Bedtime gastroenterology

20 BEDTIME GASTROENTEROLOGY

negative in blood and in the specimen. Now she is underfollow-up and does not show any deterioration.

After this case we have seen several similar cases wherein absence of any constitutional symptoms the patientis followed up regularly and these patients don’t seem todeteriorate. So these are most probably cases of non-specificmesenteric lymphadenopathy which, it appears, may notbe restricted to the abdomen only but may spread to otherparts of the body. Specific challenge is not identified butdoesn’t appear to be tuberculosis and patients seem to dowell without any specific treatment.

Page 33: Bedtime gastroenterology

21CASE 11

CASE 11: A CASE OF ABDOMINALPAIN AND WEIGHT LOSS

A 55-year-old male was seen for continuous supraumbilicalpain for about a year. He was previously fit and well and hadnever seen a doctor before!! It started with dull ache in thesupraumbilical area which had no significant relationshipwith eating, bowel movement or urination. This was treatedwith H-2 blockers and some anti-spasmodic. The painpersisted and hence a sonogram was carried out whichshowed small right kidney. His CBC was showing Hb of14 with normal indices and normal white cell count.Urea and creatinine were normal. Urine showed a fewpus cells. IVP showed non-functioning right kidney.

He was treated with antibiotics but the pain persistedand he started loosing weight slowly. He was advisedneprectomy which he agreed to but this failed to relieve hispain. A general surgical consultation was obtained and acolonoscopy and a barium meal were carried out andwere normal. The pain still persisted and now after aboutten months he had lost 9 kg in weight which was about15% of his body weight. He had lost his appetite andhis Hb was now 9.2 with mild microcytosis andhypochromia.

He now developed abdominal fullness and hiccup.

Q. What would you do now?

A. That this patient had an organic illness was now amplyclear from all the information we had. We went through hishistory first. At all the time he had supraumbilical pain andnow he was developing microcytic hypochromic anaemia

Page 34: Bedtime gastroenterology

22 BEDTIME GASTROENTEROLOGY

with bloated feeling, loss of appetite, loss of weight andhiccup. We have seen many times that when the stomachis full of content patients develop hiccup. Our first step wasto carry out an upper GI endoscopy. Normal barium a fewmonths ago did not deter us as barium could be missing thefiner details, sometimes in as many as 80% of cases.Gastroscopy showed stomach full of debris even after aprolonged fasting in a patient who was hardly taking anysolid diet for past few days. Though there was no obstructiontill the third part of the duodenum, this substantiated ourbelief that there was an obstructing lesion not far from thereach of the endoscope and hence we got barium done whichshowed obstruction in the 4th part of duodenum. This wasfound to have adenocarcinoma of small bowel.

The important lesson from this case history is that renalpain is never epigastric in nature and progressive anaemiaand weight loss are HARD evidence which one ignores athis own peril. Barium is not full proof and quality of theimaging system, the second time it was done on digitalX-ray machine, and the time a radiologist spends on suchcases can play a significant part in the ultimate analysis.

Page 35: Bedtime gastroenterology

23CASE 12

CASE 12: MIDDLE AGED WOMANWITH RECURRENT VOMITING

A 45-year-old lady was seen for recurrent vomiting, heartburn and loss of appetite. She was not feeling well withprogressive fatigue for about three months. A routine physicalrevealed mild microcytic hypochromic anaemia andsonography showed a “pseudocyst” in the pancreatichead region. There was no preceding history of anyabdominal pain and her amylase and Lipase were normal.

On examination she was pale and appeared to have lostweight recently. Abdomen was soft other systems werenormal.

An upper GI endoscopy showed a swelling in the regionof pylorus which was compressing the pylorus. Themucosa of the swelling was smooth and biopsy was negative.

Q. How will you proceed now?

A. This patient had a cystic lesion in the pancreatic headregion without any history of any pancreatitis so wesuspected that this cystic lesion was compressing the antralregion. Biopsy was negative for malignancy. A CT scanshowed a pancreatic mass. Next we did needle aspirationbiopsy of the mass which showed inflammatory changesonly. Our experience is that pancreatic mass is notoriousfor this type of confusion. It is always better in such casesto explore than to wait indefinitely. This will help to cometo firm diagnosis as well offer on the spot treatment. Thispatient was proven to have carcinoma of head with regionalmetastasis.

Page 36: Bedtime gastroenterology

24 BEDTIME GASTROENTEROLOGY

CASE 13: AN ALCOHOLIC WITH MASS IN THE LIVERA 42-year-old male was seen for nausea, vomiting, and painin right lower chest of about 15 days duration. He alsocomplained of low grade fever. He drank 2 pegs of whiskydaily for last 3 years. He had jaundice of uncertain aetiologysome 3 years ago. No other significant history.

On examination he was deeply jaundiced but had nostigmata of chronic liver disease. Liver was enlarged andtender. Spleen was not palpable.

His LFT were, bilirubin: 10 mg. (80% conjugated), SGPT:600 iu, Alk. Phos: 232 iu, PT: 18/14 sec., albumin: 3.5Globulin: 3.7, Alpha feto protein: 5 iu, Hb: 10.2, WCC17,000 with 70% polymorphs.

Sonography showed a large mass in the right lobe,left lobe showed heterogenous echo texture, spleen wasenlarged, no splenic varices seen, portal vein was dilated.

Q. How will you investigate this case further?

A. This patient appeared to have chronic liver disease withmass in the liver which was very large and in all probabilityit was hepatoma eventhough alpha fetoprotein was normal.In alcoholic liver disease we have not seen manyhepatocellular carcinoma and world literature also favourshepatitis B and C as the predominant cause of hepatoma.Hence, he was screened for hep B and C and was found tobe positive for hepatitis B. As he had normal alpha fetoproteinwe had to carry out liver biopsy which in present state ofknowledge we would have avoided to prevent seeding alongthe tract in a case of liver mass with raised alpha fetoproteinand positive reaction to either Hep B or C. He turned out to

Page 37: Bedtime gastroenterology

25CASE 13

have hepatoma on biopsy. In an isolated mass lesion it ismore useful to biopsy the normal looking liver to decide if itis normal enough to take over after resection of the mass.This patient had established cirrhosis in the liver and hencenot suitable for surgery.

Page 38: Bedtime gastroenterology

26 BEDTIME GASTROENTEROLOGY

CASE 14: A 15-YEAR-OLD BOY WITHPROLONGED JAUNDICE

A 17-year-boy came for consultation with jaundice. He waspreviously fit and well and did not abuse drugs, tobacco,etc. He had no injection or blood transfusion. Some 6 weeksago he had travelled quite extensively eating out in roadsidehotels. Somet 4 weeks ago he developed intestinalsymptoms, anorexia, nausea, dark urine and jaundice. Hewas examined by another gastroenterologist who found hisliver slightly enlarged and tender. His bilirubin was 3 mg,ALT: 1004 iu, GGT: 575 iu, Alk phos: 182 iu Infectivehepatitis was diagnosed. He was prescribed vitamins, bedrest and his symptoms gradually subsided. Repeat LFTs in2 weeks time showed S. bil: 1.5 mg, ALT: 300 iu, GGT: 343iu and Alk. Phos: 142 iu Six weeks later repeat LFTsshowed S. bil: 2 mg, ALT: 940 iu, 447: 565 iu, Alk. Phos:152 iu. He was feeling very well and had already put on allthe weight he had lost. On examination he had jaundice;there were no stigmata of chronic liver disease. Liver wasnow enlarged. Rest of the examination was normal.

Q. How will you proceed now?

A. In a previously fit and healthy individual without anypredisposing factors for Hep-B, this is a classic story forfaecal-oral type of infective hepatitis namely type-A and Eboth of which are sporadic and epidemic. Type-C hepatitiswas a remote possibility. LFTs in this patient shows classicalrelapse which is mainly biochemical. It is more commonwith Hep-A, about 8-15% patients relapsing. This relapsemay be biochemical only, i.e. only the blood tests may be

Page 39: Bedtime gastroenterology

27CASE 14

abnormal without any new symptoms or it could besymptomatic also. Viral serology, HAV-IgM, is helpful and ifpositive as was the case in this patient, clinches thediagnosis.

Page 40: Bedtime gastroenterology

28 BEDTIME GASTROENTEROLOGY

CASE 15: A CASE OF RECURRENTCHEST PAIN AND DYSPHAGIA

A 45-year-old lady was seen for recurrent chest pain anddysphagia. She said she suffered from acidity for quitesometime but lately over a year or so she had recurrentattacks of sharp, choking type of chest pain and at thattime she could not eat anything. Any attempt to even drinka small amount of liquid would aggravate her pain and inducevomiting. She had seen a cardiologist and had routine car-diac work-up which was negative for ischaemic heartdisease.

On examination she appeared well and pointed her chestpain rather vaguely all over the chest. Physical examinationwas normal.

Q. What do you suspect? Investigations?

A. There was choking and “sharp” pain made worse by eatingaroused suspicion of a volvulus. An endoscopy was carriedout which was extremely difficult. It confirmed a large hiatushernia. Fundus had flipped over on the right making rest ofthe endoscopy examination extremely difficult. Volvulus canreliably be diagnosed by endoscopy, although experienceis needed. There are two types of volvulus of stomach, oneis organo-axial which takes place along GO junction andduodenum and the other one is organo-mesenteric whichtakes place along GO junction and greater curve where themesentery is attached. In both types at endoscopy GOjunction is patulous and on advancing the endoscope seemsto enter in the blind pouch. Negotiating the endoscope throughthe site where volvulus has taken place may be difficult.

Page 41: Bedtime gastroenterology

29CASE 15

Pylorus may be difficult to see and in almost all cases almostimpossible to enter without great skill and difficulty. Bariumexamination is conclusive in such cases and may takeprecedence over endoscopy in such selected cases.

Apart from mechanical symptoms, there is a possibilityof strangulation. Surgery is a curative in such circumstances.This patient underwent corrective operation with very goodsymptomatic improvement.

Page 42: Bedtime gastroenterology

30 BEDTIME GASTROENTEROLOGY

CASE 16: OPERATION ROOM ASSISTANTWITH RIGHT SIDED LOWER CHEST PAIN

An 18-year-old operation room assistant comes forconsultation with ache in right lower chest. He said he waspreviously fit and well. For last 15 days or so he had startedfeeling a “stitch” in right side of his chest which had graduallygot worse and now it was giving him significant “sharp” painon movement and he was unable to ride his bicycle.Breathing and coughing made his pain worse. There was nopast history of cough, cold; fever or haemoptysis, therewas no previous injury, fall, etc. His bowels opened regularly;there was no diarrhoea, abdominal discomfort or pain. Onexamination, he was comfortable, there was no jaundice.He was moderately tender over his right lower chest bothanteriorly and posteriorly. Air entry was normal on both sides.No pleural rub was heard. Chest X-ray revealed slightly raisedright dome which was poorly mobile on respiration.

Q. What is your suspicion? What investigations willyou carry out now?

A. This patient looked remarkably well for his discomfort.Amoebic abscess was suspected because of his history.An ultrasonic scan was done same day which showed acricket ball sized abscess with ragged wall, in right lobeantero-superiorly. Stool examination (X3) did not reveal anycyst or protozoa. His Hb was 12.3 and WCC was 13,500.His temperature was 100 F. Diagnosis of amoebic abscesswas made and he was treated with metronidazole anddiloxanide fuorate with prompt improvement symptomaticallyand sonologically. Had he failed to respond to treatment or

Page 43: Bedtime gastroenterology

31CASE 16

become more toxic we would have aspirated his abscessto make sure that this was amoebic indeed. Slightly raisedWCC is fairly common in amoebic abscess. Jaundice isvery uncommon and liver function abnormality if present isgenerally trivial. Generally medical treatment suffices. Apartfrom non-responders, drainage is required where the abscessis near the surface and about to rupture or that the diagnosisis very uncertain. Many a times sonographic monitoring isrequired. In tropical countries, pain in liver area should arousesuspicion of amoebic liver abscess or hepatic amoebiasis.History of previous intestinal amoebiasis is not alwayspresent and similarly stool examination may be negative ina large number of patients (up to 55%) indeed the exposuremay have been several years prior to the development ofabscess. Serology is useful in non endemic area andamoebic abscess is unlikely if it is negative. The reverse isnot true. In an endemic area it has little value.

Page 44: Bedtime gastroenterology

32 BEDTIME GASTROENTEROLOGY

CASE 17: A MIDDLE AGED MAN WITH PAST HISTORY OF VAGOTOMY AND DIARRHOEA

A 56-year-old male was referred for severe diarrhoea. Hesaid he had diarrhoea for last 6 to 7 years. The stool waslarge in amount, without any blood or mucus. There wereassociated borborygmi but no abdominal pain. There wasno urgency, tenesmus or incontinence. If he fasted diarrhoeawould ease considerably but would not go away. Over lastfew years he had lost nearly 10 kg weight. The stool wasnormal in colour and sometimes contained undigested foodparticles especially peas and carrots, etc. It was foul smellingbut was not greasy and not difficult to flush. He had noarthritis, skin rashes, red eyes or low backache. Systemicinquiry was otherwise normal. He had peptic ulcer nearly 20years ago and had vagotomy plus pyloroplasty. He did nottake any medicine routinely but had tried various anti-diarrhoeals like lopamide, codeine phosphate given by hisGP and other physicians. He denied any laxative abuse.There was no family history of similar illness or other majordisease.

On examination he looked slightly pale and weighed at52 kg. Abdomen was soft, no organs or masses were pal-pable. Rectal examination was normal, Rigid sigmoidoscopywas normal. Rest of the examination was normal.

Hb was 11.5 gm% with microcytic hypochromic picture,faecal occult blood X 3 were negative. Albumin was 3.4,calcium was 10.2 and Alk phos 137 iu /L. Stool microscopyand culture were negative.

Page 45: Bedtime gastroenterology

33CASE 17

Q. How will you analyse this case. How will you decideon further investigations?

A. This patient had diarrhoea of long duration. There arethree main possibilities (1) Diarrhoea could be result of hisvagotomy and pyloroplasty. (2) There may be another medicalcause of his diarrhoea and previous surgery could be acoincident. (3) fictitious diarrhoea. Considerable judgmentis required here. Mild iron deficiency anaemia, with negativeFOB Negative (X 3) is compatible with previous gastricsurgery. Weight loss is gradual and not dramatic and durationof diarrhoea is very long ruling out carcinoma, tuberculosis,and thyroid disease, absence of folate deficiency picturewould rule out tropical sprue or unmasking of Celiac disease.Normal electrolytes, calcium, etc. makes VIPoma andGastrinoma unlikely. Relationship with milk should give goodidea about lactose intolerance and lactose tolerance test isin order. This logical approach will narrow down possibilitiesand in my experience shorten the list of investigations whichwould be otherwise formidable. A good barium follow throughis required. My inclination is to take biopsies from sigmoidcolon to look for Crohn’s disease, microscopic colitis, etc.stool should be examined for laxative abuse likephenolphthalein. Lactose tolerance test, fasting blood sugarto exclude diabetes and thyroid function in case it isundiagnosed thyrotoxicosis. I would stop investigating hereand treat his diarrhoea as post vagotomy and pyloroplastydiarrhoea. Further investigations like colonoscopy, intestinalbiopsy and hormone assays should be carried out when thepatient is under observation and there is a very strongindication to do them.

Page 46: Bedtime gastroenterology

34 BEDTIME GASTROENTEROLOGY

CASE 18: AN 18-YEAR-OLD WITHFEVER AND DIARRHOEA

An 18-year-old boy was seen for fever. He was fit and welltill about a month ago, when he had a bout of fever anddiarrhoea lasting for about 2 days. He took paracetamol andLoperamide and felt better but not quite alright. A week laterhe again had fever for which he had paracetamol andantimalarial, chloroquine, which settled the fever for nexttwo days when he had high grade fever. He now complainedof constipation. He felt anorexic. He self-medicated himselfwith paracetamol for three days till a family friend broughthim for consultation.

He was a student at a local polytechnic. He deniedtobacco, alcohol or drug abuse. On examination he lookedill, his temperature was 103 F. His pulse was 102/min. Nojaundice or glands were detected. Abdomen was soft andsplenic tip was barely palpable. Rest of the abdomen wasnormal. Lungs, heart and nervous systems were normal.

Q. What investigations would you carry out? What isthe likely diagnosis?

A. Two patient has fever, intermittently for about 4 weeksfor which no great effort has been made to find out theaetiology. Any patient with fever without any obvious causeshould have at least following investigations viz. CBC withdifferential, ESR, Smear for malarial parasite, and bloodculture. Urine should be examined for microscopy. The patienthad a bout of diarrhoea, followed by constipation few dayslater. He also has relative bradycardia. And this takentogether with high grade fever should alert one to the

Page 47: Bedtime gastroenterology

35CASE 18

possibility of typhoid in a tropical country. S. Widal shouldbe done in this case. In fact, any patient, in a proper set up,complaining of abdominal pain and fever should have typhoidmeticulously looked for. Sometimes malaria and typhoidcan’t be separated and treatment for both will have to bestarted till a firm diagnosis is established.

Widal test done twice at 7-10 days interval confirmedrising titre suggesting typhoid.

Page 48: Bedtime gastroenterology

36 BEDTIME GASTROENTEROLOGY

CASE 19: AN OLD MAN WITHHAEMATEMESIS AND MELENA

An 84-year-old male was referred for haematemesis andmelena. He was a known case of rheumatoid arthritis andwas taking piroxicam for last several years. He did not drinkalcohol or smoked. He had an episode of “melena” nearly ayear ago for which he received ranitidine. He remained welltill a fortnight before consultation when he was reported tohave passed dark stool and had retching which brought outdark coloured vomit. He was seen by his general practitionerwho prescribed ranitidine. He had further 3 to 4 episodes ofpassing dark coloured stool, each lasting 2 to 3 days, overnext fortnight. Day before consultation his Hb fell to 4.9 andhence the referral.

On examination he had gross rheumatoid arthritis of hishands and feet. He looked little bit confused. There was amarked pallor. His abdomen was soft; there was notenderness or organomegaly. Peristalses were present.Rectal examination showed normal coloured stool. Therewas resting tachycardia and postural drop of blood pressure.Other systems were normal.

Relatives were interviewed who supported patient’sstatement about absence of any pain at anytime. They wereworried about his mental state and physical condition.

His investigations, showed Hb 4.9 gm, normochromic,normocytic picture. Reticulocyte count was 4%,electrolytes, urea, creatinine and liver function tests wereall normal. Clotting profile, including platelet count wasnormal.

Page 49: Bedtime gastroenterology

37CASE 19

Q. How will you manage this case? List investigationsin order of preference.

A. This patient looked ill and confused. Very low Hb, restingtachycardia and postural drop of blood pressure suggestssignificant blood loss. He needs blood transfusion straightaway to bring his Hb up to an arbitrary lever of 10 gm. This“allows” him to bleed till some thing is done about bleeding!!According to his relatives he was quite “with it” before fewdays. Low Hb can produce this type of confusion especiallyin the elderly.

He was transfused 7 units of PCV with CVP measure-ment over next 36 hrs. An upper GI endoscopy wasperformed which showed gastritis and an occasional erosion.There was no active bleeding and no altered blood.

Relatives were instructed to preserve any stool passedfor visual examination. His ranitidine was continued. Twodays later he had further episode of “melena” It was actuallymaroon/ dark brown coloured blood, mainly blood clots andnot actual melena. A Nasogastric tube aspirate did not showany fresh or altered blood. Over the years I have foundthese two parameters, namely the colour of blood andNasogastric aspiration is very helpful in determining theapproximate lever of lesion. Here we thought the lesion wasbetween ligament of Treitz and proximal colon.

A colonoscopy was performed. We were looking forangiodysplasia and diverticular disease of right side of colon.Good views were obtained and up to caecum the colon wasnormal. A small bowel enema was done next which showedtwo jejunal diverticuli. In previous days this would have beenenough for a patient to have resection of his diverticuli, but

Page 50: Bedtime gastroenterology

38 BEDTIME GASTROENTEROLOGY

experience has shown that many a times diverticuli couldbe a “red herring”. Angiography or technetium scan can helpin such dilemma, the former more conclusively. The patientunderwent technetium scan for technical reasons whichsuggested that the bleeding could be from high jejunum,near the area where he had diverticuli. A resection was carriedout. At operation we did an endoscopy which did not revealany other lesion, especially any ulcer due to NSAID(piroxicam) drug. To have NSAID induced ulcer in small bowelis not uncommon and must be kept in mind. Histologyshowed one of the diverticuli to have aneurysmal dilatationof a medium sized artery. The patient had no further bleed.With increasing availability of skilled interventional radiologistsuch cases could be managed by mesenteric angiographyand embolisation which would avoid surgery and thereforeis the choice of treatment.

Page 51: Bedtime gastroenterology

39CASE 20

CASE 20: A 30-YEAR-OLD LADYWITH HAEMATEMESIS

A 30-year-old lady was referred with haematemesis. Shehad been keeping unwell for last 6-7 year following a severeand prolonged jaundice. Since then she felt unusually tired,anorexic and had malaise and aches and pains. Sheintermittently noticed jaundice for which she was treatedwith popular herbal remedies. About 8-10 months ago shenoticed swelling of her abdomen and feet. This was treatedwith diuretics. About six month ago she had moderatehaematemesis for which she was admitted under surgicalservices, where she was found to have oesophageal variceswhich were injected with sodium tetradecyl sulphate. Herspleen was moderately enlarged. Her HB was 8.4 gm, WCC:7.9, Prothombine time: 50/14 sec, bilirubin: 3 mg, SGPT:180 iu, GGT: 96 iu alk. Phos: 154 iu urea, creatinine andelectrolytes were normal. She had no further bleed andwas discharged with follow-up instructions.

Q 1. What instructions do you think the surgeonwould have given her?

Q 2. How will arrange further investigation?Q 3. What is the long-term prognosis?

A 1. This is a case of portal hypertension which from herhistory followed a prolonged jaundice. Surgeon advised herto attend gastroenterology clinic to find out aetiology of herportal hypertension, which they thought was due to cirrhosisof liver. They had explained to her that she will need repeatedinjections in her vein or banding of her veins to obliterateveins in her gullet and that this is generally very successful.

Page 52: Bedtime gastroenterology

40 BEDTIME GASTROENTEROLOGY

They had also mentioned about shunt procedure should thisfail unfortunately. She took a long time to come to our clinic.Ideally we prefer to deal with endoscopy as soon as thepatient is stable so that there is no chance of further bleed.

A 2. She needs ultrasound of her liver, spleen and portalvein. Hepatitis serology particularly B, C need doing in viewof her previous history. This showed that she had hepatitisB infection with ‘e’ antigen reactivity. HBV-DNA was morethan 100 pico gm. Liver biopsy showed chronic activehepatitis with fibrosis. If serology were to be negative sheshould have iron and copper studies, alpha-1 antitripsynassay, etc. to find out the cause of her cirrhosis. History ofalcoholism must also be inquired into. Autoimmune hepatitisalso behaves like this and ANA/ ANF may be required.

A 3. Untreated this patient has poor prognosis. Recentintroduction of antiviral agents like Interferon plus Lamivudineor Adefovir and Entecavir or Interferone Ribovirine ispromising in hepatitis B and C cases respectively wherecirrhosis has not been established yet. Once cirrhosis isestablished no drug available yet can reverse it, however,we have reached a stage where liver transplant is anaccepted mode of treatment. Shunt operation have theirproblems of hepatic encephalopathy, shunt failure andoperative mortality and morbidity and are not generallypopular except in cases of non-cirrhotic portal fibrosis wherebecause the liver function is preserved, there is a very lowincidence of encephalopathy and I have seen patients whowere doing very well after many years of shunt operation.This patient was found to have varices which we obliteratedwith banding and then she was put on latest vialablepegylated interferone.

Page 53: Bedtime gastroenterology

41CASE 21

CASE 21: A CASE OF SUDDENONSET OF BLOODY DIARRHOEA

A 35-year-old lady was referred because of bloody diarrhoea.She was well before about a month ago, when she had violentargument with her husband over another woman. Nextmorning she had bloody diarrhoea. She did not consult thedoctor for next 3-4 days till her stools were showing onlyblood most of the time. There was associated urgency andoccasional incontinence. She had this feeling of incompleteevacuation. She consulted her GP

Q 1. If you were her GP how you will investigate her.Q 2. What is the likely diagnosis? How does it

behave?Q 3. What is the prognosis?

A 1. In a number of cases like this, after severe psycho-logical trauma, patients have bloody diarrhoea as firstmanifestation of colitis. Experts tell us that there is no causeand effect relationship between the two but always keepulcerative colitis in mind.

GP should be checking her stools for microscopy andculture preferably three times or more. He should also doCBC, ESR and platelets, as platelets are known to be highin an attack of colitis.

A proctosigmoidoscopy in un-prepared colon should benext investigation. Generally I would wait for stool exami-nation before sigmoidoscopy because infective colitis cangive almost identical appearance. This is important becauseif it is infective colitis or amoebic colitis, a proper treatmentwill cure the disease for once and all where as idiopathic

Page 54: Bedtime gastroenterology

42 BEDTIME GASTROENTEROLOGY

ulcerative colitis may need life long treatment. Atsigmoidoscopy examination biopsy and swabs and smearsare taken to look for any bacteria or parasite. There is nodiagnostic picture for idiopathic ulcerative colitis.

A 2. This lady had ulcerative colitis. Repeated stool testswere negative for any bacteria or parasite.

A 3. A small minority either have only one attack or thefirst attack is so fulminant that it requires surgery. For anover whelming majority the disease has remission andrelapses at various frequency all throughout their life. Mildto moderate acute attack can be managed by either topicalor oral salazopyrine or 5-aminosalicylic acid. For severeattack with severe diarrhoea and bleeding generally oral orsometimes IV steroids are required together with othersupportive treatment. Salazopyrine and 5-ASA compoundsare of proven value to reduce the frequency of relapse.

In long run a patient with colitis is at a risk of developingcolonic cancer. This is generally uncommon before 10 yearof colitis. The risk is proportional to the duration of thedisease and the extent of disease. It is for this reason thatcolitis patient should have 1-2 yearly endoscopy with serialbiopsy.

With recent refinement in surgery for colitis, especiallycreation of a pouch, has removed stigmata of ileostomyand surgery is more acceptable.

Page 55: Bedtime gastroenterology

43CASE 22

CASE 22: A STUDENT NURSE WITHFEVER AND DIARRHOEA

An 18-year-old student nurse was admitted with fever anddiarrhoea. She was well till about 3 weeks ago when shehad upper respiratory tract infection for which she receiveda course of antibiotic. About 4 days after stopping thetreatment. She started having ‘crampy” abdominal pain whichwas mainly in hypogastrium followed by diarrhoea. She tookhome remedies for this over next 2-3 days but consultedher GP when there was blood in her stools.

On examination she was febrile, abdomen was diffuselytender. She had leucocytosis.

Q. What is the likely diagnosis? How will you proveit? What is the treatment?

A. Although this could be ordinary antibiotic associateddiarrhoea; fever, diarrhoea, pain and tenderness in abdomenafter stopping antibiotics should lead one to the diagnosisof pseudomembranous colitis. It can follow any type ofantibiotic, but clindamycin most frequently, and can occurafter about a week of antibiotic therapy or sometimes, as inthis case, follow the course of antibiotics. Many a times thepresenting feature of diarrhoea and fever is so dramaticthat the initial illness is almost forgotten and under suchcircumstances this important condition will not be consideredDiagnosis is by sigmoidosopic examination where yellowishcoloured plaques are seen, stool examination revealsclostridium diffcile toxin but this is not routinely available.Clostridium difficile bacteria could be seen on microscopybut they may be commensals and not necessarily

Page 56: Bedtime gastroenterology

44 BEDTIME GASTROENTEROLOGY

responsible for the illness so their presence must be judgedin proper manner.

It is not necessary to have previous antibiotic exposureas, especially in olden days, any serious illness used togive rise to this condition.

Prognosis in previous days was poor with high mortalitybut this has steadily improved. Prognosis is worse in patientsdeveloping symptoms after antibiotic has been stopped orif the antibiotics were continued after the illness has started.Vancomycin or metronidazole taken orally is the drug ofchoice and now-a-days easily available also. Metronidazoleis much cheaper.

Page 57: Bedtime gastroenterology

45CASE 23

CASE 23: NEVER ENDING DIARRHOEAIN A MIDDLE AGED MAN

A 52-year-old man was referred for diarrhoea, fullness ofstomach and gradual weight loss over years and feelinggenerally unwell.

He was well till about 7 years ago when he had diarrhoealasting for more than a month. Small bowel showedthickening of ileal folds therefore he was diagnosed ashaving intestinal tuberculosis and was successfully treatedwith antitubercular drugs. At that time he had stronglypositive tuberculin test and D-xylose test was normal.Sigmoidoscopy and later a full length colonoscopy werenormal too. He remained well for about 2½ year when hedeveloped diarrhoea again. Ba-meal and follow throughshowed a short stricture of terminal ileum with proximaldilatation. Local physician suspected recurrence oftuberculosis and gave him another 9 months course ofantituberculous treatment. He promptly felt better. Subse-quently he had intermittent episodes of diarrhoea for whichvarious general practitioners, physicians and surgeons keptgiving him antibiotics with good effect. For a couple ofmonths before his present appointment he started havingdiarrhoea, fullness of stomach, rumbling in abdomen andgeneralised feeling of being unwell.

His appetite was present but he was afraid of eatingbecause of fullness in stomach and provocation of diarrhoea.He had no fever, no night sweats no haematemesis, melenaor bleeding per rectum. He had lost about 12 kg weight overlast 7 years but only 2 kg in last year. He opened his bowels6-7 times/day. His stools were pale, mushy and foul-smelling.He had never vomited. His abdomen was soft, loud

Page 58: Bedtime gastroenterology

46 BEDTIME GASTROENTEROLOGY

borborygmi were heard; no organs or masses were palpable.Rectum was empty and mucosa looked normal. Hishaemoglobin was 10.5 gm with microcytic hypochromicpicture. His stool for occult blood was negative X3. Bariummeal and follow through showed at least 3 strictures, 2 shortand one long = 7 cm in midileum. Terminal ileum wasmoderately dilated, and there was dilatation proximal to twostrictures.

Q 1. What are your thoughts about originaldiagnosis? What is happening over few yearsfollowing his treatment for the first time?

Q 2. What investigation will you carry out?Q 3. How will you treat this patient?

A 1. This man’s original diagnosis was intestinal tuberculosis.Normal sigmoidoscopy, colonoscopy, D-xylose test togetherwith barium follow through report and strongly positivetuberculin test, seen in conjunction with patient’s backgroundwould make intestinal TB a very reasonable diagnosis.Obtaining a biopsy may be difficult and many a times a trialof anti-tubercular drugs is reasonable option.

Probably this man had diffuse involvement of intestinewith miliary TB and was successfully treated with anti-TBdrugs. Treatment of intestinal TB many a times lead tostrictures which appear after a varying period. Crohn’sdisease cannot be ruled out. This man seems to havebacterial overgrowth. Every time he was given antibioticsincluding in the form of anti-TB drugs he improved promptly.

A 2. His investigation should include tests for bacterial overgrowth, H2 breath test being the easiest one of all. S folate

Page 59: Bedtime gastroenterology

47CASE 23

and red cell folate would show increase due to theirproduction by bacteria. His other biochemical parameterswould show evidence of malabsorption. It is still worthwhileto do sigmoidoscopy and biopsy because although Crohn’sdisease is uncommon in tropics it is not altogether absentand multiple non-caseating granulomas in a normal lookingrectum will clinch the diagnosis. Tuberculin test at this stagewill not be of much help.

A 3. Tetracycline 500-750 mg QDS for one month followedby 250/day as maintenance should help in bacteria over-growth syndrome. This patient was helped in the past by(inadvertent) antibiotics and it is worth a try. Vitamins andiron supplements will help; dietary advice for high proteindiet with low residue will be of help as well. Surgicalconsultation should be obtained now because his bariummeals follow through shows dilatation of terminal ileum andsome dilatation proximal to two other strictures. Althoughhe does not have ‘hard’ symptoms of obstruction his pro-gress is suggesting that he is going that way. Withwidespread acceptance of stricturoplasty resection is keptto minimum and therefore complications are less in long-term.

Page 60: Bedtime gastroenterology

48 BEDTIME GASTROENTEROLOGY

CASE 24: SUDDEN ONSET OF EXCRUCIATINGRIGHT UPPER QUADRANT PAIN

A 68-year-old man was admitted in emergency with severeright-side pain in the area stretching from right nipple toright upper quadrant below the costal margin. He felt littlenauseous with this pain. This pain had come all of a sudden.It was made worse by movement, breathing, coughing, etc.There was no history of injury, fever, prolonged immobili-zation, haematemesis, melena, etc. The pain was constant,excruciating and was not radiating. His appetite was good;he had loose motions for last couple of days. There was nohaematuria, pyuria or dysuria. He did not drink alcoholpresently, but used to drink heavily about 10 to 12 yearsago. On examination he laid motionless not even toleratingbed clothes or jarring produced by walking. His right lowerchest and right subcostal regions were exequisitively tenderpreventing any detailed examination. Spleen was notpalpable. He had florid spider naevi. Rest of the examinationincluding rectal examination done after a heavy dose painkiller was normal, his chest X-ray was normal.

Q. Do you think you can reason out his diagnosis.Investigation of choice?

A. As I write this case I clearly remember this patient whohad excruciating pain and lay absolutely motionless. Hewould not allow evening touching his right lower chest andright subcostal region. This together with florid spider naeviled us to postulate that he had chronic liver disease, i.e.cirrhosis, and one of the cirrhotic nodule, may be aftermalignant transformation had outstripped its blood supply

Page 61: Bedtime gastroenterology

49CASE 24

resulting in infarction and severe pain. He was givensupportive care and morphine injections to control his pain.Ultrasound showed a mass lesion with “mixed” echo. Biopsyshowed liver cells which were in advanced state of necrosis.Further selective staining was not possible on this specimen.Serum alpha fetoprotein values were 2000 ng/ml (0-10 normalrange). He stabilised for few weeks but was readmitted withgeneralised ill health and passed away. A postmortem hehad cirrhosis and necrotic area thought to be infarctedhepatocellular carcinoma. There was no evidence ofhaemochromatosis, hepatitis B infection, Wilson disease,Alpha-1 protease inhibitor deficiency, biochemically andhistologically. There was no colonic malignancy on post-mortem examination.

Page 62: Bedtime gastroenterology

50 BEDTIME GASTROENTEROLOGY

CASE 25: AN OLD MAN WITH SUDDENENLARGEMENT OF LIVER AND ASCITES

A 70-year-old gentleman was admitted with sudden right sidedabdominal pain, and nausea. He was a known case ofischaemic heart disease and had suffered an infarct 4 yearsago. He had diet controlled diabetes for about 12 years. Hewas on no medication. He had not drunk alcohol nor smoked.

On examination, he looked to be in pain. His pulse was112/min and blood pressure was 160/96 mm. His abdomenshowed a hard palpable liver with smooth surface. Liverwas very tender and hepato-jugular reflux was absent, otherorgans were not palpable. There was no ascites, cardio-vascular and respiratory systems were unremarkable. HisCBC showed leucocytosis of 14.5, Urea and electrolyteswere normal. His chest X-ray was normal, ECG showedold inferior infarction. Cardiac enzymes were normal.

He was given supportive treatment while waiting forfurther investigation. Next day he was found to havemoderate ascites which was a transudate.

Q. What is the likely diagnosis? Investigation youwould carry out.

A. Admitting house officer had diagnosed myocardlialinfraction however in the calm of morning and away fromanxious patient and his relative we thought differently. Manytimes I have advised my junior staff that they should firstthink of carrying the patient through the night and then thinkof diagnosis! That way they fare better than professors whowould have ten differential diagnoses and probably can’tcarry the patient through the night!

Page 63: Bedtime gastroenterology

51CASE 25

This patient firm tender liver first made us think ofmalignancy. However, when his liver functions came backin the afternoon we began to get a clear idea. His SGPTwas around 4000 iu, his GGT was 46iu and his Alk. Phos.was 196 iu Liver cell damage was obvious and than onnext day he developed ascites when we thought about Budd-Chiari syndrome. His ultrasound had, however showncongested liver and patent and slightly dilated hepatic veinand normal spleen. An isotope scan of liver was managednext. This scan showed normal uptake by liver. Generallyas the caudate lobe has a direct drainage in inferior venacava and takes up the isotope whereas the rest of the liverfails to take up the isotope in Budd-Chiari.

Liver biopsy was planned next, but by this time his liverenzymes were near normal. Biopsy showed zone 3haemorrhagic infarct, central veins were patent. Follow-up ultrasounds of liver showed patent hepatic veins andportal veins. Echocardiography showed mild cardio-megaly. His full blood count, including platelets wasnormal, autoantibodies were negative, and anti-thrombin-III and protein C levels were within normallimits. Thus although the picture was that of acute Budd-Chiari syndrome, we were unable to localise the block orthe cause of it. In view of his ischaemic heart disease, wehypothesised that he could have suffered transient rightsided heart failure without producing infarct (his cardiacenzymes and serial ECGs were normal) resulting in a pictureto similar to Budd-Chiari syndrome. His initial improvementwas interrupted by further episode of hepatic enlargementsimilar to the first one and the recovery was not so dramatic.He progressively became jaundiced, had gross ascites,

Page 64: Bedtime gastroenterology

52 BEDTIME GASTROENTEROLOGY

which was difficult to treat, and became cachetic. Shuntsurgery was thought very risky. He passed away within 6months of his initial presentation.

Page 65: Bedtime gastroenterology

53CASE 26

CASE 26: A CASE OF SUDDEN ONSETOF LEFT SIDED COLICKY PAIN

A 52-year-old gentle man was seen because of left sidedcolicky pain. He was well till that morning and had openedhis bowels as usual. Soon after he developed severe flankpain with vomiting. This was eased by pain killer. He feltquite well. Same afternoon he developed severe colickypain in left lower quadrant and vomited four times.

He drank occasional whisky but did not smoke.His examination showed that he was in pain. There was

mild tenderness in left lower quadrant but otherwise hisabdomen was soft. Rectal examination was normal. A plainX-ray showed gas in right side of abdomen no free gasor air fluid levels were seen. FBC showed mildleucocytosis and urine showed plenty of RBC.

Q. What is your diagnosis and investigations?

A. Severe colicky pain in flanks in an otherwise well lookingperson must raise suspicion of renal colic. Any severeabdominal pain would produce vomiting and should not foolyou in suspecting intestinal lesion straight away. It is onlywhen the pain is absent and the patient has vomiting thatyou suspect obstruction. In view on his alcoholic intakepancreatitis should be considered but the pain in pancreatitisis usually in the centre and crescendo in nature. Same istrue for other causes of acute abdomen.

Ultrasound scan is an important investigation which mayshow the stone or its effect like hydronephrosis. X-rayabdomen is a must, but if preparation is not good or if thestone is radiolucent you can be put on the wrong tract. Urine

Page 66: Bedtime gastroenterology

54 BEDTIME GASTROENTEROLOGY

examination is a must and RBCs in urine should arouseyear suspicion about renal colic. Intravenous urography canbe the next step then. Serum amylase can be of help whenthe above mentioned investigations have not helped.

This patient passed a small stone in next hours. IVUdone few days later was normal.

Page 67: Bedtime gastroenterology

55CASE 27

CASE 27: A YOUNG GIRL WITH RECURRENT BREATHLESSNESS AND ABDOMINAL PAIN

A 22-year-old girl was admitted in emergency with acutecentral abdomen pain. This built up over a couple of hours.This was “crampy” in nature, constant and was associatedwith nausea but no vomiting. There was no radiation of thispain. She denied any fever, haematemesis, melena, dysuria,haematuria, etc. She had opened her bowels regularly tillthat morning. Her periods were regular and painless and thenext one was not due for another week. She was single andhad never sexual intercourse.

In the past she was treated in local accident andemergency department for respiratory distress and was toldthat she had oedema of her voice box. Nearly seven yearsago she had her appendix removed for abdominal pain. Shehad otherwise good health.

On examination she was in distress. The pain was diffuseand there was some tenderness in the periumbilical region.Peristalses were present. Rectal examination was normal.Systems were normal.

FBC, urea, electrolytes and amylase were all normal.Urine was normal too.

Q. What did you think when reading this story?

A. With previous clear-cut history of laryngeal oedema,abdominal pain and appendicectomy and nonspecificabdominal pain with very little signs should alert you for thepossibility of angioneurotic oedema. Of course, othercommon problems like gastritis, renal colic, cholecystitis,and pancreatitis can also produce this type of picture in the

Page 68: Bedtime gastroenterology

56 BEDTIME GASTROENTEROLOGY

beginning but over a period only a few cases of these laterconditions will remain unlocalised.

Detailed inquiry of this patient revealed that one of her(paternal) uncle had died many years ago with respiratorydistress. Our provisional diagnosis therefore was intestinaloedema as a part of hereditary angioneurotic oedema. Whileawaiting blood count and X-ray abdomen results she wasgiven subcutaneous adrenaline with very good response.Blood was sent for C1 esterase measurement which waslow. She was put on regular Danazole under supervision ofan endocrinologist. Nearly a year after her admission she isdoing well.

Page 69: Bedtime gastroenterology

57CASE 28

CASE 28: A YOUNG BOY WITHRECURRENT PANCREATITIS

An 18-year-old boy came for consultation for recurrent,severe, central abdominal pain which was diagnosed aspancreatitis. He worked in garage for a year or so. He startedthese episodes of pain when he was only 13 years of age.The frequency varied from one a month to once every sixmonths. He did not drink alcohol or consumed tobacco inany from. He did not abuse any solvents. He ate a balanceddiet and appeared well nourished. He was on no medicationsapart for painkillers when he had an “attack”. There was nofamily history of similar illness. On examination he appearedwell and examination was normal.

Q. How would you advise his parents about furtherinvestigations? What is your thought about ERCP insuch cases? How does this type of pancreatitisbehave?

A. There are various biochemical, anatomical and otherconditions which can lead to pancreatitis. Nutrition, drugtaken, alcohol or tobacco consumed should be carefullylooked for. Hypercalcaemia, hyperlipidaemia, biliarysludge/stones, pancreas divisum, papillary stenosis,ascariasis, all can lead to pancreatitis and these needlooked into. Also there is a belief that persons working withhydrocarbons and other toxic chemicals are prone torecurrent pancreatitis, because body’s reserve of free radicalscavengers (antioxidants) is over whelmed by daily exposureto this chemicals making pancreas vulnerable to harmfulchain reactions started by free radicals leading to recurrent

Page 70: Bedtime gastroenterology

58 BEDTIME GASTROENTEROLOGY

bouts of pancreatitis. Careful history taking is thereforeessential. This patient was obviously exposed to petrol fumesbut his attacks started long before and there was noincreased frequency since he joined his present job. Inabsence of gallstones, gallstone pancreatitis seems unlikelyalthough many a times small crystals could be missed onsonography and recent introduction of endosonography couldlead to a few cases of pancreatitis, otherwise labelled asidiopathic, appropriately diagnosed as gallstone pancreatitis.ERCP/endosonography could show pancreas divisum,intraductal stones, papillary stenosis, intraductal ascariasis,etc. ERCP has unique role and on occasions can be usefulfor both diagnosis and therapeutic use. If however thecause is known ERCP is not mandatory till surgery isconsidered when knowledge of pancreatic essential.

Treatment is correcting the underlying cause. There is acurrent trend to give antioxidants like vitamin-E, A,bioselenium, etc. Pancreatic supplement is worth trying. Non-addicting pain killers are the corner stone in the majority ofcases. Deficiency should be treated accordingly. Many timespatients with recurrent attacks do not go on to develop chronicpancreatitis or pancreatic deficiency. In this idiopathicrecurrent pancreatitis severity and frequency will diminishover years.

Page 71: Bedtime gastroenterology

59CASE 29

CASE 29: BANK EXECUTIVE WHO SPENT SLEEPLESSNIGHT OVER A CYSTIC LESION IN THE LIVER

A 42-year-old bank executive was referred with a cysticlesion in the liver.

He was fit and well and gone for a routine medical check-up where he had made a casual inquiry about him havingany gallstones. An ultrasound was carried out which showeda cystic lesion with posterior acoustic enhancement. Thelesion was about 5 cm in diameter and was occupying rightlobe of liver near anterior surface. He spent sleepless nightand appeared promptly accompanied by his wife and in-laws next morning for consultation.

Q. How will you sort out this case?

A. With widespread use of ultrasound it is not uncommonto get referral like this. Congenital cysts, haemangiomasare quite common and together they are present in about7% of all ultrasound examination of liver. One would keepother causes of cysts in mind but generally experiencedultrasonologist can differentiate between various types ofcystic lesion. This patients had no predisposing factors forhydatid cyst and ultrasound picture was classic forhaemangiomas. No further tests are required in a “classical”case however if there is a doubt CT scan with IV contrast orMRI scan may be required. MRI scan is almost diagnosticof haemangiomas. Angiography may also give a charac-teristic picture of peripheral to central “blushing”. Needlebiopsy is almost never required.

Page 72: Bedtime gastroenterology

60 BEDTIME GASTROENTEROLOGY

CASE 30: SORE MOUTH AND THROATAND DIFFICULTY IN SWALLOWING

A 43-year-old male presented with a few days history ofsore mouth and throat and difficulty in swallowing with painin the area of lower retrosternum. He was previously fit andwell. About five days ago, he developed malaise and feverand noticed some vesicles on his lips and tongue. Soon hefelt sore throat and pain in retrosternum while swallowing.On examination he was febrile. His mouth showed scatteredsuperficial ulcers, some showing whitish scabs over them.Throat showed similar picture.

Q. What is your diagnosis? How will you prove it?Any other inquiry?

A. Candidiasis and herpes simplex would give this type ofpicture. White scabs made us to think of candida first. Wetook the scrapping to the laboratory and looked at it undermicroscopy which did not show any fungus; therefore herpessimplex was the working diagnosis. Oesophagoscopy showeddiscreet superficial ulcers above gastroesophageal junction.Biopsy showed eosinophilic inclusion bodies and viral titreswere confirmatory. He was given Acyclovir and made a goodrecovery. The important thing in these types of infections isthe possibility of immunosuppression and detailed inquiryabout possibility of immumosupperssion should be madeand in highly suspicious cases, permission for HIV testingshould be asked for. This patient had no history which wouldsuggest immunosuppression and hence detailed tests werenot done. His GP was alerted however who reported patient’ssatisfactory general condition about a year later.

Page 73: Bedtime gastroenterology

61CASE 31

CASE 31: CHRONIC LEFT SIDED ABDOMINAL PAINA 65-year-old lady was seen in the clinic with moderately severepain in her left side of abdomen for many years. She was con-stipated as well. Her appetite and weight were steady. She hadno bleeding per rectum. Her full blood count, urea, electrolytes,liver function tests were all normal. Her abdomen was soft andno masses were palpable. There was some tenderness on herdesconding colon. Rectal examination was normal.

Q. How will you investigate and treat this patient?

A. Old lady like this visiting various specialists, hospitals,clinics is not an uncommon site. Thick files following themwill invariably dishearten you as they would have to yearcolleagues. One advantage of being a consultant is thatyou can always ask your residents to “get to the bottom” ofthese problems. Diverticular disease or irritable bowelsyndrome are the two most common underlying problemswith these type of complaints. With un-alarming story likethis routine is to be methodical and obtain a full blood count,faecal occult blood X3 and perform a flexible sigmoidoscopy.Further investigations depend upon these investigations andpatient’s progress. This lady had innumerable sigmoido-scopies, couple of barium enemas and a few colonoscopies,ultrasound scans and what not. She knew all about anti-spasmodics, fibre preparations, laxatives and high fibre dietsand the rest. She had severe diverticular disease of hersigmoid colon. Combined meeting with surgeons are veryhelpful in these types of cases. In view of her severedistress, surgical resection was considered as the only optionleft. She is being followed up for more than two years afterresection and her pain is more manageable than before.

Page 74: Bedtime gastroenterology

62 BEDTIME GASTROENTEROLOGY

CASE 32: AN UNMARRIED MAN WITHODYNOPHAGIA AND DYSPHAGIA AND

UNEXPLAINED DIARRHOEAA 30-year-old man presented to medical unit for dysphagia.He was single and had no previous health problems. Henoticed increasing difficulty in swallowing over a month. Thisdifficulty was mainly to solids. There was associated painas well. He was referred for endoscopy.

At endoscopy he had (one of the most fulminant) oeso-phageal candidiasis and a hard fibrous stricture. Biopsiesdid not show any malignancy. Repeated attempts at stricturedilatation were unsuccessful over next two months. Meanwhile he had severe diarrhoea. No organisms were seen orcultured. Colonic biopsy and colonoscopy were unremark-able. D-xylose and screening tests on stool for excess fatwere normal. Liver function, urea, electrolytes, glucose wereall normal. Tuberculin test was negative at 48 hours. He wassingle and had homosexual relationship. He was treatedwith Flucanazole 400 mg/day and metronidazole. Ba-mealand follow through done with difficulty was unremarkable.His stool (X 6) was negative for any bacteria or parasites.His oesophageal candidiasis responded to flucanazole. Hisdiarrhoea was treated symptomatically with codeinephosphate. He was referred to oesophageal surgeon forpossible resection of his stricture.

Q. What would you want to do with such a case?

A. This was seen when AIDS was in its infancy and notmany drugs were available except for zidovudine. He hadone of the worst oesophageal candidiasis I have ever seen

Page 75: Bedtime gastroenterology

63CASE 32

and with his lifestyle and diarrhoea of uncertain aetiology;we were worried about him having AIDS. In fact,oesophageal candidiasis is an AIDS defining diagnosis. HIVtest, therefore requested which were positive and in suchcircumstances repeated dilatation is probably the onlychoice.

This patient was lost to follow-up.

Page 76: Bedtime gastroenterology

64 BEDTIME GASTROENTEROLOGY

CASE 33: A MIDDLE AGED MAN WITH WEIGHTLOSS AND A GIANT GASTRIC ULCER

A 42-year-old male was admitted with haematemesis andmelena. He was unwell for about 3 months when he startedfeeling tired. About 2 months ago he complained ofbackache, in lower thoracic region. Soon afterwards hecomplained of dyspepsia. He was treated with H2-RA withgood relief in dyspepsia. He stopped treatment on his own.A couple of weeks later he complained of epigastric painand increasing backache. He was seen by another physicianwho found some tenderness just above umbilicus but restof the examination was normal. Ultrasound showed bulkyhead of pancreas but this was not confirmed on CT scan.He was treated with Ranitidine following a barium-mealexamination which was reported as normal. This backachegot much worse over next few days and he also developedfever which ranged from 102–103F. Blood cultures X2, urinemicroscopy and throat swab were taken but wereunremarkable. He continued having fever, abdominal painand backache, till he had haematemesis followed by melena.

There was no history of previous dyspepsia. His appetitehad gone down and he had lost about 8 kg in weight. Hewas not taking any NSAIDs nor did he take any alcohol ortobacco. There was no past history of any illness or operation.He had no family history of note.

At endoscopy he was found to have a greater curveulcer measuring approx. 5 cm × 3 cm with prominent rolledout margin and a nodular base which showed a clot. Rest ofthe examination was normal. Multiple biopsies were takenfrom the edges.

Page 77: Bedtime gastroenterology

65CASE 33

Q. What do you suspect? What is the outcome?

A. A large (giant) gastric ulcer with rolled out edges. In ayoung person is highly suggestive of lymphoma, which thispatient had. Carcinomas generally don’t have prominentrolled out edges and they become more frequent after theage of 50.

Prognosis in case of a lymphoma depends upon celltype and extent of involvement only. Cure rate would be ashigh as 70%. In any case surgery +/– radiotherapy andchemotherapy used judiciously gives average 5-year survivalin the vicinity 50%.

Page 78: Bedtime gastroenterology

66 BEDTIME GASTROENTEROLOGY

CASE 34: A JUDGE WHO COULD NOT PERFORMHIS DUTY DUE TO EXTREME TIREDNESS

A 60-year-old man presented to his physician with extremetiredness. He had been previously fit and well except forone episode of left ventricular failure about 9 months ago.He took regular exercise and could perform his duty as ajudge. In retrospect however he thought he was probably99% fit for previous few weeks. He did not smoke or drankalcohol. He was not taking any drugs. His appetite was goodand his weight steady. His bowels were on constipation sidefor quite a while. There was no history of haematemesis,melena or bleeding per rectum. On examination he was verypale and there was smooth hepatosplenomegaly with liverspan of around 17 cm. Rest of the examination was normalincluding rectal examination. His Hb was 4.7, WCC 7.8 andfilm showed few nucleated red cells, with anisocytosisand poikilocytosis. MCV was 107 f1, MCH was 29, MCHC31.2, platelets 136, ESR 47mm/hr., S.bil 10 mg, SGPT 37iu, GGT 12 iu Alk phos. 127 iu Total protein 7.8, albumin3.0 and globulin 4.7. Urine showed trace of protein.Ultrasound showed diffuse liver enlargement. Kidneyslooked of normal size No retro peritoneal lymph nodesor other abnormality were detected.

Q. What is your line of thinking here? Investigations?Diagnosis?

A. Haemoglobin of 4.7 gm is suggestive of a definiteproblem in this patient. MCV is 107 fl, which could be due topresence of premature cells in the circulation. In perniciousanaemia MCV is much higher. There is a smooth hepato-

Page 79: Bedtime gastroenterology

67CASE 34

splenomegaly which is due to infiltration by something.Lymph nodes are not enlarged, WCC is not raised, andglobulin is raised. It is thus very tempting to put all thesethings together and suspect that bone marrow is infiltratedor there is altered haemopoiesis and lymphoid element couldbe involved. Same process has involved liver and spleeneither directly or by way of their metabolic product, e.g.amyloid.

Investigation, therefore, should be (1) protein electro-phoresis in blood and unine, s.B-12 and Folate, s.Ferritin(or TIBC and s.Fe) FOBx3, Bone marrow for haematologicalproblem and (2) Rectal biopsy +/- liver biopsy for hepato-splenomegaly thinking that amyloidosis could be responsiblefor it. These investigation should lead to clear idea as towhat is causing the problem or what should be done next.The results were:1. S.B12 and Folate, Red cell folate- FOB X3, s. ferritin: all

within normal limits.2. Protein electrophoresis: a monoclonal peak seen3. Bone marrow: plasma cell myeloma4. Rectal biopsy: Amyloid seen

The final diagnosis therefore is multiple myeloma withamyloidosis.

Page 80: Bedtime gastroenterology

68 BEDTIME GASTROENTEROLOGY

CASE 35: PRIMIPARA WITHJAUNDICE IN LAST TRIMESTER

A 31-year-old primipara was seen in obstetric ward forjaundice. She was previously fit and well and apart frommild hypertension in pregnancy, her obstetric record wassatisfactory. She started with “flu” about 8 days ago. Shehas slight cough, fever and malaise for which she took coupleof paracetamol and hot lemon sips available over the counter.She lost her appetite and felt nauseous and started vomitingwith increasing frequency over previous three days. Shedeveloped right upper quadrant pain. She was given vitaminsand metochlopromide by her GP and blood samples weresent for CBC and liver function test. Her GP visited herdaily and on the day of admission he noticed mild jaundiceand received her blood results and hence the admission.There was no past history of acute or chronic hepatitis,injections, blood transfusion or recent travel to areas withendemic infective hepatitis. Her sexual partner was well.She was on iron and folic acid.

Her examination revealed mild ankle oedema and rightupper quadrant tenderness but the liver was not palpable.Liver dullness was present.

Her investigations were:Hb: 12.4 gm, WCC: 13,300, Plt: 84,000, Reticulocytes:

2%, ESR: 41 mmFilm showed: Neutrophilia, thrombocytopaenia, normo-

blasts and basophilic stippling.PT: 15/14 Sec, APTT: 41 sec. (upper limit 37 Sec), FDP

were within normal rangeS. bil: 2 mg, ALT: 297 iu, GGT: 201 iu, Alk Phos: 104 iu,

Alb: 3.9 gm, blood sugar: 52mg. Urea: 40 mg, Creatinine:

Page 81: Bedtime gastroenterology

69CASE 35

1.2 mg, uric acid: 7.mg, electrolytes: Normal, Urine: trace ofprotein only

Q. How will you manage this case? What else wouldyou want for investigations? Prognosis?

A. Jaundice in pregnancy could be coincidental or relatedto pregnancy. The former could be infective hepatitis, alco-holic hepatic, flare-up of chronic hepatitis or obstructivejaundice, etc. These therefore must be kept in mind andinvestigated appropriately. Jaundice associated withpregnancy could be because of hyperemesis gravidurum,toxaemia, acute fatty liver and recurrent cholestatic type.Urgent ultrasound is needed which showed hyper-echogenicity. Gallbladder and biliary tree were normal.Hepatitis serology was sent but results could not be obtainedimmediately. In view of blood picture and mild disturbancein clotting profile a fine needle liver biopsy was carried outunder ultrasound guidance after fresh frozen plasma. Thisconfirmed acute fatty liver of pregnancy. Hepatocytes wereswollen and showed micro-vesicular fat droplet. There wasno parenchyma necrosis.

In consultation with the obstetrician and neonatologistthe patient was offered induction which was accepted. Therewas dramatic recovery postpartum. In such cases death isdue to renal failure or extensive bleeding associated withdisseminated intravascular coagulation. Subsequentpregnancies may also be affected. Acute fatty liver ofpregnancy is more common with primipara, male child andin twins. Recently more reports are coming wheresubsequent pregnancies are also affected. This is a diseaseof last trimester and affects younger women. Early

Page 82: Bedtime gastroenterology

70 BEDTIME GASTROENTEROLOGY

recognition is important otherwise the morbidity and mortalityare quite high for the mother. The child nevertheless stillhas very high mortality.

Aetiologically this is one of the mitochondrial cytopathy.Generally the mother is a heterozygous and carries a childwhich has long chain 3-hydroxyacyl coenzymedeficiency. When this happens there is defective fatty acidoxidation and infiltration in the liver cells. When the child isunaffected mother being heterozygous escapes. Inmonozygous twin pregnancy understandably there are moreproblems.

Sodium valproate and toluene poisoning produce almostidentical clinical picture.

The blood film picture in this patient is an alerting featurefor acute fatty liver of pregnancy.

Page 83: Bedtime gastroenterology

71CASE 36

CASE 36: MAN WITH RECURRENTPOST-PRANDIAL VOMITING

A 46-year-old man presented with recurrent post-prandialvomiting for over 6 months. He said many times, but notalways, he felt heavy feeling in his epigastric region, asensation akinto a ball in his epigastrium and would vomitrecently eaten food. He would feel fine afterwards. Therewas no blood or significant amount of bile in his vomits. Hedenied any intercurrent abdominal pain, burning, regurgitationor nocturnal pain. He emphasised that at other times hecould eat and drink normally. His weight was stable, appetitegood and he had no fever. He felt marvelous otherwise. Hisupper GI endoscopy up to 3rd part of duodenum was normal,rapid urease test in antral biopsy was negative. Ba-mealfollow thro’ was reported normal and ultrasound showed noabnormality in upper abdomen. His FBC, ESR, urea,electrolytes fasting sugar and liver function tests werenormal.

Q. What do you think about his underlying problem?Investigations?

A. This patient could have underlying motility problem.However, one should also keep underlying psychiatricproblems in mind which can manifest in most bizarre fashion.Generally their story is vague and you feel something “roundabout” the way they describe their symptoms. Good longexplorative chat with the patient or their close associatescan be invaluable. Ideally he should have manometric studyof his upper GI tract wherever possible but as this is notavailable nearby, we settled for sonographic assessment

Page 84: Bedtime gastroenterology

72 BEDTIME GASTROENTEROLOGY

following a “standard meal”. This suggested delayed gastricemptying. He was put on regular cisapride, a prokinetic drugfollowing which there was some improvement in hissymptoms. Perhaps in future better investigations may leadto more specific treatment.

Page 85: Bedtime gastroenterology

73CASE 37

CASE 37: A CASE OF RECURRENTDUODENAL ULCERATION

A 53-year-old man came for recurrent epigastric pain. Hehad at least 4 documented ulcers in last 15 years. He wastaking intermittent H2 RA and antacids and latelyOmeprazole. He would improve while on treatment butrelapse within few months of stopping treatment.

Q. How will proceed to investigation him?

A. Recurrent ulcer is a fairly common phenomenon. Inquiryshould be made about smoking/tobacco consumption,alcohol and aspirin or similar drugs, all of which could beresponsible for either difficulty in healing of ulcer or recurrentulcers. Classically gastrinoma producing Zollinger-Ellisonsyndrome would immediately come to mind but in majorityof cases we find that H. pylori is responsible for recurrentulcer and therefore H. pylori should be looked for in gastricantral biopsy. Gastrinoma should be screened for next. Itmay exist individually or in association with multipleendocrine neoplasia, ‘MEN’, syndrome. Fasting calcium,gastric acid output studies, serum Gastrin levels and otherhormone studies will be required for its diagnosis. This patienthad his serum calcium done which was 18 mg whichwas very high and hence he was started on saline diuresisand sent to endocrine unit for further evaluation. He turnedout to have MEN-I which is parathyroid adenoma andGastrinoma. His parathyroids were resected but Gastrinomawas not localised. He was one of the first patients to begiven Omeprazole for Gastrinoma way back in 1986 whenthis drug was an experimental drug.

Page 86: Bedtime gastroenterology

74 BEDTIME GASTROENTEROLOGY

CASE 38: A MENOPAUSAL LADY WITH JAUNDICEA 45-year-old lady presented with jaundice. She was unwellfor sometime with arthralgia mainly of her small joints ofhands, and malaise. She had a flu like illness about a fortnightago followed by jaundice. On examination she had moderatejaundice with hepatomegaly of about 7 cm below costalmargin. The liver was smooth, non-tender, and firm withrounded edges. Spleen was not palpable. There were fewspider naevi on her back and shoulders. Small joints of herhands were slightly tender but there was no gross swellingor deformity. Rest of the examination was unremarkable.S.bil was 3 mg, SGPT 742 iu, Alk. Phos 123 iu, PT 18/16sec. HB 11.3 gm, WCC 12.4, ESR 46 mm.

Q. What inquiries will you make? What otherinvestigation you will order? What is the likelydiagnosis?

A. Inquiries should be made about previous hepatitis, bloodtransfusions, etc. for possible Hep-B/C infection. Drugshistory should be carefully elicited, to rule out drug inducedchronic hepatitis. Similarly family history of neuro-psychiatricdisturbances with or without, jaundice is important withregards to Wilson’s disease. Photosensitivity, pleurisy,underlying ulcerative colitis, measles rubella or cytomegalo-virus infection in the past may be responsible. Severeemphysema may suggest Alpha-1 protease inhibitordeficiency.

Further investigation should be serum globulin, ultra-sound of liver, spleen and biliary tree. HBsAg, HCVantibodies, anti-nuclear factor, s. copper, Ceruloplasmin andurine copper and anti-smooth muscle antibodies. Liver biopsy

Page 87: Bedtime gastroenterology

75CASE 38

is a must to assess the extent of damage and presence orabsence of cirrhosis. It can also give clues about Hep Binfection. In general initial work-up suggests appropriate lineto follow and not all of the above mentioned investigationwill be required. Once initial investigations give a pointerfurther diagnostic tests will be required accordingly.

This patient had chromic active hepatitis of auto-immunevariety. Both her ANF and anti smooth muscle antibodieswere positive. Anti-double stranded DNA was alsopositive thus she had systemic lupus erythematosus. Sheinitially responded to steroids only but increasingly sherequired higher doses for maintenance and therefore alsoput on Azathioprine with good effect.

Page 88: Bedtime gastroenterology

76 BEDTIME GASTROENTEROLOGY

CASE 39: A YOUNG GIRL WITH RECURRENTJAUNDICE AND VAGUE RUQ MASS

A 24-year-old lady was admitted because of jaundice. Shedescribed herself as a healthy person who was prone todevelop “jaundice” now and again. A typical attack wouldstart with low grade fever and RUQ pain colicky in naturefollowed by jaundice. She had noticed fullness in RUQ afew times when she had an attack. She would be givensymptomatic treatment and bed rest with gradualimprovement in her condition. Her GP suspected Gilbert’ssyndrome however thought of obtaining a second opinion.

Detailed inquiry as regards to infective hepatitides wasnegative. She was not taking any drugs apart from thosewhich she was prescribed at the time of illness. There wasa vague mass in epigastrium which was slightly tender. HerS.Bilirubin was 4 mg, ALT 56 iu, GGT 142 iu, ALK phos279 iu, FBC, urea and electrolytes were normal.

Q. What further investigations will you order? Whatis your differential diagnosis? What is the likelydiagnosis?

A. Ultrasound examination is next important investigation.As she has obstructive jaundice which is recurrent, asso-ciated with RUQ pain, fever and epigastric “mass” one mustthink of recurrent cholangitis and choledochal cyst shouldbe considered at her age. Ultrasound scan suggested acystic lesion probably associated with lower CBD. ERCPwas next diagnostic investigation carried out after a week’santibiotic treatment. This showed an apple sized diverticulumfrom lower CBD, the type-II choledochal cyst. This tallied

Page 89: Bedtime gastroenterology

77CASE 39

with her story of feeling a “mass” in her upper abdomenduring jaundice. She was operated upon with resection ofcyst to avoid future risk of developing carcinoma. With adventof MRI one could by-pass ERCP for diagnostic purpose.

Page 90: Bedtime gastroenterology

78 BEDTIME GASTROENTEROLOGY

CASE 40 : A YOUNG BOY WITH RECURRENTINTESTINAL OBSTRUCTION

An 18-year-old boy came with history of recurrent abdominaldistension and vomiting. He had this problem for a few yearsnow but was getting more troublesome to an extent that hewas afraid of eating although no specific food constantlybrought this on. He was loosing weight as a result. He broughtwith him several abdominal X-rays taken during theseepisodes, which showed small bowel obstruction. Severalbarium meals follow through examinations failed to showany reason for obstruction. He had several negativelaparotomies.

He was second of three children and all except him werekeeping well. There was no family history of similarcomplaints. He had patent ductus arteriosus operated inchildhood. Examination was unremarkable.

Q. Likely diagnosis? Investigations?

A. Chronic intermittent intestinal pseudo-obstruction is themost likely diagnosis here. The story is very suggestive ofintestinal obstruction and it is quite common for thesepatients to have at least one negative laparotomy and severalbarium (meal/enema) examination till the” penny drops”. Itcan be sporadic or familial with autosomal dominant orrecessive pattern. It could be because of disorders of smoothmuscles or myenteric plexus. The whole GI tract and some-times other viscera like bladder, ureters, etc. can be involved.

Investigations should include tests for thyroid functions,amyloidosis, and systemic sclerosis. Also motility of therest of the gastrointestinal tract, e.g. oesophageal, colonic

Page 91: Bedtime gastroenterology

79CASE 40

should be checked as they may also be involved. This canbe checked by barium studies, ultrasound studies, pressuremanometry, radio-opaque marker studies (most convenient)radionuclied scanning, etc. If possible endoscopic oroperating biopsy of involved organ should be carried out forprecise diagnosis.

Treatment is mainly palliative, however, recentintroduction of Cisapride may help some patients.

Page 92: Bedtime gastroenterology

80 BEDTIME GASTROENTEROLOGY

CASE 41: AN OLD LADY WITH DIARRHOEAAND SUDDEN INCREASE IN THE SIZE

OF A SWELLING IN HER NECKA 63-year-old lady came with persistent diarrhoea for abouta year and recent weight loss of over 10 kg. She had aswelling in right side of her neck for 16 years (!) which wasgetting bigger in last few weeks.

She was a frail lady who denied any cough, haemoptysis,hoarseness of voice, bleeding P/R, anorexia, etc. Onexamination she had a swelling in the right lobe of thyroid ofabout a tennis ball size. It was firm and immobile. A thrillwas easily palpable. Lymph nodes were not palpable.Abdomen was soft, no organs were palpable. Rectal exami-nation was normal. Chest X-ray, abdominal ultrasoundand colonoscopic examination were all normal. Herthyroid functions showed mild hyperthyroidism. S.Calcium was normal. FBC showed mild anaemia.

Q. What is the likely diagnosis? Investigations?

A. She has thyroid malignancy with diarrhoea withouthyperthyroidism and this is most likely to be medullarycarcinoma of thyroid. Serum calcitonin will be helpful. Thispatient had very high S. Calcitonin level and fine needleaspiration showed cells suggestive of medullary carci-noma. Medullary carcinoma accounts for 5-7% of thyroidmalignancies. It could be a part of MEN associated withparathyroid and/or pheochromocytoma. Other hormones/chemicals secreted are histamine, prostaglandin, ACTH andother polypeptide hormones. Where facility exists, thesehormones should be assayed.

Page 93: Bedtime gastroenterology

81CASE 41

It can be sporadic or familial with autosomal dominanceand in the later case usually bilateral and multifocal.

Sometimes provocative tests using calcium orpentagastrin may be required.

If isolated and localised, surgery offers good prognosis.If part of MEN syndrome, the prognosis is not so good.Prostaglandin inhibitors are sometimes helpful for severediarrhoea.

Page 94: Bedtime gastroenterology

82 BEDTIME GASTROENTEROLOGY

CASE 42: A YOUNG GIRL WITH LOWERABDOMINAL PAIN AND DYSPAREUNIA

A 20-year-old married lady was referred because of lowerabdominal pain. She had menarche at the age of 14 years.She was previously fit and well till about 2 years ago whenshe first noted left sided lower abdominal pain. The painwas “dull ache” most of the time but would get worse onexertion forcing her to abandon her work and rest for fewdays. This pain had no definite relationship with her menstrualcycle. There was no bowel disturbance. Her waterworks wasalright, her appetite was good, weight steady and she deniedhaving any fever. NSAIDs were prescribed by her GP whichhelped her temporarily. Her FBC, ESR, urea, electrolytes,LFTs, urine and stool for FOBx3 and microcopy wereall within normal range.

Q. What would you consider as the cause/s in hercase? How will you proceed with investigations?

A. In a young, otherwise healthy woman gynaecologicalproblems or irritable bowel syndrome are the two commonestcauses of lower abdominal pain. Detailed history and exami-nation including pelvic and rectal examination together withfew simple blood tests like FBS, ESR, stool for microcopyand occult blood and urine microscopy are generally veryuseful indicators for further investigation. It there was asense of incomplete evacuation, mucus discharge irritablebowel syndrome would be suspected. Pelvic inflammatorydisease would be picked up on examination and blood tests+/– microscopic examination of any discharge. Ovariancongestion is an important cause lower abdominal pain. It

Page 95: Bedtime gastroenterology

83CASE 42

may be unilateral or bilateral. Tenderness on ‘ovarian point’at the junction of upper third and middle third of a line fromumbilicus to anterior superior iliac spine, is a valuable signso is painful movement of cervix and tenderness and painon palpation of ovaries. Ultrasound scan may showpolycystic ovaries. There is generally history of infrequentintercourse due to dyspareunia. Depression is fairly commonand this may not all be due to pain. US scan in this case didnot show polycystic ovaries. She was diagnosed to haveovarian congestion and treated with medroxy progesteroneacetate and psychotherapy with good results.

Page 96: Bedtime gastroenterology

84 BEDTIME GASTROENTEROLOGY

CASE 43: AN OLD MAN WHO GOT HISLIVER TESTS DONE OUT OF CURIOSITY

A 71-year-old man came for consultation because of“abnormal” liver function tests. Eight years ago he had type-B hepatitis from which he made a very good recovery. Ayear later he, out of curiosity, got his “liver tests” done whichare tabulated bellows. Over next eight years he remainedvery well. On examination he did not reveal any stigmata ofchromic liver disease. Liver was not palpable; there was noevidence of splenomegaly or ascites. An oesophagoscopywas normal.

Liver function tests

8 yr 7 yr 5 yr 3 yr 0 yr

S.bil 10 mg 2 mg 2 mg 2 mg 2 mgSGPT (ALT) 2684 iu 102 97 99 98GGT 1933 iu 84 88 67 69Alk. Phos 267 iu 122 104 100 102S.Alb 3.03 gm 3.72 3.6 3.6 3.72S.Glob 2.71gm 2.5 2.7 2.8 2.94PT 19/14 Sec. 14/12 15/12 15/12 14/12HBsAg +ve +ve +ve +ve +ve

Q 1. What is the likely diagnosis?Q 2. What other tests you consider useful?Q 3. How will you treat it. ?

A 1. Chronic hepatitis due is Hep B virus is the most likelydiagnosis.

A 2. HBeAg detection, HBeAb, HBV-DNA and liver biopsyare other important investigations one would like to carry

Page 97: Bedtime gastroenterology

85CASE 43

out in addition. HBeAg positivity suggests infectivity andviral replication and determines treatment suitability. Whentreatment is contemplated viral DNA levels are important.Liver biopsy is also important for staging of the disease. Ifinterferone is contemplated, endoscopy to assess the sizeof varices is mandatory and if they are very large they needto be tackled first.

A 3. Treatment is generally determined by HBeAg and SGPTlevels and on viral load. Currently Interferon and lamivudineare two important drug used in chronic liver disease due tohepatitis B.

Once an index case is found it is mandatory to look forHBsAg in other close contacts and they be followed up asper their findings and if required treated or vaccinated.

Page 98: Bedtime gastroenterology

86 BEDTIME GASTROENTEROLOGY

CASE 44: A YOUNG BOY WITH PROGRESSIVEJAUNDICE AND NEGATIVE KF RING

Consultation was invited on paediatric ward for a 10-year-old boy with jaundice. He was well before 3 months when heconsulted a local doctor for nausea and yellow urine.Jaundice was diagnosed and he was treated conservatively,however, over next 3 months jaundice progressedrelentlessly with development of ascites.

He was 2nd of 5 children. There was no history ofconsanguineous marriage in his parents. His developmentwas normal; he had no previous major illnesses. He had notreceived any injection or blood transfusion in last 2-3 years.Other siblings were well.

On examination, he was deeply jaundiced. Tongueappeared pale. There was minimal ankle oedema. No bruises,spider naevi, gynaecomastia, etc. were detected. Moderateascites was detected. Liver was enlarged and tender, spleenwas not palpable. Slit-lamp examination did not reveal anyKayser-Fleischer ring. His relevant investigations were asfollows.

3 mths 1 mth 1/2 mth on consultation

Hb - 8.6 gm/21 9.2 9.7 gm/dlSmear Retics 4% 8% 8%S.bil 4 mg 20 mg 17 mg 22 mgSGPT 273 iu 1741 iu 1844 1070 iuAlk Phos 127 iu 162 iu 154 138 iuAlb/Glob - 3.6/2.4 gm 3.2/2.7 2.7/2.8P.T. - 27/14 sec 27/14 29/13 secHBsAg -Ve -Ve -Ve -Ve

Page 99: Bedtime gastroenterology

87CASE 44

Q 1. What causes would you consider in any youngpatient?

Q 2. What further investigations would you consider?Q 3. What is the likely cause here?

A 1. Apart from A to E (F) viruses, Epstein-Barr virus,Cytomegalo virus, and other usual causes of jaundice mustbe kept in mind. Importantly with anaemia particularly ofhaemolytic type, autoimmune conditions, Wilson’s disease(Where haemolysis is due to toxic effect of copper and notimmune mediated) and in tropics malaria (which gives riseto haemolytic jaundice and blood picture which may suggesthepatitis) must be considered. Needless to say that athorough history is absolutely essential.

A 2. Relevant viral titres are mandatory. For haemolyticanaemia Coomb’s test, ANF and other tests as indicatedwill be required. Thick smear should be prepared for malarialparasites. Urinary copper excretion and S. copper and Cerulo-plasmin would be indicated. Liver biopsy with measurementof copper content is very helpful when Wilson’s disease isstrongly suspected and the facility is available.

A 3. With haemolytic anaemia and hepatitis of progressivenature Wilson’s disease was very likely. Appropriate investi-gations confirmed it. Kayser-Fleischer ring is not a must fordiagnosing Wilson’s disease. WARNING: - Discuss with yourchemical pathology department regarding your requirementsas traces of copper in needles or container can ruin yourefforts!!

Page 100: Bedtime gastroenterology

88 BEDTIME GASTROENTEROLOGY

CASE 45: A MIDDLE AGE MAN WHOSEHOLIDAY TURNED INTO A NIGHTMARE

WITH SEVERE ABDOMINAL PAINA 52-year-old gentleman was admitted following a home visit.He was previously fit and well. About 2 weeks ago while onholidays, he developed severe abdominal pain of diffusenature. He was seen by a local doctor and pain killing injectionswere given. This pain recurred again and he tried to contacta doctor but could not find one. The hotel manager gave hima pain killer injections as he was used to pain killers in doctor’absence. It took away severe pain, however, he was left withdull ache. He decided to return home. He felt weak for abouta week to ten days thereafter. About a fortnight later he noticedswelling of his abdomen and consulted his GP who suspec-ted subacute intestinal obstruction and arranged a home visit.

He was well built and quite comfortable and afebrile.General examination was unremarkable. Abdomen showedmoderate ascites. Bowel sounds were normal, systemicexamination was otherwise normal. His FBC was showingHb 14.4, TC: 8100 with normal differentials, ESR was 43mm and Platelets 2, 64,000. X-ray abdomen was notshowing any obstructive features. Ultrasound confirmedmoderate ascites. Liver, gallbladder and CBD appearednormal. Pancreas was not well visualised but grosslyno major pathology was obvious. Urea, electrolytes andLFT were normal. Ascitic fluid was straw coloured withprotein 3.8 gm, cells less than 3 per high power fieldand Gram stain was negative for bacteria and AFB. Culturedid not grow any bacteria. About 4 weeks later he developeda mass in epigastrium.

Page 101: Bedtime gastroenterology

89CASE 45

Q. What investigations would you like to carry out?What is your differential diagnosis? What would bethe mass?

A. Ascites followed sudden severe diffuse abdominal painin a previously fit person. Pancreatitis, spontaneous bacterialperitonitis, Budd-Chiari syndrome would come to your mindbefore other causes like cirrhosis, tuberculosis, malignancy,etc. S. amylase is a must in any severe abdominal pain.The same should be done in ascitic fluid if ascites followssevere abdominal pain. Budd-Chiari or similar vascularproblem of liver will generally produce abnormality of liverfunction tests. Expert sonography with or without liverscintiscan is very helpful.

Here S-amylase was marginally raised but ascitic fluidamylase was markedly raised suggesting pancreatic ascites.He developed pseudocyst of pancreas which presented asepigastric mass. No cause was found for his pancreatitis.

Page 102: Bedtime gastroenterology

90 BEDTIME GASTROENTEROLOGY

CASE 46: RECURRENT CENTRAL ABDOMINALPAINS WITH NO LOCALISING SIGN

A 37-year-old male executive came because of abdominalpain. This started approximately two weeks ago when henoticed epigastric “burning” pain, relieved to some extentby eating. He also had colicky central abdominal pain. Hewas treated with Ranitidine. He felt better vis-à-vis epigastricpain but kept complaining about colicky pain. As he wasresident of tropics, he was prescribed a course of Metronia-dazole + Diloxanide furoate although his stool examinationwas negative. Over next few days he kept complaining ofepisodes of colicky pain and hence the referral.

He was a fitness enthusiast without any vices. He wasafebrile, abdomen was soft, and no masses were palpable.FBC, urine microscopy, stool microscopy, urea, electro-lytes and S. amylase (while not in pain) were withinnormal range. Sonography for liver, gallbladder,pancreas, Kidney, bladder and appendix was negative.Over a week he kept having recurrent fleeting colicky painwithout any localising signs. Repeated urine microscopyand amylase were normal, white cell count fluctuated within11-13000, and temperature remained normal.

A barium meal and follow thro’ examination was carriedout.

Q. What could be the underlying cause?

A. Recurrent genuine abdominal pain in a fit looking personwas very puzzling. Renal colic and biliary colic were reason-ably well excluded. Amoebiasis was well treated. Ischaemia,etc. seemed very unlikely. There were no localising signs

Page 103: Bedtime gastroenterology

91CASE 46

for appendicitis, etc. Ba-Follow thro’ was arranged to look atany obscure cause. Appendix was filled and it was in pelvicposition, long and slender. There was no tenderness ondirectly palpating it under screening. It showed two welldefined faecoliths. Rest of the small bowel was normal. Sub-acute appendicitis was therefore suspected and afterdeliberation with surgeons a diagnostic laparoscopy wasarranged where the appendix appeared inflamed and thiswas removed. Rest of the examination was normal. Therewas prompt recovery of his pain. Pathological specimenshowed 3 faecoliths and mild inflammatory changes.

Page 104: Bedtime gastroenterology

92 BEDTIME GASTROENTEROLOGY

CASE 47: A 12-YEAR-OLD WITH RECURRENTFEVER AND SPLENOMEGALY

A 12-year-old boy from a very poor family was seen becauseof generalised oedema.

He was not keeping well for over two years with recurrentfever with rigors and would take any opportunity to lie down.He ate normally. He did not consult any doctor previously.Over past few months he was noticed to be out of breath onminimal exertion and his ankles started swelling.

On examination he was very pale with generalisedoedema. No jaundice, clubbing, nodes were detected.Abdomen showed moderately enlarged spleen. Liver wasnot palpable. There were no visible veins on his abdomen.Cardiovascular system revealed hyperdynamic circulation.Other systems were normal. His FBC revealed Hb of4.7 gm with microcytic hypochromic picture and highreticulocyte count. Malarial parasites were not seen, liverfunction test showed mild rise in unconjugated bilirubin andlow albumin but normal globulin.

Q. What is your differential diagnosis? What otherinvestigations will you carry out? What is the line ofmanagement?

A. Preceding history of recurrent fever with rigors was verysuggestive of malaria and “malarial spleen” or tropical spleno-megaly was the obvious diagnosis. Many a times thoughportal hypertension of any cause presents with spleno-megaly or sometimes even myeloproliferative disorders couldpresent this way and these causes should always be keptin mind and excluded by proper tests, like oesophagoscopy

Page 105: Bedtime gastroenterology

93CASE 47

for varices, liver function tests, if required liver biopsy, bloodsmear, bone marrow, etc.

In this patient portal hypertension and myeloproliferativedisorders were excluded. His anaemia and hypoproteinaemiawere combination of haemolysis and malnutrition. Long-termchloroquine is required under close supervision.

Page 106: Bedtime gastroenterology

94 BEDTIME GASTROENTEROLOGY

CASE 48: A CASE OF RECURRENT JAUNDICEA 27-year-old male, non-smoker, not taking alcohol and notengaged in any homo/heterosexual activity came becauseof recurrent jaundice.

Four years ago he was diagnosed to have infectivehepatitis following a bout of fever, nausea, vomiting andanorexia. He made an uneventful recovery from this. Forsubsequent two years he had similar episodes exactly atthe same time of the year. His investigations are shownbelow.

FBC: Normal each year, Reticulocyte count < 2%, Peripheralsmear: normal, HBsAg: Negative, HBcAb: Negative, HCVAb: Negative, ANF: Negative, Anti-microsomal Ab: Negative,Liver biopsy: Focal necrosis. No evidence of chronic activehepatitis

No Alpha - 1 antitrypsin deficiency, No haemochroma-tosis, Ceruloplasmin: normal range, Liver function tests:Normal at other times.

Q. What further tests are needed? What could becausing recurrence?

A. This was very puzzling. As far as he could be tested, nocause was found and there are a large number of suchpatients and perhaps with advent of time more tests will beavailable which could pick up such undiagnosed cases.

Page 107: Bedtime gastroenterology

95CASE 49

CASE 49: ISOLATED RISE OF SGPTA 26-year-old male had his liver functions checked about7 months prior to present consultation for feeling off colour.As his SGPT was raised he was given rest. His SGPTremained elevated for next six months and hence the referral.He did not drink alcohol, consumed tobacco or used anydrugs/chemicals. He got married only eight months ago. Hismother (aged 48 years) and his sister (aged 27 years) bothhad hypothyroidism. His maternal aunt had a small patch ofLeucoderma. His grandmother had maturity onset diabetesmellitus. He denied any fever, weight loss, skin rashes, jointpain, photosensitivity, etc. His examination was normal. Hisinvestigations were as follows:

FBC, ESR: Normal, peripheral Smear: NAD, T3, T4 and TSH:Within normal range, HBsAg, HBcAb, HCV all negative,random blood sugar: 90 mg

Liver Sonography: Changes suggestive of mild fatty infil-tration. Spleen: Normal size, Kidneys and pancreas: normal,Chest X-ray: NAD.

Q. What further investigations would you carry out?

A. Autoantibodies like ANF, Anti-smooth muscle antibodies,S. Fe, TIBC, S. Ferritin, Alpha-1 Anti-trypsin levels, SerumCopper and Ceruloplasmin levels, S Albumin and globulinand if required liver biopsy.

This is a case of isolated rise of SGPT which is fairlycommon. This could be because of alcohol abuse, drugtoxicity especially some indigenous herbal drugs, chroniclow grade hepatitis, congestive cardiac failure, obesity,diabetes, Wilson’s disease or idiopathic. Recently there are

Page 108: Bedtime gastroenterology

96 BEDTIME GASTROENTEROLOGY

reports of non-alcoholic steato-hepatitis (NASH) which isassociated with fatty infiltration and thought to lead tocirrhosis in long run in some cases and this seems themost likely cause here in absence of any detectable causethough regular and prolonged follow-up is essential.Ursodeoxycholic acid is recently claimed to have good effectin fatty infiltration. If overweight, reducing weight is alsovery helpful in fatty infiltration and thereby SGPTnormalisation.

Page 109: Bedtime gastroenterology

97CASE 50

CASE 50: A MIDDLE AGE MAN WITH EARLY SATIETYA 42-year-old, otherwise fit, man presented with fullness ofstomach following meal. This was going on for a few months.He had cut down on his meal but with no help. He otherwisefelt well, systemic inquiry was normal. On examination therewas nothing to find except mild epigastric tenderness.

Q. What further investigations would you carry out?What is the likely diagnosis?

A. This types of presentations are always difficult to eva-luate. If there are no alarming features like anaemia or weightloss, one could give general dietary and lifestyle adviceand observe with symptomatic treatment like proton pumpinhibitors. If there is no improvement or if any alarmingsymptoms develop or if the patient is above 50 years ofage, then further investigations may be required.

Upper GI endoscopy and biopsy for microscopy and rapidurease test for H. pylori. Gastric motility study if availablemay be helpful.

This patient had severe gastritis with H. pylori. Hissymptoms would fall into Non-ulcer Dyspepsia (NUD)category. Gastric dysmotility with Helicobacter pylori gas-tritis is more frequently recognised now. Cure of Helicobacterpylori may not always lead to improvement in symptoms.

Page 110: Bedtime gastroenterology

98 BEDTIME GASTROENTEROLOGY

CASE 51: A HOUSEWIFE WITH ACUTEHEPATITIS-B AND CHANGES OF

PORTAL HYPERTENSIONA 35-year-old housewife was admitted with haematemesis.She was well till about 6 weeks ago when she had developedfever, nausea and bodyache. She was found to be jaun-diced. She was positive for HBsAg (ELISA), liver functionsshowed hepatitic picture. The jaundice progressivelydeepened over next 6 weeks with development of mildascites, she had small haematemesis day before admission.There was no past history of any illness. She did not takeany drugs or alcohol. On examination deep jaundice andascites were confirmed. There were no stigmata of chronicliver disease. Her upper GI endoscopy revealed Grade I-IIoesophageal varices and congestive gastropathy. Her liverfunction tests were as follows S. bil 42 mg, SGPT 747 iu,Alk-Phos 316 iu, S. alb 3.1 g, S. Glob 3.4, Prothrombinetime 28/14 sec, HBsAg (EL ISA) reactive, HBcAb (IgM)reactive, HCV Antibodies Negative, ultrasound : coarseecho texture of liver, dilated portal vein. Spleen wasenlarged.

Q 1. Would you want to do any other test?Q 2. How would you manage her case?

A 1. This patient was believed to be having acute hepatitisB initially. Later on there was evidence of portal hypertensionand probably of cirrhosis of liver. Therefore, it was thoughtthat she could have chronic HBV infection. But HBcAb IgMwas reactive thus suggesting acute hepatitis B. Moreover,hepatitis serology suggested that HCV was negative and

Page 111: Bedtime gastroenterology

99CASE 51

therefore cirrhosis could be because of some other reasons.Wilson’s disease was suspected and confirmed by presenceof Kayser-Fleischer ring, very low Ceruloplasmin levels andvery high urinary copper.

A 2. Apart from supportive treatment, patient was put ond-penicillamine with prompt response and good recovery ofliver function test. This patient became pregnant some 2years later but unfortunately as is often the case abortedaround 12 weeks.

Page 112: Bedtime gastroenterology

100 BEDTIME GASTROENTEROLOGY

CASE 52: A SUCCESSFUL BROKERWITH ABDOMINAL PAIN

A 40-year-old man was seen because of epigastric pain, forabout 15 days. This pain was dull type with occasionalexacerbation. About a fortnight ago he was seen by anotherpractitioner where upper GI endoscopy was performed butwas essentially normal. He was treated with Ranitidine. Hispain continued. There was no definite relationship with foodor passing stool. Systemic inquiry was otherwise normal.Ranitidine was continued. Early next morning I was calledfor home visit because of severe abdominal pain, whichwas central and colicky type. There was no nausea orvomiting. He had not passed stool or flatus for 24 hrs.Abdomen was soft and peristalses were present. Systemswere normal. He was given intravenous antispasmodic andwas admitted for observation. Repeat endoscopy wasessentially normal. CBC showed Hb 8.2 gm, microcyticand hypochromic type, TC-11000, S.Bilirubin 2.08, (direct30%, indirect 70%) SGPT: 34 iu, Alk-phos 136 iu, S.amylase 201 iu, renal function tests were within normalrange. Abdominal ultrasound was normal and plain X-ray abdomen revealed non-specific gas pattern, noperforation was detected. There was symptomaticimprovement of a while but the pain recurred. Repeatamylase was within normal limits. Abdomen remained soft,rectal examination revealed faces of normal colour.

Further inquiry revealed that he did not have any majorillness in the past. He felt low vitality about 1½ years agodue to very demanding work for which he took someindigenous tonic till about a fortnight ago. Lately his appetite

Page 113: Bedtime gastroenterology

101CASE 52

had reduced with weight loss of about 5 kg over past fewmonths. Thus, he attributed to the pressure of work followingdeath of his father 2 months ago. Barium follow thro’examination was ordered suspecting abdominal tuberculosis.Barium column abruptly stopped in mid small bowel. 24 hrslater, barium had failed to move any further.

Abdomen remained soft, and there was no passage offlatus or faeces. He was kept nil by mouth and IV fluidswere started. All anti-cholinergic antispasmodics werestopped. Repeat X-ray at 48 hrs did not show any change.However, at 72 hrs barium had moved into large bowel. Patientstarted passing faeces and flatus that evening and wasdischarged without patient follow-up. His pain persisted inmilder form. Further investigations revealed : Hb 8.0 gm/dl,MCV : 82 fl, MCHC: 33, MCH: 24, Platelets: adequate,Smear showed basophilic stippling, Retic Count 8%,S.Bilirubin 3.17 mg, Direct 20%, Indirect 80%, S. amylase85 iu/L, glucose 106 mg. Tests for malaria and Coombs’test and ANF were negative.

Q. How will you manage this patient now?

A. This patient had come for initial consultation on Saturdayat 8.00 pm. His initial investigations were carried out onSunday morning. This had important bearing because hisreports took time to come and came one by one. Initial mildindirect bilirubinaemia was considered insignificant. It waswhen his FBC and liver functions were repeated afterdischarge that the penny dropped. This experiencedhaematologist (different from the first one) immediatelypicked up basophilic stippling which suggested leadpoisoning. Many indigenous or herbal medicines are

Page 114: Bedtime gastroenterology

102 BEDTIME GASTROENTEROLOGY

containing lead or other toxic substances. He had takenthese medicines for 1½ years. Serum lead and urinary leadexcretion correlated with lead intoxication. He was treatedwith D-penicillamine with very good results. He regained hisweight and haemolysis stopped with normalisation ofhaemoglobin. 1½ years he came again now with self-madediagnosis of repeat lead poisoning. He had started takingremaining tablets from last time.

Page 115: Bedtime gastroenterology

103CASE 53

CASE 53: HAEMOLYTIC ANAEMIAAND GALLSTONES

An 11-year-old boy was seen by haematologist because ofsplenomegaly and ankle oedema. His investigationssuggested mild haemolytic anaemia and gallstones. Detailedinvestigation for cause of haemolysis was negative. Bariumswallow did not show any varices. Liver function tests wereshowing mild hepatitic picture with low albumin at 3.1 gm/dl.

A 16-year-old girl was admitted for evaluation of spleno-megaly. Ultrasound scan showed gallstones. Liver functiontests were normal.

Q. What is the link between these two cases? Whatis the sequence of investigations?

A. Gallstones in a young patient with splenomegaly shouldalways mean haemolysis. All appropriate tests for haemo-lysis starting with a good peripheral smear are mandatory.Hereditary spherocytosis, sickle cell disease and variousother conditions like autoimmune haemolytic anaemia shouldall be kept in mind. In the first case all routine tests forhaemolysis were negative. However, hepatitic picture withlow albumin alerted us to the possibility of Wilson’s disease.In Wilson’s disease there is liver involvement and toxic effectof copper produces haemolysis.This was confirmed withslit-lamp examination of eye and Ceruloplasmin studies. Thischild responded very well to D-penicillamine. The secondcase had purely haemolytic jaundice and her haematologicalinvestigations were diagnostic of hereditary spherocytosis.She had splenectomy and she got better.

Page 116: Bedtime gastroenterology

104 BEDTIME GASTROENTEROLOGY

CASE 54: A CASE OF CHEST PAIN REQUIRINGSPECIAL MANOEUVRE TO SWALLOW

A 45-year-old lady was referred from a cardiologist forevaluation of chest pain. For few years she would haveintermittent chest pain, which was like choking sensation.There was no relationship to exertion, food, posture, etc.Various cardiological investigations including ECG andstress test and echocardiography were normal. Her physicalexamination was entirely normal. Detailed inquiry revealedthat she experienced difficulty in swallowing and required aspecial manoeuvre which she demonstrated as swallowinga bolus of air, then closing the pharynx and forcing the bolusand, with it, food down the oesophagus. She also hadnocturnal coughing and very foul smell from her mouth andeructation.

Q. How will you investigate her? What is your clinicalimpression?

A. The manoeuvre this lady showed is what I call “achalasiamanoeuvre”. This is done to open up gastro-oesophagealsphincter forcibly by building up the pressure. Clinically thispatient must have achalasia cardia. Investigations shouldinclude barium swallow and oesophagoscopy withmanometry if available which is considered to be the goldstandard. Here there will be basal hypertension and failureof relaxation of gastro-oesophageal sphincter on swallowing.Endosonography may show thickened muscular ring.Endoscopy may miss achalasia as the sphincter may beeasily negotiated with application of little pressure, when

Page 117: Bedtime gastroenterology

105CASE 54

the endoscope passes through the narrowed segment andenters stomach characteristic “give” is felt.

Balloon dilation gives 85-90% success rate with about2-4% perforation rate. The first time non-responders mayrespond to larger balloon next time round. It is much quickerand generally preferred mode of treatment. Heller’s myotomyis another way of treatment. Botulin toxin which paralysesmuscles is a recent introduction but the effect is shortlasting. Calcium channel blockers like Nifedipine may beuseful but as a short-term treatment only. Some achalasiasare associated with chest pain, due to muscle contractionand are called “vigorous” achalasia. Sometimes there is freegastro-oesophageal reflux following very enthusiasticdilatation but this is easily controlled with proton pumpinhibitors and posture and diet.

Page 118: Bedtime gastroenterology

106 BEDTIME GASTROENTEROLOGY

CASE 55: RECURRENT DIMNESS OFVISION AND INTESTINAL PERFORATION

A 35-year-old lady was admitted in emergency for abdominalpain and swelling. She was unwell for past few years withvague abdominal pain, intermittent fever and recurrentdimness of vision. She was seen at regular interval by anophthalmologist who suspected tubercular infection of theeyes and had prescribed anti-tubercular treatment for 3 fullcourses without long-lasting improvement. Together with anti-tuberculous drugs she was prescribed steroid eye drops.

On examination she looked unwell, febrile and hadabdominal distension with guarding. Her pulse was 140,BP 100/60, temperature 101 F, Hb 9.1 gm, normochromic-normocytic picture, ESR: 160 mm/Hr. Urea 56 mg,Creatinine 1.1, urine: NAD, X-ray abdomen showed gasunder diaphragm. Laparotomy was carried out whichshowed ischaemic looking small bowel with multiple smallperforations and multiple small ulcers on mucosal side.Histology revealed occasional non-caseating granuloma.No AFB was seen.

Q. What do you think is going on? What otherinvestigations would you like to do?

A. This patient had a long history of illness with fever andintermittent dimness of vision. She was treated with anti-tuberculous drugs because of fever and high ESR at thattime. In tropical countries, it is a common practice to treatvarious unexplained illnesses with anti-tuberculous drugs.At times there are genuine indications for such a practice,but more often either lack of investigative facility or

Page 119: Bedtime gastroenterology

107CASE 55

application of sound medical knowledge that leads to suchapproach. This lady was treated three times with anti-tuberculous drugs!!! When we saw her and went through herrecord, it appeared that she had vasculitis of some kind.Her resected intestine showed picture which favouredCrohn’s disease. Antinuclear factor was weakly positive.Vasculitis involving eye is not well known however it doesoccur. She was treated with Prednisolone which was taperedoff gradually with good response.

Page 120: Bedtime gastroenterology

108 BEDTIME GASTROENTEROLOGY

CASE 56: A CASE OF HEPATICCOMA FOLLOWING DELIVERY

A 26-year-old lady was admitted with hepatitis and coma.She was pregnant with second child. She had high bloodpressure, oedema of leg and proteinuria during later part ofher pregnancy and was diagnosed to have toxaemia ofpregnancy. Her first pregnancy 5 years previously was alsoassociated with mild toxaemia, but the outcome wasuneventful. She had her baby delivered by caesarean sectionone day prior to her becoming ill. She remained well forabout 20 hours after the operation when she was detectedto have jaundice. She became confused and comatose. Shewas oliguric and her blood pressure was 260/150 mm Hg.On examination she was afebrile, central nervous systemdid not reveal any focal localising sign, and both planterswere extensor. There was gross ankle oedema. Liver andspleen were not palpable. Soon after admission she hadgrand mal type convulsions.

Her investigations were as follows:Hb 10.0; WCC: 10,200; Platelets: 10,000 retics; 3%S. bilirubin: 10 mg; ALT (SGPT) 850 iu ; Alk. Phos: 120 iu; PT: 16/15 sec ; APTT: 36/37 Sec ; S. Fibrinogen: 4 ; FDPelevated significantly.Urea: 35 mg%; Creatinine: 1.9 mg%.

She had repeated convulsions and de-cerebrate posturingand hence she was sedated, given magnesium sulphateand hyper-ventilated to wash off CO

2 and decrease intra-

cranial pressure. Over next several days she requiredplatelet rich plasma, haemodialysis and total parenteralnutrition for paralytic ileus.

Page 121: Bedtime gastroenterology

109CASE 56

Q. What is your working diagnosis? The underlyingcause?

A. In pregnancy liver could be affected in many waysincluding pregnancy unrelated causes. Various trimestershave their peculiar diseases and in last trimester toxaemiaof pregnancy is an important pregnancy related cause. Thereseems to be a spectrum of overlapping liver functionabnormalities, starting at one end with elevation of SGPTto the other extreme like in this case which is otherwiseknown as “HELLP” syndrome suggesting Haemolysis,Elevated Liver enzymes, Low Platelets. Occasionally thereis disseminated intravascular coagulation as well. There maybe associated multi-organ failure as in this case. This patientrequired almost a month before she was well enough to gohome. A year later she came with acute hepatitis B fromwhich also she recovered unharmed.

Page 122: Bedtime gastroenterology

110 BEDTIME GASTROENTEROLOGY

CASE 57: A CASE OF MALARIA WITHJAUNDICE AND MULTIORGAN FAILURE

A 23-year-old male was admitted with increasing jaundice,abdominal pain, oliguria and haematuria. He was well beforea week or so when he had fever with rigors and severeheadache. Blood taken at that time revealed vivax malariaand he was treated with chloroquine. For a day or so he feltbetter and then again started having fever and headache.The fever was without rigors now and ranged from 102-104 F. Over next few days he developed abdominal painand then noticed decreasing urine output and urine was redin colour. His appetite was preserved and there wasoccasional nausea. He was treated now with anti-bioticstogether with chloroquine. His bilirubin jumped from2 mg on day 3 to 15 mg on day 7 with SGPT remainingaround 100 iu Hb : 4.5, WCC : 9000, platelets :5000,reticulocytes: 14%, Prothrombine time was 17/14 sec,Urea 42 mg, Urine showed plenty of RBCs.

On examination he appeared unwell and febrile. Liverand spleen were not palpable. CNS was normal Pulse andBlood pressure was normal.

Q. What would you do now? What is your workingdiagnosis?

A. When we had a good look at his history and blood resultsit was apparent that in all probability he had falciparummalaria with hepatopathy and renal involvement. Hisplatelets were very low and this would account for hisabdominal pain, supposedly due to intramural bleeding inthe intestines as in Henoch-Schönlein purpura and

Page 123: Bedtime gastroenterology

111CASE 57

haematuria. Probably Vivax malaria was a misdiagnosis.Blood was taken for F antigen for falciparum malaria andthick smear was also made. Both confirmed falciparummalaria grade III. Platelets were 5,000 only and plateletswere given to tie over the crisis. He was treated with Quinineand Doxycycline. His course was stormy in the sense thatfrom oliguria he developed anuria requiring haemodialysisas both his creatinine and potassium were high. We closelymonitored him because if there was any further deteriorationin his general condition he would have required exchangetransfusion. He was also transfused with packed cells tobring his haemoglobin up. Fortunately on daily monitoringhis parasitaemia promptly decreased and at no time therewas any CNS involvement or respiratory distress. He wentinto high output renal failure on recovery but this did notrequire any further intervention. Over the years we havelearnt that any patient with predominant fever and jaundice,especially when conjugated bilirubin is predominant andSGPT is less than 3 times normal, where there isunexplained deterioration or low platelet count suspectfalciparum malaria and even treat it empirically if the patientis not responding or the clinical picture does not fit. Newercompounds like Artemether and Artiether are very effectiveand very safe even if the patient has viral hepatitis.

Page 124: Bedtime gastroenterology

112 BEDTIME GASTROENTEROLOGY

CASE 58: A DEVOTED LADY WHODEVELOPED DIARRHOEA AND WEIGHT

LOSS AFTER HER FATHER’S DEATHA 54-year-old lady was seen for abdominal pain anddiarrhoea. She was previously fit till about 3 months agosince when she had started this problem. Her abdominalpain was central in nature and persistent. She had loosefoul smelling stool 6-7 times a day. There was associatedweight loss of about 12 kg in three months. She had nofever. She had a course of anti-amoebic and a course ofCiprofloxacin but her diarrhoea continued. Her physicianasked for Barium follow-through’ which suggested thickenedsmall bowel folds with mild dilatation and some flocculationsuggestive of malabsorption. On basis of this report shewas put on anti-tuberculous treatment but there was nobenefit at one month and hence the referral.

Fout months ago, her father had died and she went toNarmada river banks for last rights. She said that all herproblems started since then onwards. On examination therewas nothing to find.

Hb: 9.4, with microcytic hypochromic picture, WCC:12,000 with 32% eosinophils, ESR: 43 mm / hr

Urea and Electrolytes normal, LFTs were normal. Albumin3.1 and Globulin 2.2. Stool revealed strongyloidosis sterco-ralis. Barium films were reviewed and the report seemed OK.

Q. What is your diagnosis? How would you furtherinvestigate and manage this case?

A. This lady was so insistent that she had acquired thisillness following her visit to the river banks that we had to

Page 125: Bedtime gastroenterology

113CASE 58

take her seriously and it was not too difficult to believe herthat as her eosinophil count was very high. She was notresponding to anti-tuberculous treatment and barium showedthickened folds. Repeated stool tests were carried out andin one sample stool showed strongyloids. All in all wesuspected eosinophilic gastroenteritis. A small bowel biopsywas taken from 4th part of duodenum and showed massiveinfiltration with eosinophils and even S.stercoralis. She wastreated with Albendazole 400 mg twice a day for three days.Sometimes when you hit the nail on its head the responseis near dramatic. Immediately after the first dose she gotrelief from her pain and diarrhoea. Thiabendazole is analternative. In chronic case Ivermectin could also be tried.Eosinophilic gastroenteritis is either patchy or diffuse. Incase it is diffuse there may be associated nausea, vomiting,and abdominal pain. Weight loss could be marked.Sometimes when there is serosal involvement there isascites only, which is rich in eosinophils. Mostly eosinophilicgastritis is idiopathic and responds well to steroids.

Page 126: Bedtime gastroenterology

114 BEDTIME GASTROENTEROLOGY

CASE 59: AN ALCOHOLIC WITH ASCITESA 42-year-old male was referred for ascites. He was unwellfor some three months with anorexia, abdominal discomfortand feverish feeling. He was seen by his physician and wasprescribed antacids and antispasmodics with some relief inhis abdominal discomfort however his anorexia persisted.A fortnight later he noticed abdominal distension for whicha sonography was ordered which showed inhomogenousecho texture of liver, splenomegaly and gross ascites. Nointernal septation was seen. His Hb was 10.4 withmicrocytic hypochromic picture. His bilirubin was 1.9mg, SGPT 94 iu, Albumin 3.0 gm, globulin3.9 gm. As hewas alcoholic he was diagnosed to have alcoholic cirrhosisand was advised to cut down on salt and fluids and wasprescribed anti-diuretics. He was referred for second opinion.

He was neo-rich, flambuoyant type of guy who wasdrinking alcohol for last 20 years but lately he had stoppeddrinking anything but alcohol starting from morning andcontinuing all throughout the day. He was surrounded byfriends who were no less alcoholic than him. His wife whowas from a very conservative background was forced tojoin in. There was no previous illness.

On examination he had hepato-splenomegaly and a fewspiders on his back. There was moderate ascites. No obviousjaundice. He appeared very scared but fully alert andoriented. Above mentioned reports were confirmed. UpperGI endoscopy revealed grade - I esophageal varices.HBsAg and HCV were non-reactive. Ascitic fluid was:protein 5.4 gm, sugar 67 mg, Cells 900 mainlylymphocytes, no malignant cells were seen.

Page 127: Bedtime gastroenterology

115CASE 59

Q. What is our diagnosis? How will you confirm it?

A. Obviously his ascitic fluid shows exudative pattern verymuch suggestive of tuberculosis. We did his chestX-ray which was normal. His ascitic fluid was sent forpolymerase chain reaction (PCR) testing for AFB and thistest was positive for tuberculous bacilli. All these findingswere enough to suggest that this alcoholic had both portalhypertension and tuberculous ascites. This is generallyall that one requires as tuberculosis in alcoholics is extremelywell known. To be absolutely certain we carried out adiagnostic laparoscopy which showed shrunken liver withmicronodules and both liver surface and peritoneum werecovered with tubercles which was confirmed on liver biopsy.He was treated with anti-tuberculous drugs and diureticswith excellent improvement. This case emphasizes thatdiagnostic ascitic tapping is a must, however, obvious isthe cause.

Page 128: Bedtime gastroenterology

116 BEDTIME GASTROENTEROLOGY

CASE 60: A YOUNG BOY WITH ASCITESA 10-year-old boy was admitted with abdominal distension.He was not well for last couple of years complaining oftiredness and shortness of breath on exertion and pain inright upper quadrant. Parents said he would come back fromschool and straight away go to bed feeling very exhausted,not eating much complaining all the time fullness. He sawhis family physician and then a pediatrician but nothing muchwas found and he was treated symptomatically. About amonth ago he started having abdominal discomfort and thenswelling for which he saw a paediatrician who suspectedascites and this was confirmed on sonography which showedenlarged liver and normal spleen. He was suspected to havetuberculosis and treated accordingly without anyimprovement.

On examination he was listless and had tense ascites.There was no anaemia, but he had peripheral cyanosis.Jugular venous pressure was raised significantly and asystolic murmur was heard which was thought to be due topulmonary stenosis. Liver was enlarged and palpable withdifficulty because of gross ascites. There was no evidenceof chronic liver disease. His CBC, urea and electrolyteswere normal. Bilirubin 1.2 mg, SGPT 56 iu, Ascitic fluidshowed protein 5.6 gm, sugar 100 mg, cells 34,mainlylymphocytes.

Q. How will you proceed now? Diagnosis?

A. This was a difficult case. He was referred to gastro-enterology unit for ascites and hence picking up cardiaclesion with confidence was difficult. As we see so manygross ascites with cirrhosis that he also thought to have

Page 129: Bedtime gastroenterology

117CASE 60

cirrhosis however his ascitic fluid protein was very high andthen the bell started ringing. A personally supervisedsonogram was done which showed dilated hepatic veinsand there was no phasic contraction of inferior vena cava.We therefore suspected right sided heart failure and did acolour Doppler which suggested severe pulmonary stenosiswith gross right sided heart failure. High protein in asciticfluid is because of severe back pressure resulting in leakageof protein from liver surface. Typically cells are minimal asopposed to exudative ascites where there are plenty of cells.He was referred to cardiologist for further management.

Page 130: Bedtime gastroenterology

118 BEDTIME GASTROENTEROLOGY

CASE 61: A 54-YEAR-OLD POLICE CONSTABLEWITH NON-RESOLVING ACUTE PANCREATITIS

A 54-year-old police constable was seen because of pain inleft side of abdomen. He was taking alcohol 3-4 times aweek. Previously he did complain of intermittent “gas andacidity” and would take antacids and Ranitidine for that. Aboutfour weeks ago he had moderately severe central abdominalpain and this was diagnosed as acute pancreatitis as bothhis amylase and lipase were very high. His sonographyrevealed hypoechoeic pancreas. Gallbladder, CBD and Liverwere normal. He was treated conservatively. His abdomenexamination showed tenderness on left side near splenicflexure area. This tenderness persisted together with mildabdominal pain and hence he had repeat sonography whichrevealed a plegmon near the tail of pancreas. This persistedtogether with pain and tenderness and hence he had a fineneedle aspiration under CT guidance which showedinflammatory cells only. CEA and CA19-9 were marginallyelevated. A month later his sonography remained unchangedand hence he consulted another doctor who carried out anERCP which was essentially normal and there was noconnection between the mass and pancreatic duct. Hiscondition remained same but now he started loosing weight.A repeat ERCP was carried out and fluid from pancreaticduct was sampled for malignant cells but this was normal.He disappeared from follow-up.

Q. How you would have approached this case? Whatis your diagnosis?

A. Pancreatic masses are sometimes notoriously difficultto evaluate and this case is a classic example of that. Some

Page 131: Bedtime gastroenterology

119CASE 61

times even more investigations like Endoscopic ultrasono-graphy with targeted biopsy, MRI, etc. are also thrown in forgood measure. But after all these investigations pancreaticmasses can remain undiagnosed. Current understanding isthat for pancreatic masses which are not easily diagnosedit may be more appropriate to leave it to surgeon wherediagnosis, staging and sometimes treatment all could beachieved at a single sitting. This patient (we learnt) ultimatelyhad a laparotomy which to our surprise revealed pancreaticcarcinoma with local metastasis which was missed all along.With hind sight if we had not gone on and on probably hemight be at a resectable stage.

Page 132: Bedtime gastroenterology

120 BEDTIME GASTROENTEROLOGY

CASE 62: A MONTH OLD CHILD WITH JAUNDICEA month old child was referred for jaundice. He was firstborn, full term normal delivery with birth weight of 2.5 kg.Immediate postnatal examination by an experienced pae-diatrician was unremarkable. On 4th day the child developedjaundice which was thought to be a “physiological” one withbilirubin of 6 mg (70% conjugated), SGPT 154 iu, Alk.Phos. 600 iu and prothrombine time of 19/15 sec. Clinicalexamination was unremarkable with neither spleen nor liverpalpable and no fever or any other evidence of infection. Aclose eye was kept on his bilirubin which slowly crept upinstead of settling down but the child appeared well andcontinued with his feed. At about 3 weeks from birth hisbilirubin went up to 12 mg (70% conjugated) and SGPT134 iu, Alk. Phos. 796 iu, and hence the referral.

On examination the child appeared well but deeplyjaundiced. Liver and spleen were not palpable. Systemicexamination was normal. His CBC, RFT, electrolytes andblood sugar were normal. An ultrasound was reported asnormal although the gallbladder was not visualisedwhich was thought to be due to” non-fasting state of thechild”. TORCH titre revealed raised IgG forcytomegalovirus but rest was normal.

Q. How will you proceed now? What is your likelydiagnosis?

A. “Eyes do not see what the brain does not know”. Anultrasound was reported as normal although the gallbladderwas not seen in its normal place. These are majorcommunication gaps which can affect patient’s life. Obviously

Page 133: Bedtime gastroenterology

121CASE 62

this child had conjugated hyperbilirubinaemia the so called“neonatal hepatitis syndrome” which has many causes.Sonologist must know that he is looking for bile duct anatomyand gallbladder apart from liver texture and spleen and soon. A repeat sonogram was ordered which also failed tolocate gallbladder after prolonged fasting. A magneticresonant cholangiopancreatography was also obtained atthe same time which showed normal intrahepatic biliaryradicals but absent gallbladder and bile duct was also notvisualised. Now the suspicion was firmly on extrahepaticbiliary atresia. The patient was put on 5 mg / kg ofphenobarbitone for 5 days and a HIDA scan was obtainedwhich showed failure of excretion into the intestine. A liverbiopsy was very suggestive with bile ductular proliferation,bile plugs and some giant cells. At operation he was foundto have type III extrahepatic bile duct atresia withinvolvement of total extrahepatic biliary tree. Incidence ofcytomegalovirus is high in patients with extrahepatic bileduct atresia. Even in this patient there was significant risein IgG antibodies against cytomegalovirus confusing theissue. As there was no rise in IgM titres againstcytomegalovirus, we thought against treating her withGancyclovir. A Kasai portoenterostomy was done at around8 weeks.

Page 134: Bedtime gastroenterology

122 BEDTIME GASTROENTEROLOGY

CASE 63: A CASE OF RECURRENT PANCREATITISA 30-year-old male was admitted with history of recurrentattacks of acute pancreatitis for past five years. He was fitand well before that. One morning following a large dinnerand alcoholic binge he developed his first attack ofpancreatitis. This was thought to be due to alcohol and hewas advised to stop alcohol which he promptly followed.Two years later he had his second attack of pancreatitisalthough he was off alcohol. Ultrasonography during boththese attacks was normal so were fasting calcium and lipids.There was mild rise in his bilirubin and SGPT during thesecond attack. He had one more attack few months prior tothis consultation when he had normal sonography but MRIshowed suspected crystals in gallbladder. His liver functiontests were showing mild rise in bilirubin and SGPT and Alk.Phos. An ERCP showed suspected narrowing in the headand a small pancreatic sphincterotomy was done. He cameto consult if anything more could be done to prevent anyfurther attack of pancreatitis.

Q. What advice you would give him now?

A. Initially this patient was suspected to have alcoholichepatitis but even after stopping alcohol he continued tohave attacks of pancreatitis, All other treatable causes likehypercalcaemia, hyperlipidaemia were negative. Duringsecond and third attack he had slightly elevated LFTs whichcould be because of passage of a stone or pancreatic headoedema. We need to look hard for gallstones whichsometimes could be missed by all conventional tests. Recentintroduction of endosonography is very useful in such cases.

Page 135: Bedtime gastroenterology

123CASE 63

During this examination multiple tiny gallstones were clearlyidentified. Pancreatic duct was also very well identified andfound to be normal. He has undergone laparoscopiccholecystectomy. At operation GB was found to be fibrosedand adherent suggesting recurrent attacks of cholecystitis.

Page 136: Bedtime gastroenterology

124 BEDTIME GASTROENTEROLOGY

CASE 64: A CASE OF PAINFUL DYSPHAGIAAND WEIGHT LOSS

A 34-year-old housewife presented with difficulty inswallowing which was very painful. This was progressivelyincreasing for past three months. There was associatedweight loss of about 7 kg in past three months which wasattributed to difficulty in swallowing. Initially the generalpractitioner thought she had acid peptic disease and treatedher with ranitidine with some improvement but this was shortlasting upon which she was treated with lanzoprazole andagain there was transient improvement. A barium swallowdone was reported to be normal. On recurrence of herproblems she was referred for further investigations. Shewas staying in Sangli since her marriage some 15 yearsago. Her previous health was unremarkable. She had twochildren both delivered by Caesarean section, the secondone requiring blood transfusion postoperatively as herhaemoglobin was 8 gm. Both her children were well. Shehad no habits and there was no family history of note. Onexamination she appeared unwell and frightened. There wasevidence of recent weight loss. Systemic examination wasnormal. Hb: 10.2 gm, fasting and PPBS 73 mg and 98 mgrespectively. An Upper GI endoscopy was carried out.

Q. What are the likely findings and further testsrequired?

A: Painful dysphagia, odynophagia with weight loss of longduration must arouse suspicion about fungal infection ofthe oesophagus which this patient had. Other causes of

Page 137: Bedtime gastroenterology

125CASE 64

Odynophagia are herpes and cytomegalo infection. HIVtesting and upper GI endoscopy are two importantinvestigations required here. This patient had positive HIVtesting and florid oesophageal candidiasis.

Page 138: Bedtime gastroenterology

126 BEDTIME GASTROENTEROLOGY

CASE 65: A CASE OF SUDDENONSET OF ABDOMINAL PAIN

A 42-year-old patient was referred because of sudden onsetof severe abdominal pain which got progressively worse.He was known insulin dependent diabetic for many years.About 3 months prior to these complaints he had fever ofunknown origin and this was diagnosed as enteric feveralthough single Widal titre was not significantly raised. Heremained well thereafter till this present complaint whichstarted about two days before in the morning. The pain wasdiffuse but mild initially. This was treated symptomatically.Over next day or so it got worse with noticeable swelling ofhis abdomen and small amount of yellow vomiting. Hedeveloped high grade fever and breathlessness and hencethe referral.

He drank alcohol heavily till about 6 months before.On examination he was febrile, tachypnoeic and had

severe tachycardia of 140/ min. His abdomen was diffuselytender and had moderate ascites. There was no jaundice.Lungs were clear, blood pressure 100/ 60.

Hb: 10 gm., WCC: 17000, with shift to left, Bilirubin :1.2 mg, SGPT 78 iu, Alk. Phos 100 iu, amylase 143 iu,lipase 200, urea 45, creatinine 1.2, blood sugar 436 mg,Sonography showed fatty liver grade II, gallstones but biliaryradicals appeared normal, pancreas was not well seen butthere was doubtful bulkiness. There was mild ascites onday 1 which had increased to moderate amount on day 2.Plain X-ray abdomen was unremarkable.

Page 139: Bedtime gastroenterology

127CASE 65

Q. What is your diagnosis? Further tests?

A. Initially this appeared to be a case of acute pancreatitisbut amylase and lipase were not significantly raised and onsonography there was no convincing evidence of pancreatitis.Ascites was filling very rapidly and this patient obviouslywas going into septicaemia. Diagnostic ascitic tap wasconsidered with suspicion of spontaneous bacterial peritonitisin mind. This showed turbid fluid with cell count of 11,000and 90% polys. Ascitic fluid amylase was normal. pH wasacidic. Thus, clinically and laboratory-wise it was a case ofspontaneous bacterial peritonitis. He was endoscopedlooking for varices and had grade III varices suggestingportal hypertension mostly from alcoholic liver disease.Spontaneous bacterial peritonitis is well known entity incirrhosis and when there is associated diabetes need veryenergetic care. This patient responded well to the treatmentand doing well after 6 months.

Page 140: Bedtime gastroenterology

128 BEDTIME GASTROENTEROLOGY

CASE 66: A CASE OF RIGHT SIDED PLEURALEFFUSION WITH FEVER AND COUGH

A 24-year-old man presented incidentally with cough, feverand malaise. He was seen 3 years ago with alcoholichepatitis to which he responded well but was lost to follow-up. During these 3 years he had continued to drink. For aweek he felt listless with fever, cough and malaise. This didnot respond to symptomatic treatment and hence the visit.On examination he appeared unwell and jaundiced. Therewas obvious right sided pleural effusion of moderate amount.Liver was not palpable. There was no ascites or ankleoedema.

Hb: 10.4, WCC : 2.4, Plt : 87,000, bilirubin : 3.5, SGPT:89, Albumin : 3.0. Chest X-ray confirmed right sidedpleural effusion, lungs were clear. Pleural fluid protein0.3 gm, cells 20 mainly lymphocytes, no malignant cellswere seen, no AFB seen.

Q. What is your working diagnosis? Any further tests?

A. We thought about tuberculosis as this is common inalcoholics and even otherwise also. Pleural fluid howeverwas transudate. We have seen cases with ascites and pleuraleffusion in cirrhotics and biochemically this fitted the bill.His sonography showed changes of cirrhosis and portalhypertension. He was treated with diuretics and antibioticsand responded immediately. Three months on he has norecurrence of pleural effusion. Thus, isolated pleural effusioncould sometimes be an uncommon manifestation ofdecompensated liver disease.

Page 141: Bedtime gastroenterology

129CASE 67

CASE 67: YOUNG GIRL WITHRECURRENT VOMITING

A 10-year-old girl was referred for recurrent vomiting. Shewas well till about 3 months ago when she had low gradefever, cough, and anorexia and weight loss. She had chestX-ray which suggested right upper lobe infiltrative lesionsuggestive of tuberculosis and hence she was treated withfour anti-tubercular treatment and she felt better. On a routineLFT check after 15 days, elevated SGPT was found whichrose again after a week and hence this was stopped. Shefelt well and hence she was not started again on anti-tuberculous drug. 2-3 weeks later she complained of nausea,abdominal pain and vomiting which eventually got reallybad. Ultrasonography suggested a few enlarged mesentericnodes and X-ray abdomen was normal. Upper GI endoscopywas normal. FNAC from lymph node was planned but ontable sonologist could not find any nodes!!! She was treatedsymptomatically but continued to have intermittent vomiting.She was treated as Acid peptic disease. Initially she feltwell but then again her complaints started.

Q. What is your thinking? What further tests youwould do?

A. Sometimes your patient pays the penalty for yourmistakes. When you stop applying logic and commonsenseyou may make all sorts of mistake. We forgot that this childhad a reasonably certain pulmonary tuberculosis. We alsoforgot that sonography is observer dependent and that nodesonce found could be missed by other sonologist. With pasthistory like hers, intestinal tuberculosis was very likely and

Page 142: Bedtime gastroenterology

130 BEDTIME GASTROENTEROLOGY

barium would have been more appropriate instead of FNACof the lymph node. She had a barium follow-up examinationwhich showed dilated loops of small intestine. We were mostprobably dealing with a case of intestinal tuberculosis. Shewas again treated with anti-tubercular treatment givenaccording to her weight. No SGPT elevation was detectedfor two months. Now her vomiting has almost stopped andshe is putting on weight as well.

Page 143: Bedtime gastroenterology

131CASE 68

CASE 68: ACUTE PANCREATITISWITH RAPIDLY FILLING PSEUDOCYST

A 23-year-old boy was seen for rapidly filling pseudocyst.Four months ago he had sudden onset of abdominal painfor which he was investigated at a nearby town hospital andhe was diagnosed to have an acute attack of pancreatitis.He was treated along the conventional line with pain killersand antibiotics and symptomatic treatment and appearedto be settling down. Investigations at that time showed normalcalcium and triglycerides. He was not taking any drugs oralcohol. Ultrasound showed normal gallbladder and commonbile duct and hence he was diagnosed to have idiopathicacute pancreatitis. During follow-up he was found to havepancreatic pseudocyst in the area of tail of pancreas whichon further follow-up with sonography showed increasing sizeover next 6-8 weeks and hence he was referred to a surgeonfor further management. A contrast enhanced CT scanshowed oedematous pancreas especially in the region ofbody and tail with some necrosis and a large pseudocystwith internal echoes. Main pancreatic duct was visualisedup to mid body only. This was pressing on the stomach andduodenum. Gallbladder and CBD were normal. A cysto-gastrostomy was performed and the patient felt well andwas discharged. Soon after his discharge he again developedpostprandial fullness and on sonography he had recurrenceof the pseudocyst. Pancreas in the body and tail areaappeared swollen. Main pancreatic duct appeared normalup to mid body beyond which it was not traced. There wasfree fluid in the peritoneum. Amylase was raised. Liverfunctions were normal. He was transferred to our care. Onexamination the boy appeared cachetic and apprehensive.

Page 144: Bedtime gastroenterology

132 BEDTIME GASTROENTEROLOGY

His left upper quadrant was full and slightly tender. Atendoscopy previously done cysto-gastrostomy was notidentified. There was a projection of the pseudocyst in thestomach. The pseudocyst was drained percutaneously undersonography guidance. About 2 litres of maroon colouredfluid was obtained with high amylase and lipase. Asciteswas also drained and it appeared identical to the cyst fluid.In a couple of days his pseudocyst filled up again withreappearance of ascites. Sonography assisted drains wasinserted to drain pseudocyst and ascites. Pseudocystdrained about 500 ml of fluid daily with very high amylaseand lipase concentration. Ascites dried up suggesting itsorigin from pseudocyst by means of leakage. For next10 days he was treated with Sandostatin (Octreotide) butthere was no decrease in the drainage from the cyst.

Q. What further intervention would you consider inthis case?

A. An ERCP was planned to see if the cyst was communi-cating and if so, as was likely, to drain it internally. Pancreaticduct was cannulated and showed rupture of pancreatic ductin the body area with free leakage of the dye in thepseudocyst. A 7 French 10 cm long stent was inserted whichdrained the pseudocyst. Flow was established on table.External drain stopped draining immediate post procedureand was removed after 4 days of no drainage. At 8 weeksthe stent was removed and for further 4 weeks there is nosuggestion of recurrence on sonography. The patient hasregained 12 kg. Weight lost during his illness.

Pancreatic pseudocyst is a frequently occurring com-plication of acute and chronic pancreatitis. It is not related

Page 145: Bedtime gastroenterology

133CASE 68

to the severity of the disease. Nearly 60% of thepseudocysts disappear with time. If they fail to disappearby six weeks and if they are symptomatic and larger than6 cm in size consideration must be given to drainage. Thiscould be either sonography guided external drainage or ifthe cyst is projecting into stomach or duodenum thenendoscopic cysto-gastrostomy or cystoduodenostomy. Attimes if there is lots of debris a surgical cystogastrostomyis required. It is recognised that there are two types of cysts,communicating with pancreatic duct or non- communicating.Communicating pseudocysts is common in chronicpancreatitis, 49%, rather than in acute pancreatitis, 20%there are reports of successful treatment of communicatingpseudocyst by internal stenting by endoscopic retrogradepancreatography, transpapillary cyst drainage (TCD). Theprocedure is done under local anaesthesia and except foroccasional difficult case, takes very short time. Once thestent is inserted, patient could be fully mobile without anyexternal tubing. The response could be quite dramatic as inthis case. Literature reports success rate in the vicinity of80%. Complications are few and mainly in the form ofpancreatitis and infection.

Page 146: Bedtime gastroenterology

134 BEDTIME GASTROENTEROLOGY

CASE 69: A CASE OF RECURRENT DIARRHOEAA 48-year-old male was seen for recurrent diarrhoea. Hewas unwell for past three years with recurrent explosivediarrhoea and vomiting, so much so that he would looseseveral kilograms of weight during these episodes and wouldcomplain of severe weakness and cramps. He was admittedon number of occasions for his diarrhoea. Every time stoolexamination was essentially normal and he would be treatedwith intravenous fluids and antibiotics and would eventuallysettle down. He had a full length colonoscopy which showeda tiny polyp in the caecum but nil else. Barium followthru’and abdominal sonography were both normal Over past threeyears he had lost about 12 kg weight. He was otherwise fitand well and had no other medical problems. Family historywas unremarkable.

On examination he looked well and detailed examinationwas normal. He was advised to come again when he haddiarrhoeal episode. Soon he had one and he was admittedagain. Physical examination was normal again and stoolexamination was normal too. His CBC, Urea, Creatinin andLFTs were normal. There was mild hypokalaemia. He wastaken for jejunal biopsy when we detected multiple duodenalulcers in 1st, 2nd and 3rd part. Multiple biopsies were takento rule out Crohn’s disease. Biopsies did not reveal anygranuloma.

Q. What other tests you would do to come to anyconclusion?

A. Duodenal ulceration was totally an unexpected findingas at no point or even in retrospect he had complained of

Page 147: Bedtime gastroenterology

135CASE 69

any dyspepsia or burning, etc. Zollinger-Ellison syndromewas considered. We carried out both basal acid secretionand fasting serum Gastrin level and both were high. Studiesfor calcium levels, thyroid hormones and blood sugar werenormal thus suggesting isolated Gastrin secreting adenoma.A spiral CT and endosonography of pancreas were carriedout which did not reveal the site of lesion. He was put onhigh dose pantoprazole, 40 mg twice a day and respondedwell.

Page 148: Bedtime gastroenterology

136 BEDTIME GASTROENTEROLOGY

CASE 70: A CASE OF SUDDENONSET OF ABDOMINAL PAIN

A 42-year-old male was referred for sudden onset ofabdominal pain which got progressively worse. He was knowninsulin dependent diabetic for many years. About 3 monthsprior to this he had fever of unknown origin and this wasdiagnosed as enteric fever although single Widal titre wasnot significantly raised. He remained well thereafter till thispresent complaint which started about two days before, inthe morning. The pain was diffuse but mild initially. This wastreated symptomatically. Over next day or so it got worsewith noticeable swelling of his abdomen and small amountof yellow vomiting. He developed high grade fever andbreathlessness and hence the referral.

He drank alcohol heavily till about 6 months before forpast 15 years. On examination he was febrile, tachypnoeicand had severe tachycardia of 140/ min. His abdomen wasdiffusely tender and he had moderate ascites. There was nojaundice. Lungs were clear, peristalsis were present, bloodpressure 100/ 60. Ryle’s tube aspiration was negligible andyellow. Hb: 10 gm., WCC: 17000, with shift to left, Bilirubin:1.2 mg, SGPT: 78 iu, Alk. Phos: 100 iu, amylase: 143 iu,lipase: 200, urea: 45, creatinine: 1.2, blood sugar: 436mg, blood pH: 7.34, pCO2: 38, bicarb: 22 mg. Sonographyshowed fatty liver grade II, gallstones were present butbiliary radicals and CBD appeared normal, pancreas wasnot well seen but there was doubtful bulkiness. Therewas mild ascites on day 1 which had increased tomoderate amount on day 2. Plain X-ray abdomen wasunremarkable. Chest X-ray was clear.

Page 149: Bedtime gastroenterology

137CASE 70

Q. What is your diagnosis? Further tests?

A. Initially we thought he has acute pancreatitis but rapidlyincreasing ascites, only marginally elevated amylase, lipase,and “iffy” sonography report made us think hard. He hadhistory of alcoholism for a long period and we thought hepossibly had alcoholic cirrhosis and spontaneous bacterialperitonitis. A diagnostic tap was carried out which showedcell count of 11,000 with 90% polymorphonuclears. He wastreated with higher antibiotics and insulin and respondedvery well. Later on an endoscopy was carried out whichshowed oesophageal varices thus confirming our initialsuspicion.

Page 150: Bedtime gastroenterology

138 BEDTIME GASTROENTEROLOGY

CASE 71: A CASE OF ALCOHOLICHEPATITIS AND CONVULSIONS

A 42-year-old male was referred for jaundice. He waspreviously well. For last 3 months he started drinking morethan his usual amount and for nearly a month he literallykept drinking whisky whole day. His relatives detectedjaundice and hence he referral. Except for alcohol he hadno other habits. He was of athletic disposition. Past historywas unremarkable.

On examination he appeared well apart from jaundice.Liver was grossly enlarged, spleen was not palpable. Noevidence of chronic liver disease. Central nervous systemexamination was normal. He was started on supportivetreatment and Thiamine and asked to stop drinking andappeared to settle down for a couple of days. Hisinvestigations were bilirubin: 12 mg, 70% conjugated,SGPT: 349, Gamma: GT 860, Alk. Phos: 123 iu PT: 15/13sec. Albumin: 4.0 gm CBC, RFT and electrolytes werenormal. Sonography showed enlarged fatty liver. On thirdday he suddenly had two bouts of convulsions, tonic-clonictype, preceded by an “epileptic cry”. He was givenintravenous diazepam and settled down. Post convulsionshe did not develop any residual weakness and soon regainedhis full consciousness.

Q. What is your diagnosis? Any further investiga-tions?

A. This patient had what is known as “rum fits” He neverhad any history of epilepsy. His is one form of withdrawalsymptom and well known in heavy drinkers who suddenly

Page 151: Bedtime gastroenterology

139CASE 71

stop drinking.’ Typically they occur within 48 to 72 hours ofstopping alcohol and there are generally two or three boutsof convulsions. Diazepam is the drug of choice. Onlyoccasionally they develop status epilepticus. These fits donot suggest latent epilepsy. No investigation is required in atypical case. If in doubt EEG and CT scan may be helpfulto rule out any other aetiology.

Page 152: Bedtime gastroenterology

140 BEDTIME GASTROENTEROLOGY

CASE 72: A CASE OF SUDDEN ONSET OFRIGHT SIDED CHEST PAIN DURING

ENDOSCOPIC TREATMENTA 23-year-old boy was seen for haematemesis. This he hadfor the first time. The blood was coffee ground in colour andin large amount. He was treated symptomatically and withblood transfusion and was referred for endoscopy.

On examination he appeared well. No stigmata of chronicliver disease. Endoscopy showed several large gastricvarices with stigmata of recent bleed from one of them. Hewas explained about cyanoacrylate glue injection to whichhe agreed and injections were started. The first injectionwent well. As soon as the second injection was finished hedeveloped severe left sided pleuritic type pain andtachycardia and tachypnoea.

Q. What do you think is the cause of his pain?

A. This patient had pulmonary embolism. Occasionallyit is seen with cyanoacrylate glue injection for gastric varices.This “glue “polymerizes instantaneously but once in a whileit flows of distally and produces pulmonary embolism. Hehad pleural rub and a wedge shaped shadow on X-ray chestsuggesting pulmonary infarct. He was treated conservativelyand did respond to the treatment.

Page 153: Bedtime gastroenterology

141CASE 73

CASE 73: A 20 MONTH OLD CHILDWHO RETURNED TO THE CLINIC AFTER

A LAPSE OF 12 MONTHSA 20-month-old child returned to the clinic after about ayear of lost follow-up for abdominal distension. She firstcame to the clinic nearly a year ago for abdominal distension.This was noted soon after the birth and got progressivelymore apparent. She had no other complaints.

She was the first child of a non-consanguineousmarriage, born at full term. Pregnancy was uneventful.Antenatal history was unremarkable. There was no familyhistory of recurrent abortions, sudden infant deaths orneurological deficit. Vaccination chart was adequate.

Milestones appeared normal for the age. There were noconvulsions or attacks of unconsciousness. Vision andhearing appeared normal on routine check. She had not learntto crawl. Only history that the parents gave was that shecould not wait for feeding.

On examination she had “doll like cheeks”, veryprominent cheeks, but otherwise face was normal. Abdomenwas hugely distended and liver was grossly enlarged. It wasfirm and non-tender and had a smooth surface and edgesappeared rounded. Spleen was just palpable. Heart soundswere normal and lungs appeared normal. Genitals were of afemale. Musculoskeletal system revealed normal tone anddeep tendon reflexes, and power appeared good. Planterswere flexors.CBC, LFT, RFT and random blood sugar werenormal. Sonography of abdomen showed very large liverwith smooth surface and margin and spleen was mildlyenlarged. Both kidneys appeared normal. Urine for metabolic

Page 154: Bedtime gastroenterology

142 BEDTIME GASTROENTEROLOGY

screening and reducing substances was negative. A liverbiopsy was suggested but patient was lost to follow-up.

She returned to the clinic about a year later when shehad almost similar complaints but now it was apparent thatshe was not walking and could stand only with the support.Abdomen was grossly distended due to huge liver and mildto moderate splenomegaly. Neurologically, there was no focalweakness, but power in the limb muscles appeared grade4. Deep tendon reflexes were present and planters wereflexures. CBC, LFT, RFT, urine were normal. Fasting bloodsugar was 85 mg.

Q. What is your working diagnosis? What would youdo next?

A. With such a large liver and very little in the way ofbiochemical liver function abnormality and associatedsplenomegaly meant only one thing clinically, i.e. storagedisorder. It has to either glycogen storage disorder (GSD) ormucopolysaccharidosis (MPS). Typical face with absenceof azure blue reaction on metabolic screening of urinefavoured GSD. Her history of getting almost violent ongoinghungry, suggested hypoglycemia, which was not proventhough, on fasting sample.

Glucose tolerance test with lactate samples and liverbiopsy and red cell glycogen were next investigations. GTTwas normal. Basal lactate was high. It went higher duringGTT. Liver biopsy showed glycogen storage and evidenceof cirrhosis. Thus with a reasonable certainty it appearsthat this was a case of GSD type IV. GSD are a group ofdisorders which are inherited as autosomal recessivecondition. Since the synthesis of glycogen and its breakdown

Page 155: Bedtime gastroenterology

143CASE 73

requires a series of steps involving various enzymes, wesee more than one type of abnormality and hence varioussub-types of GSD. Type I, II and IV are most common. TypeI involves “final common pathway”, i.e. glucose-6-phosphatase which is the final enzyme which convertsglucose-6-phosphate into glucose. Both glycogenolysis andgluconeogenesis go through this step and hence this is thesevere most with profound hypoglycemia. Type III is almostasymptomatic and type IV produces fibrosis and cirrhosis.Enzyme assays in the fresh liver tissue is diagnostic for aparticular type of GSD. Hepatic adenoma is common inpatients with GSD. Muscle involvement is prominent in sometypes and muscle biopsy may also be useful.

Hypoglycemia requires very frequent feeding andnocturnal tube feeding and feeding with corn starch.

Page 156: Bedtime gastroenterology

144 BEDTIME GASTROENTEROLOGY

CASE 74: A CASE OF INSIDIOUS ONSET OF ASCITESA 45-years-old male was seen in out-patient for discomfortand swelling of abdomen. He had separated from his wifeand was feeling very depressed and lonely although he wasmanaging to have balanced diet. For a few weeks he hadnoticed increasing abdominal discomfort and swelling. Hedenied any cough, cold, anorexia or weigh loss. There wasno evening rise of fever. He did not drink alcohol nor did heconsume tobacco in any form. He was not exposed to anyindustrial or chemical substances. There was no past historyof note and no significant family history.

On examination he appeared well. Here was no anaemia,cyanosis or jaundice. There was no ankle oedema. No lymphnodes were palpable. There were no stigmata of chronicliver disease. Neck veins were not engorged.

His abdomen was distended with gross ascites. Rest ofthe systemic examination was normal. His CBC, LFT, RFT,PT, albumin and sugar were all normal. Three samples forfecal occult blood were negative. USG revealed gross ascitesbut liver and spleen were normal, no varices were seen,splenic and portal veins appeared normal. No enlarged lymphnodes were seen. Chest X-ray was normal too. Ascitic fluidwas straw coloured and slightly turbid, cells 600, mainlylymphocytes, no AFB seen, protein 2.6 gm (S. albumin 4.2gm), cytology was negative for any malignancy. CEA andalpha fetoprotein (AFP) were within normal range. An upperGI endoscopy was normal and no varices were seen.

Q. How will you proceed now? Tentative diagnosis?

A. Biochemical and clinical picture did not fit into anycommon conditions like tuberculosis, cirrhosis, CCF, etc.

Page 157: Bedtime gastroenterology

145CASE 74

When in doubt our protocol is to go for diagnosticlaparoscopy. This showed multiple tubercle like lesions onperitoneum, omentum and liver which were thought to bedue to tuberculosis and pending biopsy result he was startedon anti-tubercular therapy as well although neither clinicalor biochemical picture fitted into tuberculosis. Biopsy wassuggestive of mesothelioma and this was confirmed onimmuno-histochemistry. Even on retrospective questioninghe denied any known exposure to asbestos.

Page 158: Bedtime gastroenterology

146 BEDTIME GASTROENTEROLOGY

CASE 75: A 6 MONTH OLD WITHPROGRESSIVE JAUNDICE

A 6-month-old male was referred for progressive jaundiceand ascites. He was second born, FTND, with one eldersister of 3 years doing well. He was noticed to have jaundicesoon after birth. This was of conjugated type with mode-rately raised SGPT. He was suspected to have neonatalhepatitis. USG showing normal GB, IHBR and CBD. Urineexamination for metabolic screening was positive forphenyl-ketonuria. Screening for TORCH complex showedborderline positive IgM against cytomegalovirus. Noactive steps were taken for these two results. His jaundicekept increasing. Some months later he had screening forhepatitis viruses but A, B, C and E viruses were all nonreactive. He was treated symptomatically at different places.About a week before his admission he developed ascitesand hence the referral.

Liver was slightly enlarged and spleen was just felt. Hehad moderate ascites. No cataracts were found. Neurolo-gical examination appeared normal. Ophthalmic examinationincluding fundus was normal. USG confirmed ascites,normal GB and hepatomegaly with altered echo-texture.Spleen was enlarged. Portal vein was dilated and therewere varices in splenic hilum suggesting portalhypertension. Liver functions showed hepatitic picturewith very prolonged PT. Blood sugar was low at 56 mg.RFT were normal. Blood and urine were collected formetabolic screening. On third day the child went into hepaticcoma.

Page 159: Bedtime gastroenterology

147CASE 75

Q. What is your likely diagnosis?

A. This child had Galactosaemia. Phenylketonuria couldbe falsely positive. Cytomegalo tires are very tricky andunless virus could be reliably isolated, it may be misleading.The absence of cataract is not unknown although presenceof it expedites the diagnosis. This child came to us verylate and could not be salvaged in spite of intensive treatment.

Page 160: Bedtime gastroenterology

148 BEDTIME GASTROENTEROLOGY

CASE 76: A CASE OF JAUNDICE AND CONVULSIONSA 45-year-lady was admitted in emergency because ofjaundice. About 10 days before this admission she was fitand well apart form diabetes mellitus for which she was onregular insulin. Then she noticed slight fever and chills andwas treated symptomatically locally. She remained well apartfrom some headache for a couple of days when the feverappeared again. This was treated with anti-malarials andantibiotics but she failed to respond to this treatment. A daybefore the referral she was found to have jaundice with serumbilirubin of 3 mg, 60% conjugated, ALT/SGPT = 87, Alk.phos =123, and PT = 18/12 sec. Repeated testing for malariawas negative and blood culture and other tests forSalmonella typhi were negative. On arrival she was verybreathless and hypotensive and dehydrated. pH was 7.10and blood sugar was 445 mg%. She was treated for diabeticketoacidosis and appeared to be responding well. Late inthe night she developed repeated convulsions and becamecomatose. CT scan of brain suggested picture consistentwith encephalitis.

Q. What is your tentative diagnosis? How will youproceed further?

A. This was very confusing initially. In tropical countrieswe have to consider malaria and perhaps typhoid in alldifferential diagnosis but tests for these were repeatedlynegative. It appeared that the target organs were brain andliver and hence we thought of herpes simplex encephalitis.This was confirmed both on blood and CSF examination.She was treated with Acyclovir.

Page 161: Bedtime gastroenterology

149CASE 77

CASE 77: A CASE OF RECURRENTABDOMINAL PAIN

A 23-year-old male was seen in out-patient for recurrentabdominal pain. The onset of pain was sudden and fairlysevere, associated with nausea and vomiting and wouldtake a couple of days to settle down. There would beassociated tenderness in the epigastrium. He had variouspractitioners and nearly always had sonography that wasnormal all the times. His blood count would at times showmild polymorphonuclears leucocytosis. His examination wasnormal and hence he was advised upper GI endoscopy.While waiting for endoscopy he had a further attack of painand hence he was admitted.

On examination he had tachycardia and tenderness inthe epigastrium. He had significantly elevated serumamylase and lipase. Sonography after meticulouspreparation showed a cystic swelling in the region of lowerbile duct apart from showing changes of pancreatitis. Atendoscopy also he had a cystic swelling in the second patof the duodenum just above the papilla of Vater.

Q. What is your diagnosis? What would you dofurther?

A. That this patient had recurrent pancreatitis was obviousduring his last admission. Cystic lesion in the region of lowerend of bile duct was thought to be perhaps a pancreaticpseudocyst but the sonologist thought that perhaps therewas a connection of the swelling with the lower end of bileduct and that this could a choledochal cyst. At duodenoscopy,there was a huge swelling almost prolapsing in the first part

Page 162: Bedtime gastroenterology

150 BEDTIME GASTROENTEROLOGY

of the duodenum which was pushed back in the secondpart. Papilla was cannulated and the cannula entered in thecystic swelling and bile was easily aspirated with resultantcollapse of the swelling. A contrast study showed type IIcholedochal cyst. Around 25% of these types of cysts areassociated with recurrent pancreatitis and surgical resectionrelieves the patient of recurrent pancreatitis.

Page 163: Bedtime gastroenterology

151CASE 78

CASE 78: A CASE OF SEVERE DIFFUSEABDOMINAL PAIN

A 52-year-old male was admitted with severe diffuseabdominal pain. This was sudden in onset. Apart from painthere were no major abdominal symptoms. He was previouslyfit and well till about four months prior to this admission,when he was seen for superficial thrombo-phlebitis. Detailedtests were not carried out at that time.

He did not drink alcohol or used tobacco in any form.There were no major illnesses in the past. His appetite wasgood and weight stable. He had no fever. He was passingnormal stool and urine.

Family members were well and had no significant historyof illness.

On examination, he was found to be in agony. Therewas mild tachycardia. Abdomen revealed diffuse tendernessbut no guarding or rigidity. Bowel sounds were feeble.Systemic examination was normal.

On blood examination there was mild lucocytosis buturea, creatinine, liver function tests and blood sugar wereall normal. Amylase and lipase were both normal. Urine forporphyria was negative. X-ray abdomen was showing acouple of air-fluid levels.

An ultrasound was requested.

Q. What is your likely diagnosis? What further testsyou would want to do?

A. This patient had acute abdomen. Tenderness was diffuseand there was no guarding or rigidity. With history of superficialthrombo-phlebitis and near normal tests and pattern of pain

Page 164: Bedtime gastroenterology

152 BEDTIME GASTROENTEROLOGY

made us think of vascular event. An ultrasound with colourDoppler confirmed portal vein thrombosis extending intosplenic vein and superior mesenteric vein. Obviously nowwe had to look for malignancy of nearby organs and ofhaemopoietic system and coagulation parameters. CA 19-9, CEA and alpha feto-proteins were all within normal limits.Peripheral smear was normal too for any haematologicalmalignancy. Urine electrophoresis for myeloma was negative.Anti-thrombin III, protein-S, protein-C and anti-cardiolipinantibodies were all within normal range. Serum Homocysteinlevels was very high. He was put on Pyridoxine and folicacid with regular monitoring of his Homocystein level thatshowed prompt and progressive fall and two years on thepatient has had no complaints.

Page 165: Bedtime gastroenterology

153CASE 79

CASE 79: TWO CASES OF ASCITESOF UNCERTAIN ORIGIN

A 55-year-old female was seen for distension and pain inabdomen. She was feeling unwell or 3 months or so andprogressively noted anorexia, lethargy and distension ofabdomen.

She was previously healthy.On examination she appeared pale; there was gross

distension of abdomen with a vague lump in right iliac fossa.Her ultrasonography showed a lump in the right iliac fossaand ascites with strands septa in it. Remaining bloodinvestigations were normal and chest X-ray was normal.CEA, CA-125 and CA 19-9 were all within or near normalrange. Colonoscopy was normal up to caecum but terminalileum was not visualised.

A diagnostic tap was attempted and it drained a verysticky, mucinous material.

A 28-year-old male presented with right iliac fossa painand was operated by a local surgeon who found a mucinsecreting adeno-carcinoma at the tip of appendix. No lymphnodes were enlarged at the laparotomy. He remained wellfor about here months when he complained of distension ofabdomen. On sonography here was ascites. Attempt atdiagnostic tap with a 23 G needle was unsuccessful.

Q. What are your diagnoses for these two patients?What further investigations you would like to do?

A. It happens so many times in lie that you happen to comeacross rare things one after the other, sometimes on thesame day that you would not see for the whole life. Here we

Page 166: Bedtime gastroenterology

154 BEDTIME GASTROENTEROLOGY

had two almost identical cases of ascites with very stickymucinous material. The second case was, of course, themucin secreting adeno-carcinoma of the appendix. The firsthad a lump in the right iliac fossa and we were not sure if itwas also mucin secreting adeno-carcinoma or pseudo-myxoma peritonei. At diagnostic laparoscopy in the firstcase, both ovaries were normal but a mass was seen in theterminal ileal-caecum area. Special staining suggested thepossibility of pseudo-myxoma peritonei and hence she wasoffered surgical de bulking. At laparotomy the origin of theperitoneal tumour was the tip of the appendix!!

Both the patients did not do well and died within a spanof six months.

Page 167: Bedtime gastroenterology

155CASE 80

CASE 80: TWO CASES OF RECURRENCEOF JAUNDICE

A 20-year-old male was seen for recurrence of jaundice. Hewas previously healthy. Jaundice started wth fever, nauseaand vomiting. He was negative for hepatitis B virus. SGPTwas significantly raised and there were no malaria parasites.He appeared to be settling down when after about 6 weekshe noted increasing bilirubin and SGPT that were comingtowards normal but had not yet reached normal values.

A 25-year-old male was seen with history of jaundice.He was previously fit and well. He had transient fever andnausea at the beginning of his illness but that subsided. Hehad significantly elevated SGPT and he reacted positive tohepatitis E virus. After about 15 days he noted that he washaving feverish feeling and his liver function tests that weresettling down had again started going up. He was negativefor hepatitis B virus, no malaria parasites were seen.

Both patients appeared well and had no stigmata ofchonic liver disease. Liver and spleen were not palpable.Both were negative or Wilson’s disease and anti-nuclearfactor were negative in both the cases.

Q. What is your explanation regarding the recurrenceof jaundice in these two young male patients,previously fit and well?

A. These were two tricky cases. Both had biochemicalpicture that suggested hepatitis and both had recurrence.Hepatitis A is well known for relapse, the cause of which isnot certain. Generally the recurrence is after 30 to 70 days.Therefore, the first patient fitted into the relapse of type-

Page 168: Bedtime gastroenterology

156 BEDTIME GASTROENTEROLOGY

hepatitis. He reacted positive to type-A hepatitis IgM and ina due course of time he settled down as well. For the secondcase the relapse of his jaundice was far too early. He hadreacted positive to hepatitis E. We postulated that he couldhave contracted two viruses. He was tested for hepatitis Aand reacted positive to that as well. So in all probability thispatient had simultaneous exposure to both Hepatitis A andE virus that would explain the clinical and biochemicalabnormalities seen in this patient. Long follow-up of thispatient is available and has not suggested any chronic liverfunction abnormalities.

Page 169: Bedtime gastroenterology

157CASE 81

CASE 81: A CASE OF EPIGASTRICPAIN AND VOMITING

A 34-year-old male patient was referred to kidney institutefor persistent vomiting and epigastric pain. He was a caseof end-stage renal disease of uncertain aetiology and wasbeing considered or renal transplant. He presented initiallyto his physician with vomiting and during the course ofinvestigations he was found to have renal failure for whichhe was referred to the kidney unit. He was taken for dialysispending his investigations for renal failure bur his vomitingfailed to subside and he also mentioned epigastric pain. Hewas treated with proton pump inhibitors but did not respondand hence he was referred for endoscopy. Review of histests showed that he had raised urea and creatinine andsecondary hyperparathyroidism. Ultrasound showed enlargedkidneys.

At upper GI endoscopy he had giant gastric folds. Multiplebiopsies were taken.

Q. What is your diagnosis?

A. This was interesting. He had very thick folds that wethought were because of Menetrier’s disease. The biopsyrevealed amyloidosis. There was partial improvement ofhis vomiting with high dose proton pump inhibitors. The wholeGI tract could be involved with amyloidosis producing variousmanifestations like pain, mechanical or pseudo-obstruction,malabsorption and intestinal ischaemia. There is no specifictreatment for GI tract amyloidosis.

Page 170: Bedtime gastroenterology

158 BEDTIME GASTROENTEROLOGY

CASE 82: A CASE OF SUDDEN APPEARANCEOF A POLYP IN ANAL AREA

A 59-year-old male presented with a polyp like structureappearing for about a month, in his anal area. He had noprevious history of any illness. On detailed questioning hedid admit to straining at stool about a month ago and thiswas followed by pain in the anal area that he described asburning pain that gradually subsided. During this period hehad taken laxatives. Soon after that he noticed a polyp likefeeling in the anal area and hence the consultation.

On examination he looked fit and well. A bluish colouredswelling with a hint of a small stalk at one side was seen onanal margin. This was painless on palpation. Rectalexamination was normal.

Q. What further tests you would do? What is yourdiagnosis?

A. No further tests are required as the story is classical forthrombosis of external venous plexus, a “clotted venoussaccule”. Thrombosed venous plexus assumes firmconsistency and sometimes a bluish discolouration. Thereis no real stalk but the skin attachment gives the hint of astalk. No specific treatment is required except symptomatictreatment in the form of Sitz bath and analgesics. The clottedmaterial may be extruded in due course of time.

Page 171: Bedtime gastroenterology

159CASE 83

CASE 83: A CASE OF ABNORMAL LIVERFUNCTION TESTS FOLLOWING A

LAPAROSCOPIC CHOLECYSTECTOMYA 45-year-lady was operated laparoscopically for chroniccholecystitis. Preoperative sonography had shown normalCBD, normal IHBR and normal bilirubin, SGPT and Alk. Phos.Operation went without any problem and she went home on2nd postoperative day. After about a week she startedcomplaining of right upper quadrant pain. A sonographyshowed normal CBD, normal IHBR and no fluid collection inthe peritoneum. Gallbladder fossa was empty. Her CBC wasnormal, Bilirubin: 0.5 mg, SGPT: 22 iu, Alk.Phos.: was198 iu. She was treated symptomatically by the concernedsurgeon and asked to come for follow-up after a week. Herniggly pain continued and a repeat sonography after a weekshowed normal CBD, normal IHBR, no intra-peritonealcollection and empty gallbladder fossa. Bilirubin: 0.5 mg,SGPT: 40 iu, Alk. Phos.: 267 iu. She was given somepainkillers and asked to return again in two weeks time duringwhich her pain continued in irregular fashion. Repeatsonography showed normal CBD normal IHBR, minimal freefluid in the pelvis and empty gallbladder fossa. Bilirubin:0.9 mg., SGPT: 40 iu, Alk. phos.: 620 iu.

Q. What is your diagnosis? What further investi-gations you would have carried out?

A. Diagnostic tap of ascites revealed bile stained fluid andeverybody’s fear turned out to be true. There were severallapses in the management of this patient. Any pain afterlaparoscopic injury must be considered resulting from the

Page 172: Bedtime gastroenterology

160 BEDTIME GASTROENTEROLOGY

surgery unless otherwise proven. Rising alkaline phos-phatase should have alerted the surgeon to the possibilityof a retained stone or ischaemic or electrical injury to thebile duct and relying solely on the sonography is dangerousand ERCP or at least MRCP should have been done.Retrospectively the surgeon did think that the cystic ductwas short and that the CBD could have been easily damagedby the cautery during dissection. We have seen quite a fewcases subsequently where there is rising alkalinephosphatase without evidence of retained stone at ERCPor MRCP, who have subsequently developed the leak andthis is most likely to be due to electrical injury. We now feelthat when there is such a scenario where there is risingalkaline phosphatase but no stone it will be prudent to stentthe CBD prophyllactically to prevent any leak when there isultimate sloughing off of the damaged portion of CBD.

Page 173: Bedtime gastroenterology

161CASE 84

CASE 84: A CASE OF RECURRENTHAEMOPERITONEUM

A 45-year-old lady was admitted for vague abdominalpain and distension and breathlessness. This startedall of a sudden with pain followed by distension. Onfurther questioning she said that she had this type ofepisodes twice in last two years and that blood was drainedfrom her abdomen on both occasions.

In the past she had vomited blood and she had distalleino-renal shunt some 12 years ago. Liver functions at thattime were normal and liver biopsy had suggested non-cirrhotic portal fibrosis. Postoperation she had remained well.Four years ago she was operated for left ovarian cyst.

On examination there was obvious ascites andsignificant pallor. Abdomen was soft. There was lowgrade fever and tachycardia. Sonography confirmedascites. Liver was normal in echo texture. Spleen wasenlarged. Liver functions were unremarkable. She wasnegative for HBsAg, HCV and Wilson’s disease. ANA wasnegative. Alpha feto-protein was within normal range. Urea,creatinine, albumin and electrolytes were normal. CBC sug-gested normochromic, normocytic anaemia. Platelets weredecreased. A diagnostic tap revealed haemorrhagic fluidthat was negative for acid-fast bacilli on staining andby polymerase chain reaction.

Q. How will you proceed further? What is yourdiagnosis?

A. She had recurrent hemoperitoneum that came all of asudden all the time suggesting intermittent spontaneous

Page 174: Bedtime gastroenterology

162 BEDTIME GASTROENTEROLOGY

haemorrhage. Thus, tuberculosis was put of consideration.Normal sonography and normal alpha feto-protein level ruledout hepatocellular carcinoma in a patient who was negativefor HbsAg and HCV. With a history of laparotomy for leftovarian cyst it appeared to us that there could be adhesionof an intestinal loop of the scar with an ectopic varyxdevelopment and rupture of the same. A diagnosticlaparoscopy was performed which showed normal lookingliver and an intestinal loop which was adherent to the scarand a large tortuous vein on the surface, a natural shunt!!This was ligated. For past three years she had no recurrenceof hemoperitoneum.

Page 175: Bedtime gastroenterology

163CASE 85

CASE 85: A CASE OF PERSISTENTJAUNDICE AND WEIGHT LOSS

A 30-year-old male was referred for persistently elevatedSGPT. He was initially seen elsewhere some eight monthsago with jaundice that was negative for hepatitis B and Cvirus. This was treated symptomatically but his SGPTremained persistently elevated. Soon he noticed progressiveweight loss that over next 4 months amounted to some 20kg. He was investigated for this and was found to havethyrotoxicosis. This was treated with carbimazole withweight loss stabilising and thyroid status coming underolbut SGPT remained persistently elevated and hence thereferral.

On examination he appeared well. Physical examinationwas normal. Thyroid gland was not enlarged. There was noexophthalmos. K.F. rings were not present.

CBC, urea, creatinin, electrolytes and sugar were normal.

Q. How would you proceed now?

A. This was perplexing. Both jaundice and thyrotoxicosisappeared to present simultaneously. We thought of commonaetiology for both the presentations. He could have possiblyautoimmune hepatitis and autoimmune thyroid disease,Graves’ disease. He turned out to be positive for ANA,Antimicrosomal and anti-double strength antibodies. Liverbiopsy showed picture suggestive of chronic activehepatitis.

Other autoimmune diseases are common with auto-immune hepatitis, especially thyroid disease as in thispatient. Other reported findings are pernicious anaemia.

Page 176: Bedtime gastroenterology

164 BEDTIME GASTROENTEROLOGY

Presentation may not be simultaneous as in this case butsequential so it is important to have periodic check. Theremay be family history of autoimmune diseases like diabetes,vitiligo, etc.

Treatment involves treating each condition individuallyas if they were independent entities.

Page 177: Bedtime gastroenterology

165CASE 86

CASE 86: A YOUNG WOMANWITH FEVER AND JAUNDICE

A 32-year-old woman was admitted with fever and jaundice.She was previously fit and well. About 10 days prior toadmission she had noticed fever with chills that was treatedwith anti-malarials and she felt well. Subsequently she kepthaving feverish feeling and soon relatives noticed jaundiceand took her to a nearby hospital where she hadultrasonogaphy and was found to have a liver abscess anddilated Common bile duct and Intrahepatic biliaryradicals and hence the referral.

She was obese and had two children. She never hadany major illness in the past. She never had any abdominalpain prior to or during this illness. On examination sheappeared ill and deeply jaundiced. Temperature was 102 Fand pulse 120/min. There was some tenderness in right upperquadrant.

WCC 17,000, Bilirubin 12 mg. Alk.Phos 900, SGPT 230.Fasting blood sugar was 84 mg. An Ultrasonogram showeda large and several small abscesses in liver and an abscessin spleen. Both CBD and IHBR were dilated. No stone couldbe identified in CBD.

Q. What is your diagnosis? How will you proceednow?

A. This lady had multiple liver abscesses and a singleabscess in the spleen with dilatation of CBD and IHBR andtherefore although she had no identifiable stone in CBD wepreceded to ERCP, presuming them to be cholangiticabscesses. We could have gone for MRCP but we thought

Page 178: Bedtime gastroenterology

166 BEDTIME GASTROENTEROLOGY

against it as she needed drainage irrespective of the causeof obstruction and in view of her age and sex andphysiognomy stone was the most likely possibility. At ERCPshe was found to have a stone in CBD and the largestabscess did communicate with IHBR. A largesphincterotomy was done and stone was removed withoutmuch problem. CBD was repeatedly washed on table and anasobiliary drain was placed as additional safety. Her biliarytree was decompressed on sonography next day andjaundice also decresed significantly but the fever stillpersisted and hence we drained two moderately largeabscesses and continued antibiotics with resolution of herfever. She underwent laparoscopic cholecystectomy aftersix weeks.

Page 179: Bedtime gastroenterology

167CASE 87

CASE 87: A CASE OF BLEEDING PR ANDOBSTRUCTION IN SIGMOID COLON

A 55-year-old lady was seen because of bleeding PR,intermittently for 3 months. Blood was dark red and wasmixed with stool. There was no frequency of stool, nourgency, no fever and no weight loss. Her appetite was goodand her weight was constant. There was no significant pastmedical history or family history. Physical examination wasunremarkable.

CBC showed mild microcytic hypochromic anaemia. Restinvestigations were normal. CEA was within normal limits.Colonoscopic examination showed a very large polypoidlesion in the caecum but it was so large that it was notpossible to go beyond it so it was not possible to see itsattachment with the intestinal wall. Biopsy showed adenoma.Subsequently she was lost to follow-up.

A few days later a Radiologist rang up to say that shehas the lady in his barium room and that the barium is notgoing up beyond the sigmoid colon.

Q. What are your thoughts on what is happening inthis case?

A. It occurred to us that this could be an intussusceptionproducing obstruction in the sigmoid area. With IV Buscopanand head low position this was very elegantly demonstated.She underwent surgery and the findings were confirmed atthe surgery. Fortunately there was no malignancy in thespecimen inspite of its big size.

Page 180: Bedtime gastroenterology

168 BEDTIME GASTROENTEROLOGY

CASE 88: TALES OF TWO OLD MENWITH CHOLESTATIC JAUNDICE.

A 65-year-old man was seen for loss of appetite andprogressive yellowish discolouration. He had past medicalhistory of Hodgkin’s disease some three years ago and wasgiven chemotherapy to which he responded very well andwas declared free of the disease and was on regular follow-up of a cancer specialist. Currently he was on no medicationsfor Hodgkin’s disease. He denied any alcohol ingestion. Hewas on no medications. On examination he was deeplyjaundiced, liver was enlarged and firm, spleen was notpalpable, no lymph nodes were palpable. Systems werenormal. Ultrasonography (USG) showed enlarged liver withheterogenous echo pattern, no dilatation of the biliarytree. No nodes were seen. Spleen was mildly enlarged.Hb: 10 gm, normochromic normocytic picture, WCC:10,000 with 70% polys, ESR: 45. Bilirubin 12, (80%conjugated), SGPT: 67, ALP: 900 iu, GGT: 760.Prothrombine time (PT) 14/13 sec. renal functions andelectrolytes were normal. He was negative for hepatitisB and C. Chest X-ray was normal.

A 56 years old male was transferred with progressiveweakness and jaundice. He also had intermittent confusion.He had injury to his left knee many years ago and was seenby an orthopaedic surgeon for recent swelling in the samejoint and was advised that he needed arthrodesis but nofurther information was available. His appetite haddecreased recently and had lost about 8 kg of weight in last6 months. He also had daily rise in temperature. He deniedtaking any alcohol and was given multi-vitamins by hisphysician. On examination he looked cachetic and had pedal

Page 181: Bedtime gastroenterology

169CASE 88

oedema. He was jaundiced. His left knee was swollen,slightly tender and warm. CNS showed confusion anddisorientation in time, place and person. No flaps or rigiditywas detected, no focal localising signs, fundi were normal.Hb: 10 gm, WCC: 7500, ESR: 110, Bilirubin: 10 mg (80%conjugated), SGPT: 65, ALP: 1265, GGT: 540, Albumin:2.4, Globulin: 1.8, PT 14/14 sec, Ammonia 1 mg (in normalrange), Na 124, K 5.5, creatinine 1. Repeated sonographyrevealed diffuse enlargement of liver and mild enlargementof spleen but CBD and IHBR were not dilated. MRCholangiogram was normal. Chest X-ray and CT brain werenormal. CSF showed non-specific changes. Serology forHSV, HBsAg and HCV were negative.

Q. What would you do further to come to a diagnosis?What is our diagnosis?

A. Liver biopsy was our preferred next investigation.Liver biopsy showed a few non-caseating granulomas inthe first case and a large number of non-caseating granulomain the second case.As Hodgkin’s disease itself couldproduce granuloma with patient’s consent a second biopsywas obtained for culture which confirmed tuberculosis. Insecond case also biopsy was positive for mycobacteriumtuberculosis. He was put on the supervised ATT (DOT)therapy, as in the first case, together with Streptomycin0.75 gm on alternate days. A week on this treatment hefelt well enough to have his knee joint operated forarthrodesis. Synovial biopsy taken at this operation alsoshowed multiple granulomas. He did not have anyadverse effect of anti-tuberculous drugs and his liverfunctions showed progressive improvement with

Page 182: Bedtime gastroenterology

170 BEDTIME GASTROENTEROLOGY

normalisation by 3 months. His serum sodium remainedlow in spite of him having on liberal salt diet and nodiuretics. Adrenal insufficiency was suspected. A shortSynacthen test was carried out which showed impairedresponse and hence he was put on prednisolone andFludrocortisone and his serum sodium level returnedto normal.

Both these cases were admitted almost back to back indifferent hospitals and were having almost identical picture.Both appeared to have infiltrative lesion of the liver assuggested by raised ALP and GGT without any dilatation ofCBD or IHBR. In the second case, there were a large numberof granuloma but in the first case they were not so marked,so the abnormalities in the liver function tests could not beexplained on mechanical effect of granuloma or tuberculouslymph node. This was confirmed by sonography in the firstcase and sonography and MR Cholangiogram in the secondcase which showed no involvement of the biliary tree. Variousinfective and parasitic diseases are known to produce abewildering variety of the liver function abnormality whichmay manifest as isolated elevation of conjugated bilirubinas in falciparum hepatopathy or occasional hepatitic picturein septicaemia or enteric fever and in many of these, role ofendo-toxins or tumour necrosis factor alpha (TNF-Alpha) issuspected. In most of such secondary conditions there isno real liver involvement as SGPT is mildly raised and PTis generally within normal range thus suggesting intact liverfunctions. Literature reports disproportionate rise in ALP insetting of Cholestatic jaundice and it is possible that ALPsecretion in the bile is particularly affected severely. Theexact chemical pathway involved is not known. In both the

Page 183: Bedtime gastroenterology

171CASE 88

cases there was prompt resolution of Cholestatic liverfunction abnormality with anti-tuberculous drug therapy thussuggesting a possible role of chemokines or lymphokines.Both our patients did not develop any hepato-toxicity ofanti-tubercular drug thus suggesting that there is noincreased risk of toxicity from anti-tubercular drugs thanthat in the normal population. So the patient may not bedenied the benefit of full fledged anti-tuberculous drugs. Thisis especially important because many of these patients arevery ill and also have miliary tuberculosis and they need fullATT cover. Usual monitoring must be done nevertheless.Other lesions described in the literature are tuberculomawhich could be mistaken for a mass lesion, tuberculouscholangitis and biliary stricture but these are uncommon.

Page 184: Bedtime gastroenterology

172 BEDTIME GASTROENTEROLOGY

CASE 89: A CASE OF RECURRENTEPIGASTRIC PAIN

A 50-year-old male was seen because of recurrent epigastricpain which would come out of blue and would be associatedwith nausea and vomiting. There was no radiation.Pain wouldlast for a day or so and then gradually wean off. There wasno significant fever. There was no associated burning,haematemesis or melena. He was otherwise fit and welland had no other diseases.

He had several sonographies which were essentiallynormal. Spiral CECT done twice was normal. On severaloccasions he had serum amylase and lipase which wereonly marginally elevated. Bilirubin was normal and SGPTwould just show a small kick. All other routine bloodinvestigations were normal too.

Q. How will you proceed now?

A. Pre-fixed ideas or diagnosis sometimes prevent you fromthinking of alternatives. His story was classical of biliarycolic, however, he was thought to have pancreatitis becauseof marginal elevation of amylase and lipase, near normalbilirubin and SGPT and absence of gallstones or CBD stones.What he required was closer look at the CBD which we didwith endo-sonography and a small stone was promptly found.So this was a case of biliary colic. Sonography could missvery small stones and stones in the lower CBD. He hadERCP with stone extraction and lap.cholecystectomy andover a year now he does not have any episode of pain.

Page 185: Bedtime gastroenterology

173CASE 90

CASE 90: A CASE OF RECURRENT VOMITINGA 56-year-old male was admitted for vomiting. He had long-standing abdominal problem which included abdominal painand intermittent diarrhoea. This was investigated with abdo-minal sonography which was normal, upper GI endoscopywhich was normal too and a barium follow-up examinationwhich was showing extensive small bowel diverticulosis.He had several courses of antibiotics for the same. For lastyear or so his predominant presenting feature was vomiting.

On admission he was dehydrated and looked cachetic.Abdomen was soft and scaphoid. Ultrasonography wasnormal. X-ray abdomen was normal. During his admissionhe vomited large amount of bilious vomit. He did not passstool for two days.

Q. What would you do now?

A. Initially he was thought to have bacterial overgrowth butrecent change to bilious vomiting suggested intestinalobstruction. Normal abdominal X-ray was puzzling. He hadexploratory laparotomy where near the ligament of Treitz hehad intestinal narrowing due to intense fibrosis. This wastreated surgically and the patient made prompt recovery. Itpays to think of dynamic nature of existing problem, i.e.diverticular disease could lead to peridiverticular inflam-mation and subsequently intestinal obstruction like in thiscase.

Page 186: Bedtime gastroenterology

174 BEDTIME GASTROENTEROLOGY

CASE 91: A 15 YEARS OLD BOY WITHJAUNDICE AND BLEEDING PR

A 15-year-old boy presented with fever and malaise to hislocal practitioner. He was fit and well before two days whenhe noticed sudden onset of fever with chills and generalisedmalaise. When this did not subside on taking home remedieshe contacted his general practitioner who prescribed hissome antibiotic and antipyretic. Two to three days lated hewas noted to have jaundice, bilirubin: 13.5 mg. (80%)conjugated, with high SGPT, 890 iu. He was prescribedsome liver tonics. He continued with fever and in a coupleof days he complained diffuse abdominal pain, colicky innature and noticed dark red bleeding per rectum.

On examination he was febrile, 102 F, toxic looking andnauseous. He was deeply jaundiced but CNS was normal.Abdomen was diffusely tender, liver was mildly enlarged,spleen was not palpable and peristalsis were present. Noconjunctival haemorrhage was seen, no peteche wereobserved, heart and lungs were normal. Hb: 8.5, TC 5100,platelets were15000, malaria parasites were negative bydifferent methods. Serum widal and Typhidot IgM wasnegative. Urine for leptospira was negative. Over next fivedays he continued having colicky abdominal pain andbleeding PR.

Q. What differential diagnosis you are consideringin this case? How will you proceed now?

A. This patient had hepatitic picture with low platelets andits consequences. Hepatitis and low platelets are generallyfound in falciparum malaria, leptospirosis, DIC, autoimmune

Page 187: Bedtime gastroenterology

175CASE 91

hepatitis and exacerbation of chronic liver disease andsometimes fatty liver of the pregnancy. This patient hadacute onset of illness in monsoon and hence falciparum,leptospira were on top of the list but both were negative.Lately we have seen patients with dengue fever presentingwith very low platelets and hence he was tested for denguefever and turned out to be positive for the same. Prominentjaundice is not a well described feature of dengue but now itseems it may be kept in mind. Over next five days herequired 28 units of platelet units and gradually settled down.

Page 188: Bedtime gastroenterology

176 BEDTIME GASTROENTEROLOGY

CASE 92: A YOUNG HOTEL CHIEF WITHRECURRENT FLEETING ABDOMINAL PAIN

A 23-year-old man was seen for severe epigastric pain andvomiting. He was not keeping well for about a year and ahalf. This started with him having diffuse abdominal painand loss of appetite for which he was treated by his GP andhe felt better. Some months later he complained of weaknessand was found to be anaemic and this was treatedsymptomatically. A month or so ago he developed right upperquadrant pain, fever with chills and jaundice. This wasdiagnosed as cholangitis but sonography did not reveal anystone. He was treated with antibiotic and felt well. A daybefore admission he had sudden onset of severe epigastricpain with radiation to the back and vomiting.

He was working as chief in a well reputed hotel. He wassingle and used to cook for himself once a day. Onexamination it appeared that he had acute pancreatitis withraised amylase and lipase. Sonography confirmed thediagnosis. Biliary tree appeared normal and so were liverfunction test. All routine tests were normal except micro-cytic hypochromic anaemia. He was created symptomati-cally and setteled down.

Q. Have you reached to any diagnosis? Any furthertest?

A. This was puzzling why; in absence of stones he shouldhave both cholangitis and then pancreatitis. He was anemicalso. We were puzzled but the mother nature helped us.One morning he vomited out a live round worm, ascarialumbricoids. Suddenly the picture cleared. He could have

Page 189: Bedtime gastroenterology

177CASE 92

round worm induced anaemia, cholangitis and pancreatitis.At ERCP a live worm was seen half protruding throughthe pancreatic duct. This worm was gently pulled out withdormia without macerating it and remaining duct was normalwhen injected with contrast and so was the bile duct. Hewas given anthelmentic and instruction on personal hygieneand nearly all his symptoms resolved within a week or so.

Page 190: Bedtime gastroenterology

178 BEDTIME GASTROENTEROLOGY

CASE 93: A CASE OF PERSISTENT DYSPEPSIAA 27-year-old female was seen for persistent flatulence anddyspepsia. She complained of heaviness and bloating. Shesaid that she did not feel hungry and she had intermittentvomiting. There was no history of heartburn, fever, weightloss or any swelling. She had chronic constipation. Therewas no history of bleeding.

For last couple of years she was under treatment of alocal physician who had tried out all antisecretory agentsand motility agents without any benefit to the patient. Anupper GI endoscopy showed mild gastritis but nil else. Testfor H.pylori was negative. Sonography of abdomen wasunremarkable and a barium follow thro’ was normal butappendix was not filled. A gynec reference was nil.

On examination she looked well and except for mildtenderness in right flank, examination was normal.

Q. What is your thinking in this case? What wouldbe your next investigation?

A. This lady had persistent dyspepsia, non-filling appendixon barium and mild tenderness in right flank. Non-fillingappendix, per se, is not diagnostic, but taken in the contextof the case might be relevant. Spiral CT may be helpful. Inour practice we go for diagnostic laparoscopy in such casesas it is more informative than the CT and also offers atreatment if required. We thought we had enough reason fordiagnostic laparoscopy, which was promptly done and revea-led a long, tortuous, retrocecal appendix with adhesions.She reported gradual improvement in her symptoms overnext few days and after about a year of surgery she isasymptomatic.

Page 191: Bedtime gastroenterology

AAbdominal angina 16Abnormal liver functions 4Achalasia cardia 104Achalasia manoeuvre 104Alpha fetoprotein 24Alpha gliadin 13Amyloidosis 67, 157Anorexia 80Appendicectomy 55Artiether 111Aryl sulphate 5Ascariasis 57

BBiochemical parameters 47Bloody diarrhoea 41Bone marrow 67Budd-Chiari syndrome 51

CCeliac disease 33Ceruloplasmin studies 103Cholecystectomy 15Cholestasis 2Chronic liver disease 1Cirrhosis 25Constipation 6Cytomegalovirus 121

DDermatan sulphate 5Diverticular disease 37Doppler study 16Dysphagia 28

EEncephalopathy 40Extrahepatic bile duct atresia 121

FFalciparum hepatopathy 3Fasting calcium 73Flexible sigmoidoscopy 6

GGanesh kriya 6Gastric motility study 97Gastric ulcer 65Gastrinoma 73Gastroesophageal junction 60Gynaecomastia 86

HHaematuria 110Haemolytic anaemia 103Haemoperitoneum 161Haemoptysis 80Heart burn 23Hemiplegia 11Hepatoma 25Hereditary spherocytosis 103Hiccup 21Hypercalcaemia 57Hyperlipidaemia 57Hypochromic anaemia 23Hypogastrium 43

IIntermittent vomiting 129Intestinal tuberculosis 129

Index

Page 192: Bedtime gastroenterology

180 BEDTIME GASTROENTEROLOGY

Intrahepatic biliary 165Iron deficiency anaemia 13

JJugular venous pressure 116

KKayam churn 6Kayser-Fleischer ring 86

LLactose tolerance test 33Liver abscess 165Liver biopsy 67Lymph nodes 4Lymphadenopathy 19

MMelena 36MEN syndrome 81Menetrier’s disease 157Multiple myeloma 67

NNon-caseating granulomas 47

OOesophageal varices 18Oliguria 110

PPancreatic mass 23Pancreatitis 7, 57Papillary stenosis 57Phenylketonuria 147Plasma cell myeloma 67Pleural effusion 9Proctosigmoidoscopy 41

Progressive jaundice 11Protein electrophoresis 67Pulmonary stenosis 116Pulmonary tuberculosis 129

RRectal biopsy 67Recurrent abdominal pain 149Recurrent jaundice 1, 94Recurrent post-prandial vomiting

71

SSecondary hyperparathyroidism

157Serum alpha fetoprotein values

49Solitary rectal ulcer 6Spider naevi 86Spleen 4Supraumbilical pain 21

TT lymphocytes 14Thyrotoxicosis 163Transglutaminase 14Tuberculin test 47

UUnstable angina 9

VVagotomy 32Viral hepatitis 111Vivax malaria 111

WWilson’s disease 74, 161


Top Related